MCQ in Ortho Trauma

Download as docx, pdf, or txt
Download as docx, pdf, or txt
You are on page 1of 353
At a glance
Powered by AI
The document discusses a question bank of multiple choice questions in orthopaedics and traumatology. It aims to address the lack of an authentic source for MCQs in orthopaedics by compiling questions from various examinations across India.

The book contains a compilation of multiple choice questions in orthopaedics and traumatology. The questions have been sourced from various examinations conducted across India and the answers have been researched from the author's textbook on orthopaedics.

The questions have been drawn from various examinations conducted all over India, including All India, AIIMS and PGI entrance examinations.

A QUESTIO N BA NK OF

1\fULTIPLE CIIOICE QUESTlOS h"I"

ORTHOPAEDICS
AND
TRAUMATOLOGY
A COMPENDIIJM OFMCQS lNCORPORATJ.NG
THE LATEST PAl'ERS OF A.LL INDlA, AllMS AND
PGI
ANSWERS RESEARCHEO FROM
NATARAJAN' S TEXTBOOK OF
ORTHOPAEDICSA ND TRAUMATOLOG Y

Prof . MAYILVAHANAN NATARAJAN


M.B.13.S. D. Orth. M.S. Orth. M.Ch. Traoma (Llverpooll Ph Cl.
(Orth. OncoJ D.SC.
Professor of Ortl1opedic: Surgery, Madr'ds Medical College and
Research lnstitutt ,
Tha T.1mil N.)du Or- 1..G.R.. Medical University.
Orlhoped lc Surgeon.Government General HOSp1rol. Chenn,u.
T. milNadu Medical Service, Govemmenl of Tamil Nadu

ct
Jll'lf'fl

.!Jlll '111dla ,Publishers & 'Distributors 'R§gd.


Educat onal
Publishers New
Deihl • Chennal
A QUESTION BANK OF M U LT IPLE CHOICE QUESTIO NS IN
ORTHOPAEDICS A ND TRAUMATOLOGY

Copyright«, Dr.MAYILVA HAN A N NATA RAJAN

All righ ts reserved , No part of this publ ication may be reprod


uced. stored i n retrieval system or transmitted by any means,
electronic. mechanica l, photocopyi ng or otherwise wi thou t the
prior written permissio n of the publisher.

Warn mg :The doing of an unau thorised act ln relation to a


copyright work may result In both civil cla im for damages and
crimina l prosecution.

f irst Ed ition 2005

Published by Rajender K. Arya for ;


jJ /1 'Jndla tpuhlisbers 'Distributors 'R§gd.
Pu bl ishers of Medical & lJent.a l liooks
New Delhi :4380/48, No. 6, Kaushalaya Buildi ng,
Ansari Road, Darya Ganj, New Delh i ·llO 002
Ph .:23244429. 55250321 • Fax :011 - 22467613
E -Mail :aipdraj:J, hotmailcom & a,pd rajla vsnl com
Chennai : Gee Gee Shroff Cen tre. lst Floor,
113. Poonama llee Hig h Road, Chennai-600 084
Ph .:28233004 • Fax : 044 - 28204911

I.S.8.N. No. :81-8004-009-7


Prin ted at : Pashupati Offsets, New Delh i
A QUESTION BANK OF
M ULTlJ>LE CHOICE QUESTI01'S Tl'i
ORTHOPAEDICSAND TRAUMATOLOGY
A COMP£NDIUM OF MCQS lNCORPORA TING
T1JELATEST PAPERS OF ALL INDIA. AllMS AND PGI
ANSWERSRESEARCHED FROM
NA1A' RAJAN'STEXTBOOK OF
ORTHOPAEDICS AND
TRAUMATOLOGY
PREFACE

Asan academician the latesL trends in the examination


syMcm of medical students has always been a fasci nati ng
study. Hence I have been following the ad vent of MCQs
into the medical examination system wi th great iDLerest. Multi
ple choice querions have evolved from a stage of stud ied
disda in Lo grudgi ng acceptance lO a stage where they have
become ubiquitou s. Tht:!ir value in objective testing of I.he
student's grasp of the subject has been widely acknowledged by
academicians lhe world over. So it is not sw-prisi ng to see that
they arc now so entrenched in the sysiem Lhat the Under-
graduate medical sn1dcnt becomes familiar wi lh t hem from his
firsL year iu;elf and is rid of them only after his su per-speciali
ty days are over.
As it has always been my endeavour to help students learn
the fasci nating subject of Orthopaedics from all angles.
Iscoured the various books available for MCQs i.n
01thopaedics. hul the dearth of a single. authentic question
bank for MC(Js wa so glaring that i t prodded me to bring
out lhe first ed ition <lf ..A Question Bank of Multiple
Choice Questions in Orthopaedics a.nd Traumatology...
This. T hope. will address the long-felt need for an
authentic. researched book on MCQs for Orthopaedics. The
questions have been drawn from various examinations
conducted all over India. The questions have been so
segregated that Lhey are arranged i n variou s chapters
correspond ing to Lhe various topics in "Orthopaedics and
Trau matology''. Theanswers have been drawn from !he
author's "Textbook of Orthopaedics and Traumatology" and in
case of doubt, the student is advised to refer to the book. An
added feanire of this book, is thai it incorporates the lruet
MCQswhich have been asked in !herecentpapers of ALL
INDIA. Al!MS, PG! Post-graduate EnU'ancc Exams. This
hook is aimed to satisfy !he needs of nol only I.he
Undergraduates prepari ng for their Orthopaedic Paper or their
PG EnU':lnCe Exam. but <1l so 1hc Post-graduates who are
preparing for their exams.
The purpose of !hi, book would have been achieved 1f
1t
kindles in lhc students an abiding interest in orrhopcdics.

PROF MAYILVAHA..1 AN NATARA JA."I


M.S. Orlb. (Ma's), M.Ch. Trauma ( Liverpool), Ph.D.
(Orth.Onco.). D.Sc
Professor of' Orthopaedic Surgery,
Madras Medical College and Govt. General Hospital,
Chennai
CONTENTS

l ORTHOPAEDICS ·GENERAL l

2, CONGENITALDEFORMITIES Z

3 DEVELOPMENTAL DISORDERS OF BONES 11

4. INFECTIONS OF BONES AND JOINTS 20

5. BONE AND JO!NT TUBERCULOSIS 27

6. GENERALISED DISEASES OF BONES 38

7 DISEASES OFJOINTS 53

8. A VASCULAR NECROSIS OF BONE AND


EPIPHYSEAL OSTEOCHONDRITIS 69

9. TUMOURS OF BONE 72

10. NEUROLOGICAL AND MUSCULAR DISORDER 101

11. REG ONAL CONDITIONS OFNECK AND UPPER LIMB 104

12. REGIONAL CONDITIONS OFTHE


SPINE AND UPPER LIMB
113
13. ADVANCESIN ORTHOPAEDIC
SURGERY APPENDICESIN 121
ORTHOPAEDICS

14. INJURIES TO BONES AND JOINTS 122

15. INJURIES OF THESHOULDER AND ARM 135

16 INJURIES OF THE ELBOW. FOREARM AND WRIST 143

17. INJURIES OFTHE HAND 159


CONTENTS

18 INJURJES OF THE HIP AND THIGH 162

19 INJURIES OFTHE KNEE At"'ID LEG 175


20 INJURIES OFTHE ANKLE AND FOOT 177

21. INJ
URIES OF THE SP NE 180

22 FRACTURE OF THE PELVIS 185

23 POLYfRAUMA 186

24 COMPOUND FRACTURES 187

25 SOFT TISSUE TNJURIE<; INCLUDING SPORT INJURIES


189

26 NERVE INJURIES 193


27 AMPUTATIONS 205
• LATEST PAPERS(MCQ'S) OF
ALL INDIA, AUMS & PGl EXAMINATIONS 206 ·235
a I
ORTHOPAEDICS
1. The Term Orthopaedics was coiued by
= GENERAL

(a) Nicholas Andrey


(/1) Hugh Owen Thomas.
(cl Thomas Bryant
(cl) Sir Roben Jonei,. .!IPM£R 92
2. Which of the following muscles arc stance
phase muscles'!
(a) Quadriceps.
(h) Hamstring muscles.
(c) Anterior tibial.
(d) Peroneus longus.
(e) Soleus -gastrocnemiu. PG! 80.87
3. Ortolani's test is dooc !"or
(a) Congenital dislocmton hip
(h) Dislocalion patella
{r) M.::niscal inju ry of kneejoint
(d) Penhes·disease VI' : J Y09
4. Waddling gait is due to
(a) Gluteal musc e weakness
( b) P.1nrvertebrnl muscl.c weukncs,
(c) Obn1rator nerve palsy
( cf) Add uctor muscle wca lness NIMS : 2000
5. All of the followin g statements abou t a positin•
Trendelenberg's sign m·e true, except
(a) 11 occurs with coxa vara
(b) II occurs w i tll paralysis of hip adduct\11,
{c) ]f it is present on both the sides,the gait can lo1>k
normal
{d) If it is present on one iJe, 1.he patien t ha a lu rching
gait downwards towards the unsupported side

1
6. Antalgic hip gait is related to which of the following
(a) Waddling gail
(h) Trendelen herg gail
(r) Painftil hip gait
(d) Shon leg gait Tl'IMfl NADU : / 99'1

ANSWERS
1. 2. (a,e) 3. (al
(a)
5. (c) 6. (c)
4.
(11)

m I
CONGENITAL DEFORMITIES

I. Which of the following is seen in bilateral congenital


dislocation of hip'?
(u) Waddling Gail.
(b) Shenton·s line is broken.
(c) Trendelenberg lest posili ve.
(d) Allis test posi tive PG!. 90
2. Treatment of CTEV should begin :
(a) Soon after hirth.
(b} After discharge from hospital.
(c) AJler one month.
(d} Ar 2 years. PGl-90
3., Most important pathology in club foot is :
(a), Congenital talonavicular dislocation.
(h};Tightening of Tendoachillcs.
(c) Calcaneal fracture.
(d) Latl:!ral derai1gemem. TN-90
2
4. In a newborn child, abduction and internal rotation
produces a dick sound. Itis know as :
(a) Otorolani's sign.
(b) Telescoping sign.
(c} Mc Murray" s
sign.
(d) Lach man· sign. .IIPMEI? - 80.DELI-/ I
Y3
5. Clu b foot seen in a 15 year old could be trea ted
successfully by a :
(a) Appropriate footwear.
(b) Soft tissue operation.
(c) Triple arthrodesi.
(d) Quadrple fusion. PG/ 78. 83
6. Sprengel's deformity of scapula is :
(a) Undescended I Elevated scapula.
(b) Undescended ne<:k of scapula
(c) Exosll)Si scapula.
(d) None of the above. A/IMS 80.84
'!. Treatment of club foot should begin :
(a) As soon as possible after bi rth
(b) l monlh after birth
(c) Iyear after birth.
(d) None of the above.
PCT 80. UPSC 93. Kam 95: JIPMER 95
8. Treatment for chronic cases of club foot is :
(a) Tripie arthrodesis.
(b) Dorso medial release.
(c) Ampu tation.
(d) None. .TIPMER - 95
9. Phocoruelia is best de.scribed as
(a) Defect in developmen t of long bones.
(b) Defect in developmen t of Oat bones
(c) Defect of intramernbranou ossificalion

3
(d) Defect of cartilage replacement by bone.
ALI,!NOIA 96

4
10. In correction of clu bfoot by ma nipula t ion. which
deformity should be corrected first :
(a) .Forefoot adductjon.
(b) Yarns.
(c) Upper end tibia.
(d) Calcaneum AJ\!
IU.95
11. Child 314 yean is treated for CTEY by
(a) Triple arthrodesis
(b) Postero medial soft tissue release
(r) Lateral wedge resection
(d) Tendo Achilles lengthening and posterior
capsulatomy .
TN Y7
12. Von Rosen Splint in used in
Ca) CTEV.
(/J) CDH.
lC) Fracture shaft uf femur.
(d) Fracture tibia. PGl.97
13. Com monest deformity in congenital dislocation of hip.
(a) Small head of femur.
(b) AHgle of torsion.
(c') De.creased neck shaft angle.
(d) Shallow accrabul u m. PG/.97
14. Club foot in a new born is treated by
(aJ Surgery
(b) Manipulation by the mother
(c) Den nis 13rown splint
(d) Strapping BHU 87
15. Treatment of clubfoot in new born is
(a) Manipulation and corrective splint
(b} Correct ive splin t
(c) Nothing to be done for 6 months
(d) Surgical ANDH RA: 1993
16. In a newborn child, abduction and internal rotation
produces a click sound. It is
(a) 01orolani's sign
(b) Telescopi ng sign
(c) L.achman·,sign
(d) Mc Murray's sign ANDIIRA: / 993
17. Sprengel's shoulder is due to deformity
(a} Scapula
(h} Humerus
(c) Clavicle
(d) Venebra TN 2002
18. Barlow's sign is related to the diagnosis of
(a) Talipes equino varus
(b) Ulnar nerve palsy
(c) Genu varum
(d) Congenital dislocation of the hip. BIHAR./ 91:i8
19. The word "'falipes" rcfcr:s to
(a) Long feet with spidery toe
(bl Flat feet
(c) Club feet
(d) Ham mer toes Blf/AR ./ 988
20. Incongcnittil dislocation of hip, not true is
(a) Real shortening
(b) Telescopi ng
(c) Trendelenburg test
(d} !lead of femu r downwards
(e) Asymmetrical crease AMC: 1989
21. Congenital dislocation of hip is more commonly seen in
(a) Caucasi ans
(b} Negroes
(c) Japanese
(d) Eskimos ANDHRA: 1989

5
22. Rocker bottom foot is due to
(a) Over correcrcd chcb foot
(b) PoliomyeliLis
(c} Vertical talus
(d) DeformiLies of spine
23. Concerning the diagnosis of congenital dislocation of
the Wp at the earliest possible moment in life all are
true except
Ill) A ll obstetricians. midwives and general practitioners
can be ahle to diagnose congenital dislocation or the
hip at binh
(h) Thecondition is diagnos• d at birth by elicit ing a 'click'
or a 'clunk' from 1.he hip
(c) The ign is known as Barlow's or Von Rosen's Sign
(tf) Trendelcnberg's sign is positive
(e) Shenton\ lest is positive

24. Club foot is commone, among


(a) Males
(b) Binovu lar twi ns
(c) Females
(d) Uniovular twins RA.!AST!\N: / 992
25. In a newborn child. abduction and internal rotation
produces a click sound. It is
(a) Owmlani's sign
(h) Telescoping sign
(c) Lachman's ign
(d) Mc Mw.ay's sign DEL/-11: / 991
26. Causes of a painless limp since infancy includes
(a) Congenital dislocntioo of hip
(b) In fantile coax vara
(c) Pol iomyelitis
(dl All of the above. TAMIL NADU / 992
27. Ba rlow's test is done for testing
(a) CDH in child
(b) CDH in infancy
(c) Femoral neck fracmre
(d) Slipped fernoraJ epiphysis
28. Early CTEV is treated by
(a) CTEV cast from l" postnatal day
(b) Mani pulation
(c) Manipulatfon & Strapping
(d) Operative repair as early as possible UP:1999
29. Which of the following test is usefuJ in diagnosis of
congenital dislocation ot' hip?
(a) Bru·low·s test
(b) Thomas test
(c) Hibb' s test
(d) Laguerres test PG! : / 999
30. Sprengc.l's deformity is
(a) Absence of clavicle
(b) Acomioclavicular dislocation
(c) Congenita l elevaLion of scapula
(d) Recurrent dislocation of shoulder UP : 2000
31. Which carpal bone fractu re causes Med ian nerve
involvement?
(a) Scapboid
(b) Lunate
(cl Trapezium
(d) Trapezoid NllvlS : 2000
32. A club foot is
(a) A consequence of placenta praevia
(b) Characteristically associated with breech prcsem-ation
(c) Most commonly of the equino valgus variety
(d) None of the above.
33. The most common congenital anomaly among the
following is encountered in our country
(a) Congenital Pseudoa rthrosis of Tibia
(b) Congenital dislocmion of hip
(c) Congenital 1alipes equino varu.
(ti) Multiple congenital
contrnctu.res TAMIL NADU:1994
34. Madelung's
deformity
involves the
following;
C
a
)

K
n
e
e

(
h
)

W
r
i
s
t
(c) Hip
Cd) .Elbow
TAM IL NADU
: / 994
35. Spina bilida
occulta is
(a) Infection of
the spine
(b) Traumatic
fracture of
the spine
(c) Congeni tal fusion of the body of
the Vertebra
(d) Congeni tal non fusfon of !he
spi nal la mi na
KARNATAKA: 1997
36. The club foot characte:ristically
involves
(a) Foot and ankle
(b) Foot, ankle and leg
(c) Foot only
(d) Foot. ankle. leg and knee joint

37. The most common deformity seen


in club foot is
(a) Talipes equino varus
(b) Equi nus
(c) Equ.ino valgus
(d) Calcaneo equino varus
1999
38. Vat·ious congenital deformity of the
ankle joint occurs
at
(a) Calcaniom1vicular joint
(b) Talocalcaneal joint
(c) Talonavicular joi111
(d) Tarsonavicular join I
1999

8
(b) CDH in infancy
(c) Femoral neck fracture
(d) Slipped fomoraJ epiphysis
28. Early CTEV is treated by
(a) CTEV cast from l"postnatal day
(b) Mani pulation
(c) Manipulation & Strapping
(d) Operative repair as early as possi ble UP:1999
29. \'Vhich of the following test is useful in diagnosis of
congenitaJ dislocation ot' hip?
(a) Barlo,v's test
(b) Thomas test
(c) Hibb's test
(d) Laguerres te t PG! : / 999
30. Sprcngel's deformity is
(11) Absence of clavicle
(b} Acomioclavicular dislocation
(c) Congenital elt:vmion of scapula
(d) Recurrem dislocation of shoulde. UP : 2()()
()
31. Which carpal bone fracture causes Med ian nerve
involvemen t?
(a} Scapboid
(b) Lnnate
(c) Trapezium
(d) Trapezoid NIMS :
2000
32. A club foot is
(a) A consequence of placenta praevia
(b) Characteristically as.sociated with breech prcsent-ation
(c) Most com monly of !heequino valg,1s varie.ty
(d) None of the above.

33. The most common congenital anomaly among the


following is encountered in our country
(a) Congenital Pseudoarthrosis of Tibia
7
39. All of the following arc absolu te indica tions for
radiological evaluation of pel vi-8 for congenital
dislocation of hip, except
(a) Positive family
history (b} Breech
presentation (c}
Shortenfog of l im b
(d) Unstable h.ip ,\!Ti.f S :
J()IJY
40. Talipes equinovarus is
(a) Equinus, inversion. abduction
(b) Equinus, inversion. adduction
(c) Equinus, eversion., abduction
(d) Equinus. eversion, abduction PG! : / <J()7
41. CDH is due to
(a) Large acetabulum
(b) ROllltion of femur
(c) Small neck femur
(d) Small femoral head PG! : 1997
42. Phocomelia is
(a) Absence of short bone,
(b} Complete absence of extr
mities
(c) Defects of Jong bone, of limb
(d) Partial absence of extremities ALL IND/A : / 998
43. 'Pseudoarthrosis' in Triple fusion is seen at the ,joint of
(a) Calcaneocuboid
(b) Calcaneonavicula.r
(c) Naviculocubo d
(d) Talooavicular r>ELHI : / 990
44. Congenital bilateral dislocation of hip shows
(a) Waddling gait
(b) Lordosis

9
(c) + ve Trcndelenburg test
(d)+ vc von Rosen's sign. PG/ : 1990

9
ANSWERS

1. (b. C) 2. (a) 3. (a) 4. (")


5. (c) 6. (a) 7. ltl) 8. (a)
9. (tl) J O. (a) 11. 12. ( bl
13. (a) 14. (b) (b)
15. 16.
17. (a) 18. (d) (a)
19. (C) (a) (d)
20.
21. (a) 22. (c. a) 23. (c) 24. (a)
25. (a) 26. (ti) 27. (b) 28. (c}
29. la) 30. (cJ 31. (b) 32. (b)
33. (c) 34. (b) 35. (d) 36. (aI
37. (a) 38. (c) 39. (b) 40. (b)
41. (d) 42. (c) 43. ( 44. (h. )
45. (c) 46. (/J) t{)
47. (b) 48 ,·(CJ
49. (a) 50. (tf) .

II DEVELOPMENTAL DISORDERS
. OF BONES ,
1.Musculoskeletal abnormalities in neurofibromatosis i
(a) Hype1troph y of I i mb
(b) Scol iosis
(c) Pseudo arthrosis
(d) All

2. Multiple bone fracture in a new born is seen in


(a) Scurvy
(b) Syphil is
(c) Osteogenesis imperfecw
(d) Morquio's syndrome
3. The following is false of Achondropla ia
(a) Autosomal dominant.

11
(h)Mental ret.u·dation.
(c) Due to gene rm1ll1Lioo.
Cd) Shortening of limbs present. JIPMEN - 91
4. Cleidocranial dysostosis may show:
(aJ Wide foramen magnu m.
(b) Absence of clavicles.
(c) Co)(a vara.
(d) All of Lhe ahove. PG/ 81, AMU R7
5. Pseudoarthros
is or tibia is
best treated
by
(a) Internal
fixation.
(b) Internal
fixation
and bone
grafting
(c) Above
knee POP
cast.
(cl) Below
knee POP
cusL
96
6. Not associated
with
osteogensis
imperfecta is
(a) BJue
Sciera.
(
b
)

C
12
a (b} Faulty
t development
a (c) Trauma
r (d) Parathy
a roid tumour
c
L

(
c
l

D
e
a
f
n
e
s
s
.
(d) Fractures.
7. Trident hand is seen in
(a) Achondrophtsia.
(h) Scu rvy.
(c) Mucopolysacchirndosb.
(d) None.
8. Shepherd's Crook deformity is
seen in:
(a) Achondroplas i a.
(b) Gaucher·disease
(c) Hypothyroidism.
(ti) Fibrous dyspbasia.
9. Bone dysplasia is due to
(a) Fau lty nutri tion
12
10. A child with multiple fractures is suffering from
(a) Rickc1s
(b) Scurvy
(c) Hypoparathyroidism
Cd) Osrcogenesis impcrtecta DELHI /98/i
I l. Diaphyseal achlasia is difficult to trc.at because
(a) Causes pressure 011 11erves
(b) Pressure on tendon
(!") ln·egular growth
(d) Causes limitation of movements AIIMS / 989
12. A 9-year old child with high arched palate has
shoulders meeting in front of his chest. He has
{a) Erb\ palsy
(b) Cleidocr.inial dysoswsis
(c) Chondro-Osteodystrophy
(d} Co11ical hyperostosis f:S/ /989

13. The characteristics of l\1orq uio's disease include:


(t1) Spinal kyphosi5
(b) Suhnomial / Normal i ntell igence
(t} Excessive excretion of kcratosulphate in urine
(ti) Dwarfism KERA/ A / 989
(e) All of the above.
14. Absent cahicles are seen in
(al Clcidocranial dysostosis
(b) Achonclroplasia
(cl Moriquo's disease
(d) Oliver'& disease A/IMS / 990

15. Cause of congenital pseudoarthrois is


(a) Intrauterine fracture
(I>) Ncurofibromatosis
[c) Fibrous clysplas a
(d) Unknown. ANDI/RA / 99

i3
16. Ollier's disease is commonest in
(ll) Finger:,
(b) Thigh
(t) Spine
(d) Ribs J)ELHI IY94

17. Wormian bones arc seen in


(a) Osteogenesis imperfccta
(b) Scheurmanns discac
(c) Pagel's disease
(d) O,teocl a,toma DELHI 1994

18. Sclerosis of bone is seen in all except


(a) Secondaries from prostate
(b) Ruorosis
(,') Hyperparalhyroidism
(d) Osteopetrosis KERALA / 994

19. Pscudoarthrosis can be due to all except


(a) Congeni tal
(h) Post inflammatory
(c) Trau ma
(d) None of lhe above PG! / 993

20. Wormian 1,ooes are not noted i11


(a) Fibrous dysplasia
(b) Osteogenesis impcrfccta
(c) Cretinism
(d) Rickets ANDHRA 1993

21. The common features of Neurofibromatosis include


all, except
(t1) Oplic gl ioma
(b) Dumbe.11 neurofibrorna
(c) Scoliosis
(d) Periventricular calcifications ANDHRA 1993
14
22. The features of Achonclroplasia include all, except
(a) Defective head
(b) Mental retardation.
(c) Amosomal recessive
(d) Familial
23. Rone sequestrum is commonly assodated with
(a) Tuberculosis
(b) Nonspecific chronic osteomyc l itis
(c) Pyogenic osreomyclitis
(d) All of the above.

24. Sequestrum is
(tt) A piece of infected bone
(b) A Piece llf dead bone
(c) Organised inflammatory cxudates
(d) Segregated marrow tissue
25. Club foot i$ com moner among
(a) Males
(b) Binovular
twin; (cl Females
(d) Uniovular twins
26. Marble bone disease is
(a) Osreitis fragilis
(b) Osteopctrosis
(c) Ostcosarcoma
(d) Pagel's disease

27. Which of the following is a cliagnostic test for Acu


te Haematogenous Osteomyelitis?
(a) Plain X-Ray
{b) Blood cullure.
(c) CT scan
(d) Clinical examination

15
28. ln which of the following condition bilateral
symmetrical fractures occu r?
(a) R ickets
(b) Osteope1rosis
(c) Osteoge11csis i mperlecta
tdl Fluorosis NIJWS : 2000
29. \\'hich one of the following bone is more susccpliblc for
direct invasion of organisms causing Ostcomyelitis?
(a) Fem u r
(h) Ti bi a
(c) Ulna
(d) rl umerus NIMS : 2000
30. Osteogenesis imperf'ecta is due to
(a) Defective rni ner.ili7.,llion of bone
(b) Fracwre mobilization of calcium from bone
(c) Abnormal collagen defec;1
( cf) Excessive Osteoid fom1ation MANll'AL: /999
31. Themost common site for acute osteom)elitis in infants is
(a) Radial
(b) Fem ur
(c) Hip joint
(d) Tibia NATIONAi. /JOAND : / ()99
32. A short statured patient brought to Orthopaedics OPD
with a X ray showing llattened vertebra with beak. The
probable diagnosis is
(a) Achondroplasia
(b) Ochronosis
(r) Eosinophilic granuloma
(cf) Calve's disease NIMS : 2000
33. Albers-Schonberg djscasc is
(a) OsteopetTosis
(b) Osteoporosis
(c) Osteomalacia
(d) Osteilis derormans PG/ : 2000
34. Not true about Osteogenesi.s imperfecta
(a) Impaired healing of fractu re
(h) Deafness
(c) Laxity of joi m
(d) Fragile fracture UP : 2000
35. Mode of inheritance for Achontlroplasia is
(a) Autosomal dominam
(b) Autosomal recessive
(c) X-linked dominan t
(d) X-linked recessive
36. Congenital pseudo arthrosis is seen in the following
(a) Hip joim
(b) Femur
(c) R adius -ulna
(d} Tibia-fibula TAMIL NADU : / 993
37. 111 X ray, a dead bone looks as
(a) Radiol ucent
(b) Rad io-opaque
(c) Highly rad io-opaque
(d) Non visualized TAMIL NADU : 1993
38. Diaphysial achlasia is related to
(a) Multiple hereditary
cxosl<>sis
(b) Aneurysmal bone cysts
(c} Multiple Neurofi bromamsis
{d) Multiple enchondromatosis TAM IL NADU : / 994
39. Osteogenesis imperfecta is due to the following
(o) Excessi ve Osteoblastic activity
(b) Defecti ve Ostcoid formation
(c) Defective osteoclast function
\d) Defecti ve mineralisaLion of bone
TAMIL NADU : 1994
40. Trident hand is seen in
(a) Mucopolysacchroidosis
(b) Cleido-crania l dysostosis
(c) Metaphyseal acblasia
(d) Achon<lroplasia

41. Following are true about Osteogenesis irnperfecta


except
(a) Uu a l l y green stick fractu re
(b) Generally do not uni te withou t T/t
(t') Fracture frequency decreases during puberty
( 1[) Fracwre seen in trauteri ne life anti <luring panuritio11.
MP:1998
42. Albert Schonberg disease is
(a) Osteopetrosis
(b) Osteitis pu nctati
(c) Osteoporosis
(d) Osteodystrophy PG!: !99fi
43. Osteogenesis irnperfecta is defed in
(a) Bone
(h) Ca.ldfication
(c) Carti.lage
(d) Collagen PG!:1998

44. Not true about Osteogenesis imperfects is


(a) Deafness
(b) Frncturcs
(c) C,1taract
(cl) Blue Sciera
45. All are seen in osteogemesis imperfecla
except
(a) .Blue sclera
(b) Bilateral Hip dislocation
(c) Lax ligament
(d) Osteoporosis AU, IND/A: !998

46. Pseudoa rthrosis is seen in all of the following except


(a) Tdiopathic
(b) Fractu re
18
(c) Osteomy l itis
(d) Neu rofibromatosis ALL IND/A: 1998

19
47. A female with chromosomal defect, cubitus valgus,
and short neck, is most likeEy to bave
(a) Turner's syndrome
(b) Klinefeher's syndrome
(c) Marfon's syndrome
(t{) Sturge Weber's syndrome AMC: / 991J

48. Brittle bones disease is


(a)Osteoporosis
(b) Osteopetrosis
(c) Oteogenesis imperfecia
(d) Osteomalacia NATIONAL BOARD: 1990

49. Osteogcnesis imperlecta


(a) Au tosomal Dominant (AD)
(b) Au tosomal Recessive (AR)
(c) Both AD ,md AR
(d) Sex linked dominan t None of the above. PCT: /
990
50. Multi ple bone fractures are seen in
(a) Kiekets
(h) Osteomalacia
(c) Scurvy
{d) Osteogencsis imperfecta AMC: /99 I
51. A.IIare true of Acbondr:oplasia except
(a) AuLosomal dominant
(b) Shortening of limbs is present
(c) Usually due to gene mutation
(d) Mental retardation. JIPMER: / 99/
52. Jaw tumour is seen in
(a) Osteoporosis
(b) Ostcomalacia
(c) Osteopetrosis
(d) Caffey·s disease Al/MS : / 992
ANSWERS

1. (d) 2. (h) 3. (b) 4. (d)


5. (17) 6. (bl 7. (a) 8. (d)
9. (b) o.
1 (d) 11. (c) 12. (/,)
13. (e) 14. (tt) 15. (b) 16. (u)
17. (a) lit (1•) 19. (/)) 20. (a)
21. (a) 22. (c) 23. (c) 24. (a)
25. (al 26. (b) 27. (b) 28. {cl
29. (b) 30. (cl 31. (d) 32. (o)
33. (al 34. (a) 35. (a) 36. (dJ
37. (b) 38. (a) 39. (b) 40. (d)
41. (b) 42. (a) 43. (a) 44. (c:)
45. (b) 46. (/>) 47. (b) 48. (I>)
49. (c) so. (d) 51. (c) 52. ( ti)

I INFECTIONS OF BONES
AND JOINTS
l. Acute Hematogenous osteomyelitis is treated with all
except:
(a) Antibiotic.
(b) Splinliog.
(c) Analgesics.
(d) Surgery. PG/ 88
2. ·which is false regardimg acute osteomyelitis'!
(a) Staphylococcus is the usual organism.
(b) Rest and elevation relieves pain.
(c) Paremeral antibiot ics arc given.
(d) Surgery is tile onJy treatment. KERALA. 89

20
3. When osteomyelitis disseminates by Hematogenous way
the most affected part of bone is;
(a) Metaphyses.
(bl Epiphyses.
( ,:) Diaphyses
(d) Any of tl1e above JIPMER - 95
4. Tom Smith arthritis manifests as;
(a) Increa. e h i p mobility am!un1abili1y.
(b) Hip Stiffness.
(c) A+ B.
(d) Shortening of limb PAI.'96
5. Bony ankylosis result from
(a) Pyogenic a:thritis
(h) Tuberculosi, anhritis
(c) Osteoarthritis
(d) R heumatic arthritis PG/ 85.87
6. Brodies abscess usually involves
(a) long bone.$.
(b) short bones
(cl Pelv i c
hones. Al 9/
(d) Flac bones.

7. What i Brodie's abscess?


(a) Long standing localized pyogenic basccss in 1he hone.
(b) Cold abscess
(c) Subpcriostea l. abscess
(d) Son tissue abscess Jll'MER 90
8. Commonest cause of hem.atogcnous ocsteomyclitis
(a) Streptococcus.
(b) Staph aureus.
(c) Salmooella.
(d) H. influenza. JIPMER 98
9. Aclinomycosis is commonly seen in
(a) Tibia.
21
(b) Mand ible
(c) Scapula
(d) Femur.
10. Sclerosis of a Jong bone may suggest
(a) Osteoid osteoma
(/,) Sclerosing Osteomyelitis
(c) Both are correct
(d) None of the above. PG! JWJ8
11. The most common cause of Osteomyelitis is (d) E
(a) Staphylococcus aureus sc
h.
(b) Streptococcus haemolyticus
C
(c) Clostrid ium wclchii
ol
i

U
P
S
E
1
9
8
8
12. A 9-
ycar
-old
chil
d
with
high
arch
ed
palat
e ha
shou
lders

22
meeting in front of his chest. He has
(a) Erb's palsy
(b) Clcidocranial dysosrnsis
(c) Chondro-Osteodystrophy
(d) Cortica l hyperostosis
13. Non-healing sinus is a common clinical
fea ture in chronic osleomyelitis. The
most conm1on frequent cause for this
presentation is
(a) Resistant organisms
{b) Retained foreign body
(c) Presence of sequetrum
(cf) lntrnosscous cavilies

KARNATAKA.1988
14. 'Which of the following terms is
i11a1>propriate to the coudition of
Osteomyeli tis?
(a) Cloacae
(b) lnvolucrum
(c) Sequestrum
(d) Myelocoele

23
15. The most common c use or Monoarthritis inChildren is
(a) Septic arthritis
(b) Tuberculou arthritis
(c) Osteoanhritis
(d) Rheu matoid arthritis
(e) Any of the above ANDHJV\:/ 989
16. Acute suppurative arthritis is associated with all except
(a) May be caused by a pe.netrating wonad
(b) May becaused by acompound fracture involving ajoint
(c) May be due to blood borne infection
(d) Causes the joi nt to be held i n the position of ease
{e) Tends to end with the formation of a fibrous a11kylos1s
17. Tom smith's arthritis is due to
(a) Pyogenic infection in infancy
(b) TB
(c) RA
(d) OA PG!: 1999
18. Bony ankylosis occurs in
(a) Tuberculos arthritis
(b) Osieo artbrilis
(c) Rheumatoid arthritis
(d) Pyogenic arthritis MAN /PAL : 1999
19. Wl1ich ncnir hwolves epiphyseal plate'?
(a) Tuberculosis
(b} Osteoclastoma
(c) Acute osteomyetilis
(d) Osteochondrorna J/PM ER : / 999
20. The most common source of bone and joint infection is
(a) Direct spread
{b) Percutaneous
(c} Lymphatic
(d) Haemaiogenous TAMIL NADU : 1994
21. lnvolucrum means
(a) Fragmen t of <lead bone
(b) Hole fonne<l in the bone during the format ion of a
draini ng inus
(c) Ostcornyeli tis of spine
(d) Periosteal new bone forma l ion around necrotic
scqucstrum. MANJPAL: / 998
22. About Se<1ucstrum not true is
(a) ln fection nidus
(h) Lighter than live bone
(c) Dead piece of bone
(d) Heavier than live blme & 1rabeculated . PG/: 1998
23. Blood spread of osteomyelitis is to
(a) Diaphysis
(b) Mctaphysis
(c) Epiphysis
(d) Physeal plate ANDH RA:1997
24. Involucrum is found
(a) Underneath the sequcstrum
(b) Around the seques1rum
(c} At metaphysis
(d) Beneath the pcriosteum ANOHRA: 1997
25. Madura Foot is
((I) Tuberculous i nl'ec1ion of the
bone
(I>) Madura mycotic iofection of the bone
(l') Para ytic infection or the bone
(d) WacJr borne disease of the foot. KARNATAKA:1997
26. Chronic osteomyelitis is diagnosed mainly by
(a) Sequestrum
(b) Bone fracture
(c) Deformity
(d) Brodie's bsces, MANIPAL: /997

24
27. All are associated with chronic osteomyelitis except
(a) Amyloidosis
(b) Sequestrnm
(c) .Metastatic abnormality
(d) .Myositis ossificillls AU IND/A. 1999
28. The most common organism causing osteomyelitis in
drug abusers is
(a) E.coli
(b) Pseudornona s
(c) Klebsiella
(d) Staph. Aureus PG/: /
997
29. When does the lesion of Osteomyelitis appear on X-ray
(a) 2 hours
(b) 24 hours
(c) lweek
(d) 2 weeks DELH I :
199()
30 Acute Osteomyclitis of long bones commonly affects the
(a) Epi phyi
(b) Diapbysius
(c) Metaphysis
(d} Articular surface PG/: / 991$
31. A patient with swelling foot, pus discharge, multi ple
sinuses. KOH smear shares filamenlous structures.
Diagnosis is
(a) Osteomyelitis
(h) Madum Myco,is
(c) Anthrax
(d) Tel.llnus unilateral UPSC: / 990
32. Sabre tibia seen in
(a) Tubercu lous Ostcomyelitis
(/,) Syphilitic osteitis
(c) Rickets
(d) Paget's d isese IV/MS: 1991
33. A dead piece of bone is known as
(a) lnvolucrum
(b) Sequcstrum
(c) Cloacae
(d) All of these BIHAR: 199/
34. Metaphyseal lesion in ch1ldren includes
(a) f racture
(b) Osteom yel il is
(c) Dislocation
(d) Ewing's tu mour
(e) OMeosarcoma PG!: / 991
35. The ideal treatment foracute Osteomyelitis of long
bones is
(a) Antibiotics only
(b) Drilling of bone
(c) Decompression
(d) An ti biotics and delay,cd decompression.
36. Chondrolysis occu1,commonly in
(a) T.B. arthritis
(h) Syphilitic arthri tis
(c) Chondrosarcoma only
(d) Septic arthritis of infancy.
37. Arthritis of tertiary sypllilis most frequently involves:
(a) Shoulder joint
(b) Elbow joi nt
(c) Knee join t
(d) All of these.

ANSWERS
1. (d) 2. (d) 3. (a) 4. (I')
S. (a) 6. (a) 7. (a) 8. (h)
9. (b) 10. (c) 11. Ca) 12. (h)

26
13. Cl') 14. (d) 15. (b) 16. (e)
17. (a) 18 (d) 19. (b) 20. (d)
21. (d) . (d)
22. 23. (b) 24. (h)
25. (b) 26. (a) 27. (d) 28. (al
29. (d) 30. (c) 31. (b) 32. (b)
33. (b) 34. (b,e) 35. (a) 36. (J/
37. (c)

nI BONE AND JOINT


T_UBE_R_C_U_L_O_S_IS_ --'
1. Commonest site of skeletal tuberculosis
is:
(a) Tibia.
(b) Radius.
(c) Humerus.
(d) Vertebrae. Kf:RALA.R8

2. Tuberculosis of the spine is know n as;


(a) Pott' s diseae.
(b) Scheuermann·s disease.
(c) Pe11hes disease.
(d) Frieberg's disease. J!PMER - 88
3. Tuberculous arthritis in advanced cases lead to:
(a) Booy ankylosis.
(h) Fibrous ankylosis.
(c) Loose joints.
(d) Charcots jo ints. TN.89
4. Tuberculosis of the spine starts in:
(a) Vertebral body.
(bJ Nuc leous pulposus.
(c) Ann ulus fibrosis.
(d) Paravertebral fascia PG! N8
27
5. Triple deformity of knee is seen in
(a) Polio
(b) Tuberculo is
(c) ViUonodu l ar synovi Lis
(d) Rheumatoid arthrili UPSC-95
6. Treatmen t of triple defornlity is
(a) AIT
(b) ATT+ lmmobilismion.
(c) ATT + lnunobilisation +
Debridement AMU.95
(cl) None (d) Fu ed
7. The l" sign of TB is sio joi
(a) Nan-owing of in tervertcbral .,pace n nt
(b) Rarefaction of veneh ral bod ies
of spa
spi ce
(c) Destrnction of lam inae
no (b)
us Sof
pr t
oc tiss
ess ue
es. sw
elli
8. Ear ng.
lies (d
t Dc
feat cre
ure a
s of ed
TB mo
ver ve
teb me
ra nts
(a) D
.
ec (d)
re Pai
as n
28
9. Conunonest site of C.U.P.G.l:... (d) Ne
Tuberculosis spine is E.95 ura
(11) C1 - Tr l
arc
h
hu
:/
CUPCJ:: 98
E.'J9 8
(b) T - T
2 6 14. The
(c') Tiii - LI most
(l/) LI- L, commo
JO_ Earliest sign in X-Ray in T.B Spine: n type
of'
(a) Paravertabral shadow.
Spinal
(b) Narrow i ng of disc s1,ace.
Tuberc
(c) Gibbu,
ulosis
(d) Straightening of !.he spinal curves. i.s
11. Commonest site or Tuberculosis Spine (a) A
(a) Dorso lumbar m
cri
(b) Lumbar
or
(c) Sacral
(b)
(d) Flex ion and rotation. ALL Posteri
12. Triple deformity is a /NOIA 95 or
complication of (c)
(a) RheumaloiJ anbritis Centra
(b) Tubercu losis l
{c) Osteoarth ri tis (d)
(d) Septic art hritis Parnd
A/IM
iscal
S 97
h,ir.19
13. In the majority of cases, the tuberculosis 8/i
of the vertebra
starts in the 15. Earliest
radiolo
(a} Body
gical
(b) Pediclc
sign of'
(c) Spine
29
Spinal Tuberculosis is
(a) Wedging of vertebra
(b) Syndesmophyte formation
(c) Formation of parave1tebral absces
(d) Decreased join t space
16. Spina Ventosa results from
(a) Sarcoidosis
(b) Tuberculosis
(c) Histiocytosis X sym.lrome
(d) Both A + B but not C
17. Compression of single vertebra with
narrow joint space
is characteristic of'
(a) Caries spine

30
(b) Frn :ture spine
(c) Prolapsed interve11cbral disc
(d) Secondaries spine. OR!SSA: 1988
18. Shor! long bones of hand and foot are
commonly infected by the following organism
(a) Pyogcn.ic
(b) Tuberculous
(c) Fungal
(d) All of the above TAMILNADU 1988

19. The most common sile of skeletal tuberculosis is


(11) Humcn1s
(b) Tibi!1
(c) Vertebrae
(d) Radius KERALA: 1988
20. Treatment of Pott's paraplegia is
(a) Lateral decompression
(b} Posterior decompression
(<'} Excision or vertebra
(d) Amiruberculous UPSC: / 988
dmgs
21. Caries sicca is seen in
((I) Hip
(b) Shoulder
(c) Knee
(d) None of the above
22. Pa raplegia is common in Tuberculosis of dorsal
spine, because
(11) Incidence of Tuberculosis is com moner in dorsal spine
(b) Natural Leadency for l.'.)'phosis.
(c) Cru1al is naITTJw .
(d) Spinal cord ends al L.1•
(e) All of the Above.
30
23. All are featw·es ofjoint TubercuJosis excep1
(a) Synovium is u1volved
(b) Synovial fluid has < 20% blood sugar
(c) Kissing arth riLis-subchondral bone i, invol ved
(d) Pain is a common feature
(e) Lymphocyte I Monocyte ratio i& dccreaed.

24. lo Antero-lateral dec<,mpression


(aJ Laminectomy is performed
(b) Venebral bodies Me exposed
(c) Anterior longitudinal ligament is exposed
(d) Spinal nerve rnots have to be exposed.
25. Pott's spine is commont:,st in spine
(a) Cervical
(b) Thoracic
(r) Lumbar
(d) Sacral DELHI: 1994
26. instillation treatment in Osteomyelitis is
(a) Cominu·ous suction + conLinuou.s drainage
(b) lntenniue111 uction + continuous drainage
{c) Continuous suction + intermittent drainage
(d) lntennittent suction + intermittem drainage
27. Osteoid formation is normal in all of the following
except
(a) Scurvy
(b) Osteoporosi&
\c) Rickels
(d) Secondaries bone JIPM ER:/ 993
28. The mos!common cau.se of genuvalgum in children is
(a) Rickets
(b) Rheu matoid arthritis
(c) OsteoarthriLis
(d) Paget's disease A/IMS 1993
29. Paravertebral masses include au, except
(a) Tuberculous abscess
(b) Neurofibroma
(c) Oesophageal varices
( rf) Derrnoid cyst ANDH RA 1993
30. Advanced cases of Tuberculous arlbritis may lead Lo
(a) Hyperlaxi ty of joints
(b) Clutton's joi nts
(c) Fibrous anky osis
(d) Bouy ankylosis TAMI!. NADU: 1992
31. The most common cause of Para plegia of early onset of'
Tuberculosis of spine is
(a) Spinal artery thn)mbosis
lb) Sudden collapse of v,c11cbra
(c) Granulation tissue pressing l)n cord
Id) Cold absc,css pressi ng on die cord
KARNATAKA. / 992
32 A psoas abscess present on tlle right side, the corn.-ct
statement is
(a) Opposite hip flexion relieves pain
(b) Same side hip flcxion relieves pain
(c) Same side extension rel ieves pain
(d) Kyphosis may occur NITMS: 1992
33. Earliest feature of spinal tuberculosis is
(a) Gibbus
(b) Muscle spasm
(c) Pain
(d) Psoas abscess ON/SSA:1992
34. The earliest feature of tuberculosis of spine is
(a) Pain
(b) Decreased movemems
(c) Soft tissue swelliJ1g
(d) Night cries WEST BENGAL:

32
1999

33
35. Which of the following causes para vertebral absces. '/
(a} Brucclla
(b) Tuberculosis
(cl Kala-azar
(d) Typhoid r:A.Mfl NAD/J /99')
36. Early features of spinal tuberculosis in a child indudes
(a) Pain ,m sudden movemenl
(b) Gradual deformi Ly
(c) Night crie
(d) Sudden defonuity UP : /999
37. lna fracture shaft of f'emur following accident, first
thing to do is
(o) Splinting
<b) Secure airway and treat the shock
(c) lntcmal fixation of fracture
(d) Physical examination KARNATAKA: 2000
38. In Tuberculosis of spine, which one of the following
is not a cause for Paraplegia?
(a) SLretching of spinal cord in gibbus defomuty
(b) Spi1rnl artery compression
(c) Compression by granulmjon tissue
(d) Oedema of spina l cord NIM S : 2000
39. X-ray showing decreased Intervertebral space a•d
presence of Para vertebral shadow. What could be
the diagnosis?
(a) Tubercu losis of spine
(/>) Ankylosing spondylitis
(c) Eosinophilic granuloma
(d) Multiple Myeloma NIMS : 2000
40. All are true ahout Caries spine except
(a) Most common in dorso lumbar spine
(b) Early onset of paraplegia ha. belier prognosis
(c) Gradual onset paraplegiu hs better prognosis
(d) Usually associatetl with Para spinal cold absccs,
PCJ ·2000
41. Tuberculosi, of spine is best treated by
(Cl ) Anterior fusion
(b) Posterior fusion
(c) Hong Kong procedure
(ti) i\spirmion of Para ;;pinal abscess aiid instillalion of
streptomycin TAMIL NA.DU : 1994
42. All of the following movements of the hip are painful in
a patient with 11soas abscess, except
(a) Extension
(h) Adduction to abd uction
(r) Abduction to adduc tion
(d) f ixed Hexion to fur hcr full nexion
TAMIL NADU : 1994
43. Cold abscess is commonly due to infection of
(a) Skull
(b) Ribs
(c} Spi ne
(d) Sternum

44. Tuberculosis of the spine is thought to originate from


(a) By extension from the para vertebral structu re,
(b) In the cancc:Ilous vertebra l body
(cl In the ligamcntous structmes
(d) rn the nucleus pulposus JJIHAR : 1998
45. Compound palmar ganglion is
(a) Tu berculosis affection of u lnur bursa
(b) Pyogen ic affection of ulnar bursa
(c) Non specific affection of ulnar bursa
(d) U lnar bursitis due t() compound injury

46. The most common cause of Bony ankylosis at hip joint


(a) Rheumatoid anhritis
(I>) Septic arthrilis or hip

34
(c) Tubercular a1thritis
(d) Osteitis dcformans UP:/ 998
47. In Bony ankylosis, there is
(a) Painless, No move1nent
(b) Painful com plete movement
(r) Painless complete movernenl
(d) Painful incomplete movement UP·l':)98
.is. The
most
common
cause of
Kyphosis
in a male
is
(a)
Con
geni
tal
( b-)
Tub
ercu
losi
s
(c)
Tra
uma
(<{)
Sec
ond
aries

NAT
ION
A i.
HO
AU
V:1
997

35
49. Pa raplegia due to Tuberculosis
of spine most rnm monly
occurs at
(a) Cervical spine
(a) Upper thoracic spine
(a) Lower thorac ic spi ne
(a) Lumbar spine

NJ\TIONAL IJOAND: ICJCJ7


50. Bony ankylosis is caused by
(a) Tuberculous arthritis
(b) RJ1cumatoid arthriti
(c) Pyogenic arthritis
(d) Osteoarthritis PG/: / 997
51. The ideal su rJtical t reatment for
Pott's paraplegia is
(n) Laminectomy and
decompression
(b) Anterior decompression and
bone grafti ng
(c) Anterolmen1I decompression
(d) Costotransvcrsectomy
UPSC: / 997
52. Bony ankylosis of a hi p join t
in adults is commonly caused
by
(a) Frnctu res dislocation of the
hip
(b) Tuberculous arthritis
(c} Rh11111atnid arthri tis

36
(d) Pyogenic anhtitis
(e.) Ankylosing spondyl.ilis PGJ: / 990
53. A live year old child is sufferjng from painful
restriction of all movements of hip joi nt, the most
likdy cause in our country is
(11} Congenital coax vara
(b) Tuberculosis anhri1is
(c) PcrLhes' disease
(d) Sequelae or sep1ic artlu·icis of mfoncy
(e) Poas ahscess ORJSSA:/ 990
54. The most ap1iropriate for a 20 year old man suffering
from old tubcrculosb arth ritis of knee with t riple
deformity is
(a} Plaster immobilizat ion
(1,) Join1 clearance and traclitm
(c) Total k nee rcplacemen1
(d) Charnlcy's An hrodesis OR/SSA:/ 990
55. Wandering acetabulu m is seen in
(a) Fracture of acctabul um
(b) Dislocation of femur
(c) Congenital t.lislo<:aiion of hip
(d) Tubcrculosi, c)f bip DELHI: 1Y91
56. Surgical treatment in Pott's spine is indicated if there is
(a) Progrssi ve loss of function i n spiteof mc<lial treatmcm
(b} No impro\'cmcm in motor power in pi te of 3 mon
thsof
trealmcm
(c) There is no i JOJ)rovemcni in fever in 3 mon t hs of
treatmen t
(d) Patient who is an adult or middle aged. PG/: 1991
57. The most common site of skeletal tu berculosis is
(u) Hip + Spine
(b) Knee + Hip joims
(c) Knee jo int

36
(d) Cervical spine /\MC : 1992

37
58. Short long bones are commonly involved in
(a) Pyogenic
(b) T.B
((') Fungal infoctioo
(d) All or Lhc above.
59. The commonest infective lesion of the spine in lndht
is:
(a) Pyogcn ic infection
(b) Fu ngal
(c) T.B.
( cf) Typhoid
60. The earliest change of tu berculous arthritis on X-ray is:
(a) Narrowi ng of pace
(b) Bone densily increases
(cl Rarefaclinn nr bones adjacem 10 jo i nl
(d) All of Lhe above.

I ANSWERS I
l. (d) 2. (£1) 3. (b) 4. (a)
s. (bJ 6. (h) 7. <a) 8. (a)
9. (c) 10. (bl 11. (a) L2. (/))
L3. C a) 14. (d) 15. 16. (ti)
L7. {a) 18. (b) 19.
(d) Cc) 20.
21. (b) 22. ( be) 23. (l') 24.
(ad)(I>)
25. \b) 26. (a) 27. (a) 28. (al
29. (c) 30. (c) 31. (d) 32. (h)
33. (c) 34. (a) 35. (/J) 36. (a1
37. (b) 38. (b} 39. (a) 40. (c)
41. (CJ 42. (d) 43. (CJ 44. (b)
45. (a) 46. (c) 47. (a) 48. (h)
49. (c) 50. (a) 51. (b} 52. (bl
53. (b) 54. ( cl) 55. (ti) 56. (a)
57. (a) 58. (b) 59. <c) 60. (a)
6

.
GENERALISED DISEASES OF
BONES _
I. Hand Schuler Christia n Disease, which is correct?
(a) Proliferation of re1iculo endothelial
cells. Cb) Poa m cells seen.
(c) Punched out lesions in X-Ray
(d) Diabetes iJ1sipidus and Bxophiha lmos present
(e) A l l arc correct. KERALJ\.88
2. Ostcitis fibrosa cystica is seen in:
(a) Hyperpara1hyroidis111.
(b) Hypoparathyroidi sm.
(c) Hypothyroid ism.
(d} Hypcr1hyroidism. (d) Hy
pe
3. Pain in Pagets disease is relieved best by: rth
(a) Simple analgesics, yr
(b) Narcotic analgesic. o
(c) Radiation. dis
m
(d) Calci1011iu.
6. Increas
4. The complications of Paget's disease is:
ed
(c1) Osteogenic sarcoma.
densit
(b) Deafness.
y in
(<') Heart failure. skull
(d) All of !11e above. vanl1
5. Increased bone density occurs in: is seen
(a) Cushing syndrome. in: (a}
(b) Hypoparathyroidism . Hyper
(c) Fluorosi. parath
38
yroidism. l',.I.R9
(b) Multiple mycloma.

A/l1HS-86

Al!MS 87

UP 88
(c) Fluorosis.
(d) Renal Ostendystrophy PD! 9()
7. Deafness in cases of Paget's disease is due
to:
(a) Thickened cranium.
(b) Narrowing of foramina of skull.
(c) Brain compression.
(d) Otosclcrosis. PG! 81. DNIJ 89
8. Not a complication of menopause
(ti) Fracture spine.
(b) Colles fracture.
(c) rracture neck of femur.
(d) Supra condylar rraclure humcnis. K/;l?A IA.95
9. Soft tissue calcification occurs in all except
(a) Hyper parathyroidism.
(b) Sclt!roderma.
(c) Hypenhyroidism.
(d) Hyper Vitaminosis D.
39
KERAIA.95
10. Resorption of the terminal phalanx is not seen in
(o) llyperparathyroidism.
(b) Rei.ter's Syndrome.
(c) Scleroclem1a.
(d) Psoriasi . KERAI A.98
11. What is the diagnostic radiological finding skeletal
Ouorosis
(a) Sclerosis of sacroiliac joi nt.
(bl lntcrroseous membrane ossification
(<:) Osccosclcrosis of vertebral body.
(d) Ossification ofligamencs of knee joinL
.IfPi\lJER.2
K
12. Drug therapy of Paget's disease (Osleitis Deformans)
include all except:
(a) Alendronate.
(b) Et.idronatc.

40
( t) Calci[()ni n.
(d) Plicamyci n. KARNATAKA 200/
13. Sclerotic lesion in the bone is seen in all except
(a) Osteilis librosa.
(b) Osteopetrosis.
(c) M<llorheosto,is.
(d) Caffcy's disease. A I.9I
14. Absence of lamina dura in the alveol us occurs in
(a) Rickets.
(b) Osteomalacia
(c) Deficiency of viLamin C
(d) Hyperparathyroidism TN.9/
15. Ostcomyelitis of Jaw is seen in
(a) Osteomalacia.
(b) Osteopoikilosis
(c) Osteoporosis
(d) Caffey's disease. Al/ MS 92
16. AJI of following conditions may be responsible for
osteoporosis except
(a) Steroid therapy
(b} Prolonged weightle!;sness in spaceship
(c) Hyperparalhymidism
(d) Hypoparathyroidism A/. 96
17. Following are fcatw·es of Paget's cliseae except
(a) Deformi ty of hone5
(b) Secondary ostcosarcoma
(c) Lowered erum alkaline phosphatase
(d) Surgery KARNA?'.96
18. Cal'fey's disease occurs in
(<J) Infants below 6 months.
(h) Above 5 years
(c) Above I 0-20 years
(d) 20-40 years. NI/MS.96
40
19. Most Common site of Rosinophilic granuloma
(a) Rad ius
(b) Femur
(c) Skull
(d) lumbar ve11ebrae M.I'.Y8
20. Calcium content of bone is increased in
(a) Prolonged i.mmobilisastion.
(b} Glucoconicoid adminis1ration
(c} Hyperparathyroi dism
(d) Esu·ogen supplcmcntaiion in post menopausal
women
MAHE 98
21 Vertebra plana is caused by:
(a) Malignancy
(b) Tuberculo i$
(c\ Syphilis
(t[) Eosinophilic granuloma PG/ 81. AMU 88

22. Drug of choice for senile osteoporosis is


(a) Esb·ogens.
(b) Androgens
(c) Calcitoni n
(d) Elhidrona1.:. JIPMER 90
23. Which is con-ect rega rding Hand Schuler Christian
DL ease?
(a) Punched out lesions in X-ray
(b) Prnliferation of reticuloendothelial cells
(r) Diabetes insipidus and Exophthalmos present
( <[) Foam cel ls seen
(e) AJJ are correct. KERALA: / 988
24. Erosion of ungual tufts in terminal phalanges of hand
may be due to
(a) Rheumatoid anhrilis
(b) Gout
(c) Hypcrparachyroidism
(tl} Psoriasis PGl. / 988
25. Action of Vitamin D is
(a) Stimulates bone marrow
(b) Increases calcium loss
(c) Stimulates absorption of calciu
m (dJ Stimulates osteoclasts UPSC./ 988
26. Note seen in Rickets 27. Which
(a) Cupping of ends of bones is not a
(b) lrritability feature
(c) Craniotabcs of
(d) Decreased alkal ine phosphatase Osteitis
fibrosa
cystica?
(a) Fra
ctur
e
(b)
Tewny
(c) Jnc
rca
sed
ser
um
cal
ciu
m
(d) Inc
rea
sed
alk
al
ine
pho
sph
au1
,e

42
ANDHRA.1989
28. The bistopalhologic feature of
Pagct's disease includes
(o) Simultru1cous otencla. tic and
osteoblaLic activity at
place,o,
(b) Osteoclastic resorptio11
(c') Replacement of bone marrtiw
by fibro vascular tissue
(cf) A 11 of the above
(e) None of the above.
29. The type of Bone changes occw·s
in chronic renal failure includes
(a) Osteomalacia
(b) Osteosclerosis
(c) Osteopcn i ,1
(t{) Osteitis-fibrosa
(e) Ostconcc.rosis
30 A diet deficient incalcium will most
conunonly result in
(c1) Osteomalacia
(b) Rickets
(c) Osteoporosis
(d) Osteitis tibrosa ALL

43
31. Renal Osteo dystrophy does not include
(a) Osteo rnatacia
(b) Osteo sclerosis
(cl Osteo porosis
(d) Osteo myli tis
32. Sub-Periosteal erosions of middle phalanges at the
radial aspect is characterL tic of
(a) Hypothyroidism
(h) Hyperthyroidism
(c) Hypoparath yroidism
(d} Hyperparathyroid ism AN DflRA: / 993
33. Rickets of vitamin D deficiency is associated with
(a) Suhperiosteal haemorrhage
(h) R icketi rosary at costochondral junc tion produced by
sublu.xation of sternal plate
(c) Apt todevelop duri:ng growth in tow birth weight infants
(d)Inorganic serum phosphacc conccmrncion +.5-6.5
mg/dl
34. Metaphyseal fracture is commonly seen in
la) Osteogenesi imperfecta
(b) Scurvy
<c) Rickets
(d) None OF.I.Hf:
1992
35. Which of the following is usef ul in the treatment of
Osteoporosis?
(a) Pota.%iu m
(h) Sodium chloride
(c) Calcium
(d) Phophate KARNATAKA: 1992
36. A Bald child witJ1swollen abdomen, byperosteous bones
with mental retardation has
(a) Hypervitaminosis A
(b) Hypervitruninosis D
(c) Down' syndrome
(d) Tuberou, sclemsis

37. All arc sclerosing di orders of bone except


(a) Osteopetro,is
(b) Melorhcostosis
(c) Caffey's
(d) Ostcitis fibrosa AU.INDJA: 199.J
38. Pseudo fracture is seen in
(a) Pseudo parathyroidism
(b) Hypnparathyroidism
(c) Osteo malacia
(d) Osteoporosis OR/SSA · /992
39. Tile following is a radiol(lgical featu re of Skeletal
Fluorosis
(a) lntervencbral ligament calcificalion
(b) Sacral sclerosis
(c) Osteolytic lesion in riel vib
(d) None JIPMER:2000
40. The characteristic finding in osteomalacia i!,
(a) J, p
(b) l Ca
(c) l Ca & P
(d) J. Ca & -1. P UP : 1999
41. Ring sign is seen in
(a) Rickets
(b) Barlow·s disease
(c) Scurvy
(d) Fibwus dysplasia NIMHANS: 1999
42. The metabolic indicator of Rickets is
(a) Low serum Ca
(b\ Low P04
(c) Alkaline phosphatase
(d) 1' Urinary hydroxy proti ne NT.Iv/ HANS : ]')99

44
43. Spina ventosa is seen in
(a) TB
(bl Leprosy
(c) RA
(d) Sarcoidosis
44. A young patient presents with enlargemen t of costo
chondral junction and with the white line of .Fraenkel at
the metaphysis. The diagnosis is
(a) Scurvy
(b) Rickets
(c) Hypcrparathyroidisrn
(d) Osteo nrnlacia K/\RNA TA.KA: ?OOU
45. Which of the following is a primary defect in Paet's
disease?
((/) Osteoblast
(b) Osteoclast
(c) Otea<;yte
(d) Fibroblast NA71UNAL BOARD .
2000
46. All of the following f ractu res are associated with
osteoporosis except
(a) Colics· fractw·c
(b) Fn(cturc of cl av icle
(c) Vertebral fracture
(d) Fracture neck of femur MANIPAL: f 99Y
47. Osteoporosis is characterized most commonly b)'
Fracture vertebra
((I)
(h) Backache
(c) Bowing or legs
(d) Abdominal pain ALL INDIA: 199./
48. Paget's disease is also known as
(a) Osteitis fibrosa cystica
(b) Osleitis defonnans
(c) Osteoporosis
(d) Osteopetrosis PGJ: 2000
49. A 65-Year-old female paLlent presents with osteoporosis.
The treatment started with
(11) Calciu m + Oetrogen + Prosgesteronc
(b) Oestrogen + Progestrone
(c) C,llciu m + Octrogen
(d) Calcium UP: 2000
50. Subperiosteal erosion is seen in
(a) Scurvy
(b) Hyperparath yroid ism
k) Hyporarathyroid ism
(d) R ickets NATIONAL 80.4RD : 199./
51. Hormone required for skeletal maturation
(o) Th yroid
(bl Testicular androgen
(c) AdreoaJ steroid
td) Aldosteronc PG! : 1994
52. The cause of ostcomalacia is
(a) Phcnytoin
(I>) Melabsorpt inn
(c) Vincristine
(r.l) Cushing' syndrome PG!: 199./

53. Soft tissue calcification with hypcrcalcaeruia is abser ved


in
(a) f-Iyperparathyroidism
(b) Alkaptomtria
(cl Gout
(d) cuhi11g'S d isea.e Bilwr ·1998
54. The most common site of" Eosinophilic granulom11 is
(a) Skull
(bl Radi us
(c) Femur
(d) Lumbar vertebrae M P: 1998

46
55. The most common site of Eosinophllic granuloma is
(a) Skull
(b) Radius
(c) Femur
(d) Lumbar vertebrae M P:1998
56. Vitami n C deficiency leads to
(a) Defective mineralisation
(h) Defecti ve Ostcoid formation
(cl No1111al collagen am.I Bone ma trix
(d) X-nt)' shows nom1al evide111:e N!MS:1998
57. Caffey's diserusc is
(a) Renal osteodystrophy
(b) fofantile cort ical hypcroscosis
(c) Ostcomyelitis of jaw in children
(d) Chroni1: nsleomyclitis i n children PCl:1998
58. Osteoporosis is seen in foUowing
except
(a) Old age
(b) Prolonged Hepmin adminisrrntion
(c) Hyperthyroidism
(d) Steroid intake.

59 All of the following statementsregardiog Pagct's disea. c


are correct except
(a) Females are affected more than males
(b) It frequently leads 10 Osteogcnic sarcoma
(c) Senun alkaline phosphates level is increa;cd
(d) All called as Osteitis defomians MANJPAL: 1997
60 Calcitonin is used in the treatment of
(a) Post menopausal osteoporosis
(b) Malignant hyper calcernia
(d Pagel's disease
(d) All of 1hc above. MANll'AL :1997
61. A 67 year old man on biochemical analysis found to ha
ve three fold 1ise of level orserum alkaline phosphatase
that of' upper limit of norm v alue during a routine
checkup but sernm calcium and phosphorous
concentra tion and liver. function test results and
normal He is asympto matic. The probable cause is
(a) Multi ple myeloma
( /J ) Pager' disease of bone
(c} Primary hyperparachyroidism
(d) Ostcoma lacia AIIMS:/ 999
62. Vitami n required for coHagen is
(a) Vitamin A
(b) Vitamin C
(c) ViLan1in D
(d} Vitamin E PG!: J9':J7
63. A scooter is hit from behind.The rider is thrown off
and he lands with his head hittiing the kerb. liedoes not
move, compl:'tins of severe pain in the neck and is
una ble to turn his head. Well-meaning onlookers rush
up to him and try to make him sit up. What would be
Lhc best course of action in this situation?
(a) He shou ld be propped up and given some water lO
drin k
(b) He should not be propped up but twnc<l on his face
and
rushed to the hospital
(c) He shou Id be turned on his back and a upport should
be placed behind
(d) He should not be moved at all but canied to the nearest
hospital i n the same position in which he has been since
his fall. UPSC:
1997
64 Paget's disease
(a., Is pai nless
(I>) Begins as an osteoblatic
(d Is frequcncly complicated by osteogen ic sarcoma
(d) Docs nm affect bones singly (e.g. Only 1he clavicle)
(e) Is rare under the age or li)rty. BIHAR: / 990
65. Which of the following is not true regard ing
Pager's disease of bone'!
(a) Serum alkaline phosphatase is low
(b) It freq uently lead w 0St!:!Ogenic sarcoma
(c) Males are affected more than females
(d) Tnvolved area shows rapid forrnmion and resorption of
bone. KERALA: 1990
66. In Scurvy tl1ere is deficit in
(a) Collagen
(b) Fibroblasts
(c) Elastic fibres
(d) All of the above NATIONAi.BOA RD: / 990
67. Manifestations of Fluorosis includes
(a) Stiffness of bad. ligaments
(h) Caries Leeth (due to deficiency of fluorine)
(cl Genu valgum
(d) Dental changes
(e) Stiffness of bones and tendon PG/: / 990
68. The most common cause of bilateral Genu valgum in
our country is
(a) Tuberculosis
(b) Multiple trauma
(c) Pol iomyelitis
(d) Osteoporosis
(e) Rickets PG/: 1990
69. The commonest cause of ostseomalacia in our country
i
(a) Repeated systemic infections
(b} Idiopathic steatorrhoca
(c) Dieteic deficiency of vitamin D and calcium
(d) Poor protein intake
4
9
(e) Chronic renal disease. PG/: 1990

4
9
70. The most common site of fracture of the f'ollowi ng bone
in senile osteoporosis is
(a) Venebra
(b) Neck of femur
(c) Radius
Id) Shaft of fcmur TAM!LNAU: / 990
71. Calcitonin is used in
(a) Paget"s disease
(b) Hyperparathy roidism
(c) Osceosclerosis
(d) V itam i n I) intoxication . AJ!MS: / 991
72. Bony lesion in Ril'ket.s b due lo
(a) Increased parathorumone leading to osteodastic acti vity
(hJ Reduced dietary calcium absorption
k) Phosphaturia
(d) Increased calcium excretion /\llM S:/ 99!
73. Action of Vitamin D is that it
(a) Stimu lates bone ma.rrow
(b) Increases calcium loss
(c) Stimu lates absorption of calcium
(,/) Stimulates osteoclasts ,IMC: !991
74. A patient presents with bone pains and on investigating
calcium and phosphorus levels in serum were found to
he norm al except with elevation of serum alkaline
phosphatase. The 1>robablc diagnosis is
(a) Osteomalacia
(b) Pc1get ·s disease
(c) Osteoporosis
(d) H yperparathy roidisrn ANDHIU:/ 99I
75. In hyperparathyroidism bone resorption is seen in all
except
(a} Jaws
(b) Metacarpa ls
50
(c) Ribs
(d) End of long bones. ORfSSA: 1991
76. Multiple osteolytic lesions in a 2 year old child in skull
and long bones are due to
(a) Neuroblastoma
(b) Hjstiocytosis - X
(c) Wilm's tumour
(d) Mulliple Mycloma
(e) Thalasscmia ma.ior PGf: / 991
77. Hypercalcemia occurs in
(a) Multiple Myelorna
(b) Hyperparathyroidism
(c:) Vitamin A intoxication
(cl) Ilea! resection
(e) Thia1:ide diurelics !'GI: 1991

78. Generalised osteopcnia


({l) SCUf\'Y
(b) Pagcl°s disease
(c:) Osteoporosis
(d) Steroid,
(e.) Osteogencsis PG!: / 991
impe1tecra
79. Ostcomalacia canses include
(a) Phcny toin
(b) Malabsorption
(c) Indoor stay
(d) Vincristi ne !'GI:/ 991
(e) Cushing's syndrome

80. \¥hich is not true about Paget's disease of bone'?


(a) Oen there is no ymptom
(b) Alkali ne phophates i low
(c) Thickening of hone due to widening of concx
(d) All of these.

51
81. Which is I are X-ray finding of infantile scurvy'?
(a) A dcne li ne between metaphysis and cpi phys,a l
cartilage
(b) A clear band of rarefaclion on the diaphysiaJ side
(c) Both of these
(J) None of these.
82. ln nutrional rickets changes occurs at the :
(a) Metaph ysis
(b) Epiphysis
(c) Diaphysis
(d) All of these.
83. Which is true about Osteomalacia?
(a) ALkaJine phosphatase is raised
(b) Seru m phosphate is low
(c) Serum caJciu m is low or normal
(d) A II the above.
84. Osteitis fibrosa c.ystica is due to
(a) Paget's disease of bone
(b) Polyostotic fibrou cJysplasia
(c) Parathyroid adenoma
(it} All of the above.

I ANSWERS I
I. (11) 2. (a) 3. (d) 4. (1{)
5. (c) 6. (c) 7. (b) 8. (d)
9. (c) 10. (a) 11. (b\ 12. (ii)
13. (a) 14. (c) 15. (d) 16. (d)
17. (c) 18. (a) 19. (b) 20. (d)
21. (d) 22. (a) 23. (e) 24. (d)
25. (c) 26. (d) 27. (b) 28. (d)
29. (a) 30. (a) 31. (bd) 32. (d)
33. Cb) 34. (a) 35. (c) 36. (b)
37. (d) 38. (c) 39. (a) 40. (d)
41. (c) 42. (a) 43. Ca) 44. (h)
45. (b) 46. (h) 47. (a) 48. (b)
49. (c) so. (b) 51. (h) 52. (b)
53. (a) 54. {c) 55. (c) 56. (IJ)
57. (/J) 58. (c) 59. (a) 60. (c)
61. (b) 62. (b) 63. (c) 64. {Cd)
65. (Cl) 66. (a) 67. (ad) 68. (el
69. (c) 70. (a) 71. 72. (h)
73. (c) 74. (b) (11)
75. (ii) 76. (£')
77. (a) 78. (cd) 79. (abc) 80. (al,)
81. (C) 82. (a) 83. (d) 84. (c)

111
(a) T
u
b
DISEASES OF er
c
ul
o
si
JOINTS s.
(b) R
I . HLA B27 isjlSSodated with; h
(a) Rheu matoid an hriti,:;. e
(b) Ankylosing spondylitis. u
(c) Rheumatic arthri tis. 1
(d) Gouty anhritis. 1
1
2. Calcification of menisci is seen in: at
(a) Hyperparathyroidi snn. oi
(b) Pscudogout. d
(c) Renal Osteodystrophy. ar
(d) Atromegal y. th
ri
3. Bamboo spine is seen in; ti
s.
(c) Ochronosis.
(d) Anky losing spondylosis.

Al.89.

A.i.X9.

JTPMER.88
4. The following is inYoJved in Rheumatoid arthritis:
(a) Synovial fluid.
(bI Synovi al membrane.
(c) Canilage.
(cf) Subchondral bone. UP.88
5. Increased density of skull vault is seen in:
{a) Hyperparathyroidism.
(b) Multiple Myeloma.
(c) Fluorosis.
(d) Renal osteoclystrophy. PGl.90
6. Soft tissue calcifica tion around the knee is seen in :
(a) Scurvy.
(b) Scleroderma.
(c) Hyperparathyroidism.
(d) Pseudogout PGl.90
7. Osteo arthrosis does not affect
(a) Knee joint.
(b) Hip joint.
(c) [merphalangeal jo int.
(d) MetacarpophaJangeal joim.
(e) Shoulder Joint. KERAL4.96
8. Ostco arth.-ifo foUow"ing is not a predisposing ractor
(a) Diabetes mellitus.
(b) Defective jo i nt Po,i tion.
(c) Weight bearing jt1ints.
(d) Incongruity of articular surfaces.
(e) Old age. KERALA. 96
9. Treatmen t of Osteo arthritis include all except:
(a) Graded muscle exercises.
(b) Replacement of anicular surfaces.
(c) Correction of defonnities.
(d) lncrease the weight bearing by I.he affected joint.
(e) Rest to the jo i n t in acute phase. KERALA. 96
54
10. Treatment of Rheumatoid arthritis include all except
(a) Give rest to the joint.
(b) Con-cction of deformities.
(c) Synovcctomy
(d) Exercises.
(e) lm muno suppressive drugs. KERALA.96

11. Diabetic charcot's joint affect most commonly


(o) Knee.
(b) A nkle.
(c) Hip.
(d) Fo<:1t joint. ALL INDIA.97
12. Disease where dist.al lnterphalangeal joint is
characteri- stically involved.
(t1) Psoriatic arthritis
(b) Rheumatoid.
(c) SLE.
( r[) Gout. TN.99
13. The joint commonly involved i11 sphilitic artbritis is
(a) Hip
(bl Shoulder
(c) Wrist
(rf) Knee

14. The cause of rheumatoid arthritis is


(a) Familial
(b) Immunological
(c) lufecli ve
(d) Traumatic PG/ 88
15. Bleed ing intojoint cavities is not common in
(a) Hemophil ia.
(b) TTP.
(c) Christmas disease.
(d) None of tbe above. .!IPMER 90

55
16. Para-articular erosion are most commonly seen in
(17) Osteoarthirtis
(b) Rhcu111atoid anhritis
(t) Gout
(d) Acu te suppurative arthritis D/:,Ll/1 96
17. Positivity of HLA B 27 in Ankylosing spondylitis
(a} 10%
(b} 96o/
(c) 78%
(d) I00%. NI/M S 96
,
18. Earliest rad iological sign of the Osteoarthritis is
(a} Narrowi ng of joi nt space
(b) Ostcophytc fomia1ion
Cc) Cystic lesion in canccllous bone
(d) Sclerosis in subchondral bnne IJIHAR : / 988
19. The early X-Ray changes of Ankylosing spondylitis
would he
(a) Disc space narrowing
(b) Anterior osteophytc formation
(c) Sarcoiliac joint eros ion
(d) Facetal joi nt ankylosi, AMU.70
20. The earliest manifestation of AJkapton11 ri:. is
(a) Ankylosis of lumbo-dorsal spine
(b) Ochronotic arthri tis
k} Prostatic calculi
(d) Pigmentation o[ tyrnpanic membrane
(e) All of the above

21. Swan neck deformity is a feature of


(a) Syphilitic arthritis
(b) Gou ty arthri tis
(r) Rheumatoid arthritL
(d) Osten arthritis
22. Terminal interphalang.eal joints of hands are
commonly involved in
(a) Psoriaric anhropatt:y
(b) Rheu matoid mtbritis
(c) Still's d sca;;e
(d) Ankylosing spondylici,
23. Charcot'sjoints have all of the following
characteristics except
(a) Copious effusion in thejo i nt
(b) Painful limitation of jo int movemen1
(c) Hyperrnobili1y of jo
int (,/) Osteophytc
fom1ation.
24. Pseudogout has crystals of
(a) Sodium Pyrophosphate
(b) Monosodium urnte
(c) Calci u m Pyrophosphate
(d) Sodium urace.
25. ln Fracture acctabulum, late complication is
(a) Os1coanbrius
(b) Tru·dy sciatic nerve palsy
(c) Recurrent Dislocation
(d) None or lhe above. DF.!HI · 1994
26. Tn Fracture neck of femur in old persons is best treated
by
(a) Replacemem artbroplasty
(b) Thomas's splint suppon
(c) No treatment
(11) lmernal fixation with SP nail KARNATAKA: 1994
27. Congenital dislocation of hip is usually due to
(a) Shon femur neck
(b) Small femur head
(c) Displacement of capital epiphysis
(d) Large acernbulum PG! 1993
28. In Reiter's disease, untrue is
(a) Conjunctivitis
(b) Ulcer on palm & soles
(c) Interstitial lung disease
(d) Af1er sexual contact. Al/MS J9CJJ
29. The most common arthritis that affects lhe wrist is
{a) Osteoarthri tis
(b) Tuberculous anhrili,
(t) Rheumatoid arlhritis
(d) Gou L ANDHRA: 1991
30. Periosteal reaction is not common in
(a) Syphilis
(b) Gout
(c) Osteomylitis
(d) Tuberculou dactyhlis. ANDHRA / 993
31. Hypertroph.ic osteoarthropatby is best observed in
(a) Carcinoma of lung
(b) Mesm:helioma of pleu ra
(c) AV M of tumg
(d) Cirrhosis of liver ANDHRA: 1993

32. Erosive arlhritis is not noted in


(a) A myloi<losis
(b) Hyperparathyroidi sm
(c) Psoriasis
(d) Sickle-cell disorder ANDHRA / 993
33. Joint least affected by Ncur<1pathy
(a) Shoulder
(b) Hip
(c) Wrist
(d) Elbow DELHI J 99J
34. Haemartl1rosis witl1prolonged bleeding time is seen in
(a) Haemophilia

58
(b) Christmas disease
(c} Von Willebrand' s disease
(d) All or lhe above. TAMlL NADU 1929
35. Ankylosingspondylitis thejoint least commonly involved
is
(a) .Elbow
(b) Sacroi liac
(c) Ankle
(d) Spinal Al/MS: 1993
36. AD are trne about gouty arthritis except
(a) Arthritis is rnanifcslcdafterlong attack of hyperu ricemia
(b) TI1ere is good correlation between level of uric acid
and severity (>f gouty arthritis
(c} Synovial analysis i diagnostic
(d) Allopurinol is u-eatmem of choice in acute gout.
All MS: 1993
37. A ll of the followi ng are well know n featu res of
Rheumatoid arthritis except
(a) Bilateral hip artlu-ilis
(b) Erosion of distal interphalangea l jo ims
(c) Pleural effusion
(cl) Hypocomplemcntemia PG/: 199]
38. Which of the following structures prevent hyperextension
of hip?
(a) lliofemoral ligament
(b) Ischicifemoral ligament
(c) Pubofemoral ligament
(d) Pu boischial l igamen t AIIMS: 1<.J94

39. Which of the following is not characterized by bony


lesions?
(a) Gout
(b) Psociasis
(c) SLE
(d) Rh. Anhritis AJJM S:1994
40. The most common cause of Neuropathic joints is
(a) Leprosy
(b) Diabetes
(<;) Rheumatoid anhritis
(d) Syphil is JIPMEI?: / 993
41. All of the following are well k now n features of'
Rheumatoid arthritis except
(o) Bilateral hip artlmtis
(b) Erosion of distal intcrphalangeal joinL
le) Pleural effusion
(d) Hypocomplcmcntemja PG/ :
1992
42. Which one of the following is not associated with HLA
B27?
(a) Ankylosi ng spondylilis
(b) Rcitcr's syndrome
(c) Sjogrcn·s syndrome
(d) Psoriatic aithritis UPSC : 1999
43. Tertia ry sypbilltic arthritis most frequently involves
(a) Spine
(b) !lip
(c) Ankle
(d) Knee TAMIL N.4 DU : 1999
44. Neuropathic joint of foot and ankle is most commonly
due to
(a) CTEV
(b) Hansen's disease
(c) Polio
(cl) Myceroma TAMIL NADU . / 999
45. Swan-neck defonnity is
(a) Flex ion of Mctacarpopha.langcal joi nl and extension at
interphalangeal joint
(b) Extension at ProximaJ intcrphalangeal joint and flexion

60
at Distal interphalangeal joint

61
(c) Flexion at prnx imal interphalangealjoint and
extension
at distal interphalangeal joint
(d) Extension at Me1ncarpophalangeal joi nt and llexion at
imerphalan gea l joint NIMS: 2000
46. One of lhe following is lo be considered as differential
diagnosis for foreign body in plain X-ray of knee joint:
(a) Fabella
(b) Calcified bursa
(c) Patella
(d) Chondmmatosis NIMS: 2000
47. In a gouty arthritis, the characteristic X-ray findings
includes
(a) Osteoporosis
(b) Erosion of joi nt
(c) Soft tissue calcification
(d) Narrow ing of jo im space NIMS : 2000
48. Hyperextcnsion of' PIP joints and bypertlexion or DIP
joint is known a
(a) Trigger finger
(b) Bou tonier's defo mity
(c) Swan-ne.ck deformity
(d) Mallet finger KERALA : 1999
49. The most common cause of Neuropathic joint is
(a) Diabetes
(b) Syphilis
(cl Leprosy
(d) Rheumatoid MAN /PAL: / 999
50. The most COllllllonly involved joint in pseudo gout
(a) Knee
(b) Great 1oe
(c) Hip
(d) Elbow RAJASTHAN : l

61
9Y3

61
SI. ln a patient suffering from tabes dorsalls charcot'sjoi
nt occurs most commonly at
(a) Elbow
(b) Tarsometatar.;al
(c) Wrist
(d) Knee RAJASTIIAN : J 993
52. Diss(lciativc sensory loss occurs in
(a) Tuberculosis of spine
(b) Disc prola pse
(c) Tabes dorsalis
(d) Syringomyclia TAMIL NADU : 1993
53. Clinical features of Gout are all except
(a) Chronic patien t with renal complication allopu1inol
is
drug of choice
(I>) Positive bircfringem crystal i n the Synovi,il fluid
(c) Hyperuriccmia
(d) Mostly affects smaller jo i m. UP 1993
54. Pain in smalljoints in an elderly lady ismost likely due
to
(u) Rheumatic ar1h1it i s
(b) Rheumatoid arthritis
(c) Psoriaiic arthritis
(d) Reiter 's disease 1/PSr : 1991
SS. Which of the following is inflammatory arthritis?
(a) Rheumatoid a rtlu·itis
(b) Osteoarthritis
(<') Osteochondrit is
(d) All or the above. ALJ_ fNDIA ·1994
56. The most common cause ol' large epiphysis is
(11) HcmophiJ ia
(b) Pager's disease
(c) Osteomalacia

62
(d) Cushing's disease ALL !ND/A: / 994

63
57. Not true about Osteogencsis imperfecta
(a) Impaired healing o[ fracture
(b) Deafness
(c) Laxity of joims
(Ii) Fragi le fracture UP : 2000
58. Ncuropathic joi nts of ankle and foot. are most
commonly caused by
(a) Polio
(b) Club root
(c) Mycetoma
(d) Hansen·s disease TAMIL NADU: 1999
59. Deforming Polyarthritis is associated with all of the
following except
(a) Rneu matoid mthritis
(b) PsoriaLic arthritis
(c) Behcet 's syndrome
(d) Anky losi ng spondyli tis J /1,M ER :
1999
60. A 35-Yea r-old businessmen presents suddenly wit h
severe pain. sweHing and redness in left big toe in
early morning. Most likely diagnosis is
(a) Rheumatoid arthritis
(/,) Ciomy n hri1is
(c) Pseudogou t
(d) Septic artltriti PG/ : J<J94
6J . RA factor is used mainly
(a) Screening patieots for Rheumatoid arthritis
(b) Predicti ng multisystem disease
(c) Predicting severity of disease
(d) Monitoring treatment

62. Which is true about Ankylosing spondylitis?


(a) Sacrol i.ac joints unusually involved
(b) Peiipheral joint is rarely involved
(c) Conrn1on manifestation in females
(d) Iriti s is seen more frequently UI': /994
63. One of the following is not associated with
Ankylosing spondylitis
(a) Pain is more i n tbe early moming
period (b) Pain is rel ieved i n lying down
position
(<-) Morning stiffness more than 3 mouths
(d) May be associated with uvei tis NlMS:1998
64. A young male presents with joint pains and backache.
X-ray of spine shows evidence of saeroilitis. The most
likely diagnosis is
(a) R heumatoid arthritis
(b) Ankylosing spondylitis
(c) Polyanicular ju venile arthritis
(d) Psoriatic arthropalhy UPSC:199!1
65. The most common site of prin1ary osteoarth rosis is
((I) Hip joint
(b) Kneejoint
(c) Ankle jo i nt
(d) Shou l der jo im TAMIL NAOU:/ 997
66. Cluuon's joint are
(a) Syphilitic joint
(b) Encl stage Tubercu lous join ts
(c) Associated with tmu ma
(d) Usually painful

67. The following is the commonest cause of loose body in


a joint
(a) Osteoarthri tis
(b) Ostcochondral fracrure
(c) Synovial chondromatosis

64
(d) Osteocbondritis <lissecan. B!HAR:/ 990

65
68. Bleedi ng into joint cavities is not common in
(a) Chrisuna disease
(b) Hemophilia
(c) LTP
(d) None of ahove. JJPMER:1990
69. Artbroscopy is contraindicated in
(a) Chronic joint disease
(b) Loose bodies
(c) Haemophila
(([) Meniscal tear NATfONAi, 80AfU): 1990

70. A woman presen ted with right shoulder pin and


Rheu matoid factor test came IIS negative, but the 1min
responded to the Prednisolone therapy. The diagnosis
includes
(a) Osteopetcosis
(b) Scronegati vc Rheumatoid arthri tis
(c) Polymyalgia
(d} Polymyalgia rheumtica NAT/ONA/, BOARD: f 990

71. One of the following is true f egard ing Ankylosi ng


spondylitis
(a) Commonin females
(b) Sacrolitis is common
(r.) Anti ONA anrl nt innclear ntihndies re presen1
(d) Symmetrical peripheral arthritis
NATIONAL BOARD: f 990
72. Which of the following agent has I have proven to be
a value in treatmen t of Ankylosing spondylitis·?
(a) Phenyl butazone
(b) Intramuscular golc.l
(c) Naproxen
(d) Penicillamine PGJ: 1990
73. Still's disease is
(a) Post 1raomatic bone formacioo i n the lateral ligamen
t
<rf the knee
(b) Spastic diplegia
(c) R heu matoid arthritis i n child hood
(d} Rheumatoid arthritis 111 the elderly. HIJ-/AR: 19CJI
74. SubltLxation of atlanto-occipital joint is set:n in all
except
(a) Gout
(b) Parpharyngcal abcess
(c) Rheu matoid arthritis
(d) Ankylosi ng spondylit is DELHJ :JY 91
75. Neuropathic joint may aiise in
(a) Syringomyelia
(b) Tabes dorsaJjs
(c) Leprosy
(d) All of the above. NATIONAi.HOARD: 1991
76 In which of the following Polyarthritis is skin lesion
least
likely
(a) Sarcoidosis
(b) Pson atic arthritis
(c) Gonococcal arthritis
(cl) Ankylosing $pomlylilis PG!: 1991
77. All of tl1e following drugs are used in an attack of
Gout
except
(a) Aspi ri n
(b) ldorncthaci n
(c) Colchicine
(d) Phenylbutazone AM C:/ 992
78. Sudden attack of acute pain in great toe is due lo
(a) Rheu matoid arthritis
(b) Gout
(c) R heumatic fever

66
(d) Psoriatic aJth titis. ANDH RA: / 992

67
79. Followi ng are the reconized causes of Hydrarlbrosis.
EXCEPT:
(a) Tu hercu loi,
(I>) Cbarcot"s jnim
(c) Sarcoidosis
(d; Osteo-arLhritis
80. Foll.owing aretlte recognized causes of Charcot's
(Neuro pat hic) joint , EXCEPT for:
ra) Peripbcrnl neuritis
(h) Syringornyclia
(c) Tabes dorsal is
(d) Hysterica, jo int.

81. The treatment of dtoice for Ankylosiug spondylitis is


(11) PhenylbuLazone
(/,) RadioLherapy
(c} Steroid
(d) All of the above.
82. Which joi n t is commonl y affected i n Ankylosi ng
spondylitis?
(a) Ankle
(b) Knee
(c) Shoulder
(d) All of the above.
83. ln Rheuma toid arth ritis.which is involved?
(a) Synovial nuid
(b) Sub chondral bone
(c) Synovial membrane
(if) Cartilage.

84. Severe disability in primary osteo arthritis of hip is best


managed by:
(a) Artl1rodesis
(b) Atth roplasty
(t') M<:: MLLrray" s OS[COlomy
(cl) lntm-articu l ar hydrocortisone and physiotherapy.
85. Which is characteristic F/0 ostco arthritis on X-ray?
(a} Spurring
(b) Subchondral sclerosis
(c) Dimunition of cartilage space
(d} Allof these.
86. "Gouty arthritis" usually involves first:
(a) Ankle
(b) Great toe
(c) Thumb
(d) Shoulder jo im.

I ANSWERS 1

1. (b) (b)
2. (b) 3. (d)
5. (c) 75. (d)
6. (d) 7. (e)
9. (d) 79. (c)
LO. (d) 11.
.13. (d) (b)
83. (CJ
14. (b)
L7. (b) 15. (
18. (a)
21. (c) b)
22. ta) 68
25. (ll) 19.
26. (al
29. (,) (c)
30. (b)
33. (a) 23. (b)
34. (cl
37. (b) 27. (d)
38. (11)
41. (b) 31. (a)
42. (b)
45. tb) 35. (a)
46. (d)
49. 39. (c)
(a) 50. (a)
54. (/,) 4t
53. (b)
58. (cl) (b)
57. (a) 47. (b)
61. (b) 62.
(bd) 51. (d)
65. (b) 55. (a)
66. (Cl )
69. (c) 59.
70. (h)
73. (c) (c)
74. (a)
77. (a) 63.
78. (b)
81. (11) (b)
82. th)
85. 86. (b) 67. (a)
(d) 71.
4. (b)
8. (a)
12.
(a)
16.
(b
)
20
.
(/,)
24.
(c)
28
.
(,)
32
.
(/J
)
36. ( h )
40. (b)
44. (11)
48. (h)
52. (if)
56. (r)
60. (b)
64.
(b}
68. (C)
72. (a)
76. (d)
80. (d)
84. (h)
VASCULAR NECROSIS OF BONE AND EPIPH
8
EOCHONDRITIS

1. Avascuhu- necrosis of lnead of femur can occur in:


(a) Sickle cell anacmii:i.
(/>) Caisson·s disease.
(c) Intmcapsular fracture neck.
(d) Trochanteric fracmre. PGUi9
2. Avascular necrosis of head of femur can occur in:
(a) Subcapital fracrurc.
(b) lnteruochameric fracture.
(r) Transcervical fracmrc.
(d) Central dislocation of hip. AU,IND/A 92
3. Avascular necrosis occurs due to frach1re of
(al Medial femoral epicondyle.
(b) Olccranon.
(c) Talus.
(d) Fibula. AU !ND/1\.99
4. Osteocondritis dessicans occurs at
(a) Lateral surface lateral conclyle.
(h) Medial surface of lateral condyle.
le) Medial surface medial condyle.
(d) Lateral surface medial condyle. Al!MS <JtJ
5. Which is true about Pcrthe's disease
(a) Not Painful.
(b) It manifests at puberty.
(c) lnvolve neck or femur.
(d) Viral etiology. Al 91
I\. Avascular necrosis of head of femur occurs
at
(a) Trochanteric region.
(b) Transcervical region.
69
{c) SubchondraJ region
(d) Subcapital region AllMS 92
7. Pathological changes an Caison 's disease is due to
(a) N,
(bl o
(c) CO,
(dJ co·
8. Legg-Calve-Perthe. disease is commonly seen in the
age group of
fo) 1-3 years
(h) 3- 10 years
(c) I0-20 years
(d) 20 years & above. AU.IND/t\: 1989
9. Osgood-Schlatter disease is
(a) A traction injury of femoral cpiphysis
(b) A traction injury of the posterior epiphysis of o,calc1s
(<') A traction inju1y of vertebral cpiphysi
(ti} A traclion injury nf the libial tubercle of lhc 1ib1al
epiphysis
(e) None of the above is true.
IO. Pw1ched out lesion in skull with beveled edge are seen
in
(a) Secondaries deposits
(/J) Osteomyelitls
ft:) Multiple Mydoma
(d) Eosinophilic granu loma A/IMS: 2000
11. A 5-ycar-old girl presents with pain in the right hip, and
lim p.What is lhc probable diagnosis?
(11) Slipped capi 1a l femoral epiphysis
Cb) Tuberculosis of hip
(c) Pertl1es·disease
(d) CDH. NIMS:2000
12. A Patient is using ora I steroids for a period of 5 years
and patient complaints of pain in the both hip regions.
70
'hicb one of the following is a diagnostic modality for
confinnation of diagnosis?
(a) Plain X ray.
(b) CT can.
(c) MR!.
(d) lsotope Bone can. NIMS:2000
13. Osteochoudiritis is not seen in -disease
(a) Pel ligrini-S1ieda.
(b) Panner's.
(c) Calve's.
( d) Kohler's. DEUil: /999
14. Ostcocondri tis in Osgood Schlatter disease affect which
bone?
(a) Capitulu m"s.
(b) Metatarsal.
(c) l\'.avi<:ular.
(d) Tibial t11berosity. UP:1993
15. Legg-Calvc-Perthes' disease is
(a) Femoral epiphyseal os1eochondritis.
(b) Osteochonc.lritis of tibial tu bercle.
(c) Slipped upper femoral cpiphysis.
(d) Spinal osteochondrit is.
I 6. The late radiographic changes st-en in active Perlhes'
disease are all except
(a) Narrow join t pace.
(b) A widened femoral neck.
(c) An irregular density of the epiphysis.
(d) An iITegular epiphyseal line.
(e) A Oattened head. NATIONAi.BOARD: /Y90
17. Legg-Calves-Perthes ' disease is
(a) Mild pyogenic arthriti.
(b) Slipped fomor.il epiphysis.

71
(c) Avascular necwsis of head of femur
(d} Low grade Tul:>erculosis of hip OR/ SSA: 1991
18. The commonest cause of limp in a child of seven years is:
(u) T.B. hip
(b) C.D.H.
(c) Pe111le·s disease
(d} Slipped upper femoral epiphysis.

I ANSWERS I
J. 2. 3. 4. (d)
(a.b.c)
(a,c) (c) 8. (h)
5. (h)
6. (b) 7. (aJ 12. ( d)
9. (d)
I0. (c) 11. (C) 16. (a)
13. (a)
17. (c) 14. 15.

m
(t/) (11)
18. (c)
3. Tu
mo
ur
TUMOURS OF BONE mo
I st
se
1. Commonest benign tumour of the bone is: nsi
(ti) Osteoma. tiv
(b) e
Osteochondroma. (c) to
Osccoid osteoma. Ra
(dj Chondroma dio
the
2. Sunray appearance is seen in: rap
(a} Osteogenic sarcoma. y
(h) Ewing's Sarcoma. is:
(c) Multiple myeloma. (a) O
(d) Osteoclaswma. sr
eo
72
genic sarcoma.
(b) Ewing's sarcoma.

A.l.88

,b,,J.89
(c) Chondrnsarcoma.
(d) Ostcoclastoma A.l.117
4. Osteogcnic sarcoma metastasizes commonly
to:
(a) Liver.
(b) Lung.
(c) Brain .
(d) Regional lyrnphnodes. UPSC 87
5. In Carcinoma prostate with metastasis which is raised:
(a) ESR.
(b) Alkaline phosphmasc.
(c) Add phosph,ua. e.
(d) Bilirubin J'N.IIIJ
6. Which of the following arises from epipbysis:
(a) Ostcosarcoma.
(b) Ew ing·s sarcoma.
(c) Ostcoclastoma.
(d) Multiple Myelo1m1. TN.89

73
7. ln multiplc myeloma ,;vbicb of the following i seen;
(a) Raised serum calc.ium.
(b) Raised alkaline phosphatase.
(c) Raised acid phosphatase.
(d) All K£NALA.90
8. Commonest tumour arising from the metaphysis is:
(a) Osteoclastoma .
(b) Oteosarcoma.
(c} Ewing's sarcoma.
(d) Synovial sarcoma . KERALA.89

9. Treatment of solitary bone cyst is;


(a) Curettage.
(b) Excision.
(c) Curettage and boue grafting.
(d) Irradiation. PGJ.90

74
10. Physalipbarous cells (ILarge vacuolated cells) on Histo·
pathology are characteristic of:
(a) Ostco,arcoma.
(h) Ostcoclastoma.
(r) Liposarcoma.
(d) Chondrosarcoma.
(e) Chordom,L JIPMER78. AMU'8./
11. Vertical striations on vertebral bodies are seen in
(a) Hemangioma.
(bl Pagets disease.
(c) Vertebral mcta,rnsis.
(d) Osteoporosis. JIPMER.95
12. Most reliable method for detecting bony metasta es is;
{a) MRI
(b) CT Semi
(c) Radiography
(d) SPECT. .!IPMER'95
13. Trne statemen t regru:ding osteogenic sarcoma b
(a) Affects middle aged people.
(b) X-Ray hows hone.y cnmbi11g
(c) Cru1be a complication of Pagct"s disease of bone.
(d) All of the above. 1'1\L'96

14. Commonest benign tunlOur under 21 years of age:


(a) AneurysmaJ bone cyst.
(b) Osteochondroma
(c) Gaint Cell Tumour.
(d) Os1eoid osteoma PAl .'96
15. A boy presenting with swelling at lower end femur 1ith
calcified, nod ula r shadow in lung has;
(a) Osteosarcoma.
(h) Osteochondroma
(c) Tu berculosis femur lower end.
(cl) Osteomyelilis. AIIMS.<J6
16. la an 8-year-old ch.ild the least common cause of lytic
bone lesion in proximal femur.
(a) PJasmacy1oma.
(b) Histiocywma.
(c) Metastasis.
(d) Browo tumour. All MS. 97

17. Most Common lesion of hand is


(a) Enchondrom .
(b) Synovioma .
(c) Exos1osis.
(d) Osreoclastoma. All MS'97
18. Rare site of metastasis in bone
(a) Skull.
(h) Spine.
(c) Upper end of humerus.
(d) Below elbow and knee. IN '99
19. A 8 year old child ha:,: a swelling in diaphysis of
l'emur. Histology reveals, small dear round
symmetrical cells, minimu m cytoplablll, necrotic
areas, and mini mum osteoid and chondroid material
cells. Most likely. it contains.
(a) Mucin.
(bl Lipid.
(c) Iron
(d) Glycogen Al!MS.2K
20. Most reliable method Jor detecting bony metastases is:
(a) MR L
(b) CT Scan
(c) Radiography
(d) SPECT.
.lli'MJ:;t?'2
K
21. Ivory osteoma commonly arises in the:
(a) Slrull.
75
(b) Ribs.

76
(c) Pelvis.
( c[) Vertebra UPSC 86, NIMHANS Rfi
22. Osteoblastic secondaries can arise from:
(a) Carcinoma Prostate.
(b) Thyroid Carcinoma.
(c) Renal Carcinoma.
(d) Breast carcinom,L UPSC.85. A P.85
23. Eochondroma commonly arises from:
(a) R ibs.
(b) Vertebra.
(c) Tibia.
(d) Phalanges. AMU 88
24. Osteogenic sarcoma can develop in:
(a) Osteoblastoma.
(b) Pagels duscases,
(c) ostcoid ostcoma.
(d) All of Lhc above. AMU 88
25. The treatment of encbondroma is:
(a) Amputaiion.
(b) Irradiation.
(<') Local excision.
(d) Curettage and bone crip filling AM U 88
26. Onion peel apperance i:n X-ray suggests:
(a) Osteogenic sarcoma.
(h) Ewing's sarcoma.
(c) Osteocclastoma.
(d) Chondrosarcoma
27. Commonest site of a bone cyst:
(a) Upper end of humerus.
(b) l.-Ower end of tibia.
(c) l.-Ower end of femur
(d) Upper end of femur
28. Commonest site for osteogenic sarcoma is:
(a) Upper end of femur.
(b) Lower end of femur.
(<") Upper end of tibia.
(d) Lower end of tibia.
29. Kach rumal, a 46 year old man has expansive growth
metaphysis with endosteal scalloping & dense punctatc
calcification. Most likely bone tumour is
(a) Osteosarcorna
(bl Chondrasarcoma
(c) Osteoclastoma
(d) Osteoi<l ostcoma AI 2002
30. A 70 year old lady presented with mild low back pain
tenderness in L3 vertebra. On examination Hb 8gm
ESR UOmg/lhr NG ratio of 2:4, likely diagnosis is
(a) Waldenslorms
(b) Mu l ti ple myeloma
(cl Bone sccondaric::s
(d) None A/IMS 200
I
31. Age group of osteogenic sarcoma is
(Ci) 1-10
(b) 10-20
(c) 20-30
(d) 30-40 JIPMER 90
32. The lytic lesion in the epiphysis inchildren is seen in
(a) Osteogcnic sarcoma
(b) Osteoclastoma
(cl Aneurysmal bone cyst
(d) Chondroblastoma 1V 9 /
33. Pain in thigh more at night relie,•ed by aspirin is
(a) Steosarcoma
(b) Osteoclastoma

7
7
(c) Ewings tumour
(d) Osteoid osteoma KERA /..,- '.-N
34. Bone tumour metastat ising to bone is (d) Ostt
(a) Giant Cell Tumour !
(/,) Ewing's Sarcoma oSai
(c) 01oodro Sarcoma ·co
ma

KE
RA
UI
94
35. Osteocl
astoma
is
commo
n in age
group of
(a)
Below
10
years
(b) 10-
20
years
(c) All
age
groups
(d) 20-
40 yeats

J4
36. J3one
Cysts
most
commonl
y occur
in
(a) Spine
(b) Humerus
(c) Femur
(d) Tibia

37. The most confirmatory test for


myeloma is
({I ) Aspi ration of the lesion and
histology
(b) Bence-Jones protein in uri ne
(c) Serum electrophoresis
( rf) Technitium 99 rowonuclidc bone
sc,u1
Al/MS 80. RHU 87
38. On microscopic examination,
.uspected G iant Cell tumour must
be differen tia ted from
( a) Osteosai·coma
(b) Osteoicl Osteoma
(c) Brnh of the above
(d) None of the above.
39. Ewing's tumour of bone
(a) Shoukl be locally e1'cisecl
(b) Should be trea ted by immediate
ampu rntion

78
(c) Looks like a cm 011.ion on X-ray.
(d) Ha;; a soap-bubble appearance on X-ray.
(e) None of the above h, correct. HJHAR / 988
40. The most common Primary Malignant bone tumour is
(a)Osteodas1oma.
(b) Ewing's sarcoma.
(c) Ostcogenic sarcoma.
(d) Osteogenic sarcoma. B!HAR./ 988
41. Osteoclastoma shows
(a) Expa11sile osteolytic area in the diaphysis.
(b) Expansilc osteolytic area in the epiphysb.
(c) Osteosclerotic area in the metaphysis.
(d) 0$Lcoly tic area in the metaphysis. PG!.1988.
42. Ewing's sarcoma can be confused l1istologically with
(al Myeloma.
(b) Osteosarcoma.
(c) Osteomycli tis.
(d) Giant Cell tumour. N.OHTAK: / 988
43. A cl1ild has smooth tender swelling at lower end of knee
for 6 months. On X-ray examination there is new bone
formation but no jo int involvement. dinl!nosis is;
(a) Ostcosarcoma
(b) Osteoclastoma
(c) Arthritis followed by resitl.ual osteomycl itis
(d) Chronic osleomyclctis
(el Any or the above. AMC 1989
44. Ewing's tu mour is not associated with
(a) Sunray appearance
(b) Pus like mate1ial on aspiration
le) Diaphysis involvement
(d) Mottled appearance of bone rnarrow. ANDH RA. /
989

79
45. Osteoclastoma characteristically involves
(a) Hip join t.
(b) Fibula lower end.
(l') Tibia lower end.
(d) Radius lower end. BIHAR.1989
46. The most common site of enchondroma is
(a) Rihs.
(b) Phalanges.
(c) Clavicle.
(d) Sternum. DELHI. 1989
47. Ewing's tumou r of bone
(a) b a giant cell tumour.
(b) Is also a secondary deposit from a ganglioneruoma.
(c) Usual l y occurs in middle age.
(d) Is characterized by subpcriostcal new bone formation.
which gives the appearance of ·onion peer.
(e) Ca n be 1reated satisfactorily by re,ection and the
i nsertion of prosthesis.
48. The common mode of presentation of a case of Solita ry
Bone Cyst is
(a) Pain.
(b) Fracrnre.
(c) Inflammation.
(ti) Expansion or the involved area. RO/lTAK: 1989
49. Regarding Fibrosarcoma
(a) It has predilection for femur and ti hia.
(/1) Amputation is the answer to I.h is.
(c) X-ray shows moth eaten appearance abnur a lyt ic
area
(d) All of the are true.
(eJ None of the above: is u-ue. ROHTAK.1989
50. The most common Bone tumour is
(al Ostcosarcorna.
(b) Osteoclast0rna.

80
(c) Secondaries.
(d) Multiple myeloma. TAMIL NADU. 1989
51. Very poor prognosis of Osteosarcomas because these are
(a) Highly malignant ru mow·s.
(b) Inoperable ui;ually.
(c) Metastasize to lung very fastl y.
(d) Resistant LO radiotherapy. TAMIL NADU. 1989
52. Treatment of Fibrosarcoma is
(a) Surgery (wide excision ).
<b) Surgery + Radiotherapy.
(c) Chemotherapy.
(d) Surgery + Chemotherapy. AllM.S: 1990
53. Pain in thigh more at night relieved by aspirin is
(a) Ewing's cumour.
(h) Ostcosarcoma.
(c) Osteoid osteoma.
ld) Osteoclastoma. KERAL1.1994.
54. Oe!-teosarcoma of distal femur is best treated by
(a) Amputation.
(b) Disarticulation al hip.
(c) Chemotherapy.
(d) Radiotherapy. ANDHRA.1994
55. Tumour not arising from cartilage is
Ia)
F.nchondroma.
(b) Chondrnsarcoma.
{<;) Osteoblastoma.
(d) Ostcochom.lroma_ PG!. 1993
56. Osteosarcoma differentiated from Myositis ossificans
by radiology
(a) Location.
(b) lllfection is cause.
(c) Shape of swelling.
(d) Pelipheral lield of
differentiation. 81
JJPMER: 1991
57. The most common cause of Neuropathlc joints is
(a) Leprosy.
(b) Diabetes.
(c) Rheumatoid ruthri1is.
(d) Syphilis. JJPM /;R: IYYJ
58. In tllDlour surgery, wide resection means;
(a) Shelling out through pscudncapsulc of tumour
(b) En block resection witl1 a cuff of normal tisue
(c) Ex1ra-compar1mental En block rcsec11on
(d) None of the above. KARNATAKA./ 993
59. Ev.ing's sarcoma is treated by
fo) Excision and filing with bone
(b) Surgery + Radiogbarapy
(c) Surgery + Chemml1erapy
(d) Whole bone imidiation + Chemotherapy.
ANDHRA l 9Y3
60. The most common tumour thaJ invol ves bonei.
(a) Giant Cell tumour
( b) Chondrosarcorna
(c) Multiple myeloma
(d} Metastatic tu mour from cxtraosseous i te.
AN[)Hf?A.IY93.
61. All of the following lesions araise from metaphysis,
except.
(a) Os1eogcrtic sarcoma
(b) OsteoblasttlmH
(c) Condromyxoid fibroma
(d) Gaint cell tumour. ANDHRA l 9Y.J
62. Chondroblast belongs to
(a) Labile cells
(b) Permaoen t cells
(c) Stable cells
(d) All of the above. I\NDHRA. / 993.

82
63. Osseus metastasis is most common if tumour is in
(11)Bronchus
(b) Colon
(c) Pancreas
(d) Adrenal De/111. 1993
64. viuch is a diapbyseal bone
tumour
(a) Giru11 Cell tumour
(b) Ostco$arcoma
( c) Fibrosarcoma UPSC·!CJIJJ.
(d) Ostcoid Osceoma

65. The most common site of Multiple myeloma is


(a) Skull
(b) Humeru,
CL-} Pelvis
(d) Vertebrne ONISSA-1992
66. The tumou r associated \\ith Hormonal abnormalities is
(a) Fibrosarcoma
(b) Chondrosarcoma
(c) Osceosarcoma
(d) Fibrous dyplasia PG!: /992
67. Treatment of Ostcodas1oma of lower end of radiu, in
early stage ls
(a) Bone cureuage and fill ing
(b) Amputacion
(c) Radiotherapy
(d) Conservative RAJ ASTAN: 191.J]
68. Diagnostic investigation for bone tumours is
(a) FNAC
(b) Frozen section biopsy
(c) lncisional hinpsy
(d) C.T.Scan. DELH!. / 992
69. Soap bubble appearance is seen in
(a) Ewing· sarcoma
(I,) Osteoid osteoma
(c) Os1e11clastoma
(d> Osteo sarcoma DELH I.· / 1)111
70. The most common prrimary malignant tumour of the
bone is
(a) Osteogenic sarcoma
(b) Ewinu's sarcoma
(c) Osteoclastoma
(d) Fibro sarcoma Nll,WS:/ 991
71. The treatment of choice for Enchoudroma is
(a) Excision
(b) Amputation
(cl Curettage and bone grafting
( d) Rad imherapy ORISSA. l':192
.. is true about Osteosarcoma?
72. Which
(rt) lt is due to os1eocla 1
(b) Jt occurs in fifth t\l sixth decade
(c) Lymphatic spread common
(d) Sunray appearance i due to new bone for mat ion.
AITMS:2000
73. The rad iological diagnosis of Ewing's sarcom a includes
(a) Onion skin
(/1)Sun-ray
( c) Soap bubble
(d) Calcification J\NDHRJ\: 1000
74. Chonclroblastoma invariably invol ves
(a) Epiphysis
(b) Diaphysis
(c) Metaph ysis
(d) Flat bone ALL IN!)JA:2000

84
75. Adamantinoma of limb bones are most frequently found
in the
(a) llumcrus
(b) Femw·
(c} Tibia
(d) Radius ALL INl>IA:2UIJ(J
76. Which of the following is not associated with Mulliplc
Myeloma
(Cl) Punched out lesions 111 the skull
(b) Amyloid JeposiLion in 1bc joi n1
(c) Aseptic necrosis of I emoral head
(d) Mcrasrmic calcificati on 4 /.L IND/II.
1000
'
77. 'Which of the followin is true regarding aneuri mal hone
cyst?
(a) More conunon ufter the age or 50 years
(b) More common in vertebra
(r) Characterislically elliptical in long bones
(if) Due m atherosde:ro,i, ..\LL /NOIA: 20UU
78. Osteolrophic Secondaries are all except
(a) Carcinoma lung
( h) Carconoma prostmc
(<'l MulUplc Myeloma
(d) Carcinoma breat AU IND/A: 2000
79. Calcification is found in
(a) Chondroblastom:.s
(b) Os1coclastoma
(c) Chondrosarcoma
(d) Osteosarcoma AU.IND/A: 2000
80. Themostcommon presentation of unicameral bone cyst is
(a) Asymp1omatic
(/;) lnfection
(r) Swelling
(d) Frncmre 1hrough cyst ALL IND/A: 2000
81. Radiological feature or Ewing's sarcoma least mimics
to all except
(11) Rheumatoid arth ri t b
(b) Osteomyelitis
(c) Ostcogenic sarcoma
(d) Recticulum sarcoma ALL IND/A: 1noo
82. In multiple Myeloma main light chain defect lies in
(a) lgA
(b) LgD
(C) JgG
(d) lgM

UP.1999
83. Which is the rare site of metastasis in bcmc:
(a) Skull
(b) Upper end of hu merus
(c) Spine
(ti) Below elbt,w & knee TAMIL NA!)U: / 999
84. The most common site of Osteogenk sarcoma is
(r,) Ribs
(bl Distal end or femur
(c) Proxi mal enc.I of humerns
(I)) Distal end of libia TAMIL NADU: IY 99
85. Aneu.rysmol bone cyst is
(a) More common af1cr!he age of 50 years old
(b) Most commonly affects venehra
(c) Due to therosclerosis
(d) Characleristically lollks dlipl ieal i n long bones.
TAMILNADU: / 999
86. Acu te Osteomyelilis i-esembles which primary bone
tumour?
(a) Ewi ng's tumou r
(b) Osteochondroma
(c) Ostcosa:rc.;oma
(d) Chondroma TAMIL NADU ! '.ICJCJ
87. Which of the following is a wrong statement rega rding
Cbondrosarcoma?
(o) This t11mow·occurs mainly between the ages of '.!O and
60 years
(b) ltcan present as secondary malignruu changein Paget·s
disease
(c) High-grade t u mou rs shows poorl y differen t iated
cartilaginous pauern with anaplastie cells.
(d) Cen tral lesions havi ng a better prognosis
NIMS. 2000
88. All are true about Osteoid osteoma
except
(11) Malignan t couver:.ion is 1101
uncommon
(b) They cause night pai n which is relieved by aspiill
(c) Common ly seen in ft:males
(d) Dou ble density seen in X-ray KERA!.·\:/
999
89. Which of the foilowing turuow·arises from :Vlctaphysis?
(a) Chondrnsarcoma
(b) Ostcosarcoma

-
(c) Ewin g's sarcoma
(d) Osteoclaswrna
. MAN/ PAL.: J 9Y9
90. X-ray appearance of Ewing's sarcomu resembles
(a) Acu te o,neomyelicjs
(/J) Eosinophilic granulom.i
(c) Neu.roblaslOma
(d) Osteogenic sarcoma JIPM£t<: 1()99
91. All are true regarding Ewing's tumour of bone except
(a) Arises from Endothelial cells in tl1e bone marrow
(/.,) Onion peel appearance on X-ray
(c) Radiotherapy is the treatment of choice
(d) Forms a differen tial diagnosi:, for osteornyclitis
92. 1n bony metastasis, primary should be looked for in all
excepr
(a) Prostate
(b) Thyroid
(c) Stomach
(d) Broncbu,
93. Ewing's sarcoma can lbe confused histologically wilh
(11) Myeloma
(I>) Osteosarcoma
(c) Ostcomyclitis
(d) Giant Cell Tumour UP: / 9<;3
94. The most common site of Chondroblastoma is
(a) Din phy;,i
(bl Epiphysis
(c) Soft tissue
Cd) Metaphysis UP: /993
95. Oslt'Oblastic Secondaries are seen in all except
(al Carcinoma pro,tatc
('7) Carcinoma breast
(c) Carcinoma thyroid
(tf) Multiple Myelonm ALL /N[)/A: !9W
96. The most common site f'or hone cyst is
(a) Lower end or femur
(b) Upper end of humerus
<c) Rad ius
(d) Lower as suc.:h AU IND/ A. 199./
97. A 5-year-old child presents with swelling in the arm. X
ray shows circumfcre11tial growth in the mid shaft of
humerus.The probable diagnosis is
(a) Ostcogcnic sarcoma
(b) Ewing' sarcoma
(c) Chondro sarcoma
(d) Ostcom yelitis NI/MS. 2000
98. Which of the following iswrong about Ewing'ssarcoma'!
(t1) IIis diaphyst'aJ in origin
(b) Roselle shaped cellu l ar pattern is seen

88
(c) 5- 10% are Associated with chromosomal translocation
(d) ii is radio resistant wmout PG!: 2000

99. 8-year-old child presenL5 with one-year back f'ratturc


neck of humerus and X-ray reveals cystic lesion.
The probable diagnosis is
(a) Osieoclastoma
(b) Osteogenic sarcoma
(c) Osteomycli1is
(d) Unic.imcral bone cyst UI': 2000
100. A IO-year-old boy presen ted with pai n and
massivt sweUing left thigh. 0111 examination reveals
diaphyscal lesion and soft tissue swelling. The likely
diagnosis b
(a) OsteO$arcoma
(bl Ewi ng's sarcoma
(,·) O,teocla,Lorna
(d) Ancurysmal bony yst. Al/MS: N0\!.2000
101. All of tbe following features are charartel'ist ic of
Osieoclastoma, except
(a) Pathological fractu re
(bl Pred ilection for the female \l!X
(c) Recurrence followinglocal remov.il
(d) ·sun-ray' spicules ou X-ray
102. Bones most commonl y involved in metastasis in
carcinoma prostate is
(o) Skull
(b) Sternum
(c) Pelvis
(d) R ibs Nl1timwl Board · / 994
103. About unicameral bone cyst. true is
(a) Spondy 1isue i nside cyst
(b) Fibrous septa
(c) Bony septa
(d) Angiomatous malformation
104. Sclerosis of Yertebral end plates can occur in CoUowing
except
(a) Sarcoidosi
(b) Osteoporosis
(c) Rheumatoid arthriti
(dJ Healing Osteom,llacia RAJASTHAN : / 994
!05. The most conlillon malignan t bone tumou r in 5-15 years
age group is
(a) Fibrosarcoma
(b) Osteosarcoma
(c) Ostcoma
(tl) Osteochondroma UP.·/ 994
106. Osteochond roma is conunonly seen in
(a} Ribs
(b) Scapula
(c) Epiphysis
(d) Metaphysis in femur UP:1994
I 07. The majority of giant cell tumours (Osteoclastoma) of
bone arc ,matomicaUy located in the
(a) Yencbrae
(b) Small bones of hand and foot
(c) Proxi m I humerns :).nd proxi m!tl femur
(d) Proxi mal tibia and distal femur BJHAR : 1998
108. The most com mon mode of metastasis in ocstogenic
sar coma
(a) Lyumphatic
(b) Subperiostcal spread
Ic) Transcortica I
(d}
JIPMEN: / 99N
Hacmaiogenous
109. Presence of' Bence .Jones Protei ns is found in
(a) Eosinophilic granuloma of bone
(b} A lk apton u ria
90
(cl Multiple Myeloma of bone
(d) Osteog,:inic sarcoma KARNATAKA:/ 998
110. A child presen ted with upper leg swelling wilh pulmo
na ry nodule. The most probable iliagnosis is
(a) Ostcosarcoma
(b) Osteoclastoma
(c) ChondrohlaSl<ima
(d) Chondrosarcoma MP·/ 9':18
111. Giant cell tumour of IJ.one (Ostcoclastoma) is radiologi
cal characterized by
(ll) Expansi ve ostcolytic lesion
(h) Onion peel appearnnce
(c) Stippled osteoporosis
(d) Pe,iosteal new bone formation m1dcodnia.ntri:mgk for
mation.
112. Aneurysmal bone cysts
(a) A.re 1.nie aneurysms of nutrien t arteries
(b) Occur only in flat hones
(c) Are the same as osseous haernangiomas
(d) Manifest as osteolytic lesions UPSC:1998
113. Which of the following is not true about Ewing's sar
coma'?
(a) Usually diaphyseal
(b) Periosteal thickeni ng appear as onion peel appearance
(c} Egg shell crackling present
(d) Rad ioscnsiti vc DEU/1: /997
114. AU of tile following statements regarding Osteosai-
coma is true except
(a) Spreads by lymphatics
(h) Arises from metaphysis
(c) Arises in Paget's diseases of bone
(d) Alkaline phosphate levels are increased.
MANIPAL: 1997
115. A 65-year-old man presents with backache and
1uinar:,• retention. The nexl line of investigation is
(a) Acid pho,phatasc
(b) Alkaline phophatasc
(c) Urodynamics
(1[) Ultrasonography Al/MS: 1999

ll 6. The most common site of os!eoblastoma is


(a) Diaphysi s
(b) Metaph ysis
(<") Epiphysis
(d) B or C l>GI: 1997
117. Giant cell tumour
(a) Periarticular epiphyseal i n origin
(b) Never recu rs after excision
(1:) Malignant potcntial noc pred icted
(dJ All of the ahovc. PUJ: I !197
118. Expansilc growth arising from the end of long bone af
ter epiphyscal closure is
(Cl) Aneurysmal bone cyst
(b) Simple bone t•ys1
(c) Os1eosarcoma
(d) Giun1cell tu mour
(e) Allof these. B/1-!AR: 1990
119. Clean-cut multiple, rounded lesions in bone are seen in
(a) Em iuophilic granuloma
(b) MuJtiple myefoma
(c) Lc1terer Siewe ui e11se
(d) Al l of 1hese
(e) None of Lhee. 811-IAR: /'J'J{)
120. Treatmen t of choice for Giant Cell Twnou r is
(a) Surgery + Ractimhernpy
(b) Local excision
(c) Chemotherapy
(d) Radiotherapy DELIII: 1990
121. Age group of Osteogenic sarcoma is
(a) 20-30
(/;) 1-10
(c) 30-40
(d) 10-20 .l!PMt;/?:199()
122. Which of the following bone tumour presents with pain
Iirst and swelling later on?
(a} Osteoma
(b) Osteoclstorna
Cc) Chondrorna
(d} Ostcosarcoma
le) Ewings·s tumour NATIONAi. BOARO:1990
123. \>Vllich of U1e followi ng is common site of bone cyst'!
(11) Humerus
( /,) Fem ur
((') Fibula
(d) Pelvis NATIONAL BOAR{): 1990
124. The preferred treatment for unic.imcral bone cyst is
(a) X ray therapy
(b) Surgicalexpoure and collapse of cavity
(c) Surgical curettage and bone graft
(d) Create a fracture am!allow healing.
NATIONAL 801\RD:/ YIJIJ
125. Treatment of Ewing'ssarcoma l by
(11) Chemotherapy
(h) Radical excision
(c) Radiotherapy
(d) lmmunothcrapy NATIONAL BOARD: 1990
126. Alkaline phosphatase is raised in
(a) Osteocla mma
(b) Osteosarcoma
(c) Chond rosarcoma
(d) Multi ple myeloma NATIONAL BOARD: 1990
127. Tumou r arising from Diaphysis is
(a) Osteoclastomu
(h) Multiple myeloma
(c) Ostcosan:oma
(d) Ewing·s sarcoma OR!SSJ\: 1990
128. Which of the following regions is most commonly
involved in skeletal lu berculosis?
(a) Spine
(b) Hip
(c) Knee
td) Ank le
(e) Elbow PGl. / 91.JO
129. Presently the hest treatment for Osteosa rcoma of tibia
is
(a) Radiotherapy
(b) AmpmaLion followed hy local radia1ion
(c) Chcmothernpy
(d) En block resection and rccoo,Lruc1ion
(e) A mpn rnlion followed by chcmo1.her.ipy. PC!:/ 0()0

130. Osteogenic sarcoma m;;ually occ11rs in the following age


grou p
(a) Below 15 years
(b) Between 16 to 25 years
(c) Between 26 to 40 years
(</) Between 41 to 60 years

131. Skeletal radiological survey of a 50 years old patient


shows multiple lytic areas in tl1e skeleton. Most like!)•
diagnosis is
(a) Multi ple pyemic abscesses

94
(b) Pagct"s disease
(c) Mu ltiple Myeloma
(d) Mu ltiple oeurofibromatosis
(e) Multiple cnchondromtosis OR/SSA:/ 99U
132. The most importan t feature of severance of media n
nerve at the elbow is
(a) Wast ing of hypotbcnar m udes
(b) Wasting of thenar eminence
(c) Paralysis of flcxor of in<lex finger
(d) Paralysis of ncxors of wrist
(e) Anaesthesia of radial 3.25 finger.;. ORJSSA:/ <)<){)
133. Solitary bone cyst is ti·eated by
(a) Curettage
(h) Excision
(c) Curettage and booe grafting
(d) Tn·ad ia1ioo TAMIL NIIDU: 1990
134. A lytic lesion in the epiphysis of a long bone of a
child is likely to be
(a) Osleoclastoma
(b) Chondroblastoma
(c) Aneurysmal bone cyst
Cd) Brodie·s abscess ALL IN/Jiii ; f')fJ I

135. Bony metastasis is seen in all c:iccept


(a) Carcinoma thyroid
(b) Carcinoma bronchus
(c) Carcinoma breast
(d) Cystosarcoma phyUoides Al/M S: / 991
136. Regarding osteoid osteoma
(a) Mos!commonly seen in patien t between JO an<l 25
years old.
(b) Slightly more common in males
(c) Usually occurs in long bone (tibia)
(d) Most commonly involved bones bemg the t"cmur
(e) All of the above. BIIJAR: / 99/
137. The most common site of appearance of osteoma is
(a) Skull
(b) Forearm
(c) Pelvis
(d) Hands ,md feet
(e) Any of the above. 8/HAR: / Y'JI
138. Cod man's triangleis in case. of
found(a) Os1eoclas10111a
(h) Baker's cyst
(c) Osteosarcoma
(d) Ewing's tumour only
<e) Any of the above. 8/IIAR: / 99 /
l39. Treatment of' Giant Cell Tumour of' upper end of Iibu.la
is
(a) Amputation
(b) Excision of upper end of tibula
(c) Curettage with grafting
{d) Radiation NIMS: 199/
140. Fibrosnrcoma
(n) Similar 10 Giam cell 1umour on X-ray
(b) Patchy opacity in X ray (Osteoly1ic le,ion)
(c) Osteoblaslic lesion
(d) Soap and bubble appearance
(e) Associated endocrine ab1101111ulities PG!: / 99/
141. Following one is a diaphyseal tumour
(a) Osteosarcoma
(b) Synovifll sarcoma
(c) Osteoclastoma
(d) Ewing's sarcoma RAJASTHAN: / 99/
142. loitial treatment of choice for Giant Cell Tumour of
bone
is
(a) Curettage and bone grating
(b) Bone grafting
(r.) Ampmation
(d) Radiotherapy UPSC: 1991
143. Which of the following occu rs late in the course of
Mul tiple Myelorua?
(a) M-Spike
(b) Renal failure
(c) Bony Lytic lesions
(d) Increased alkaline phosphates Al/ MS: 1992
144. Ewing's tumour is
(a) Radiosensitive
(b) Radioresistant
(I') Rad iosensitive but recurrent
(d) Radiocurable AN DH RA : 1992
145. Osteogenic sarcoma most commonly arises from the
:
(11) Epiphysis
(b) Metaphysis
(c) Diaphyi
(d) A II of these.
146. Osteoclastoma arises from
(a) Epiphysis
(b) Metaphysis
(c) Diaphysis
(a) Femur

147. A defin radiological feature of a bone sarcoma is the:


(a) Sun ray appearance
(bl Codman's triangle
(c) Presence of soft tissue shadow in the sk.iagram
(d) All or the above.
148. "Soap-bu bble appe ara nce" of X-r ay pictu re is
characteristic of:
(a) Osteoma
(h) Osteogcuic sarcoma
(c) Ewing·s tumour
(d) Osteodastoma.
149. Among the following bone tumour, which is most
radiosensitive?
(a) Ostemna
(b} Osteogcnic sarcoma
(c) Ewing\ tumour
(d) Osreoclastoma
150. The con11noncs1 site of chondrosarcoma is
(a) Ribs
(b) Pelvis
(c} Radi us & humerus
id) None. of tl1e. e.
151. Following are true regarding Osteosarcoma
EXCEPT:
(a) Commonest sites al'c the lower nd or the fem11r
(b) Codmrui's triangle is present
(c) Tumour tlccurs at Metaphysis
(ti) Tumour usually metastasise by lymphatic rout

152. Which of' the following has got best prognosis?


(a) Oteogcnic sarcoma
(b) Ewing's SJJrcoma
(c) Chondrosarcoma
(d) Chordorna

153. The treatment of' choice for multiple Jvlyeloma is


(a) Surgery
(II) Radiotherapy
(c) Hormone
(cf) All the above.
154. Which is not associated with multiple Myeloma?
(a) Amyloid deposition m the joims
(b) Met,l talic calcifica1ion
(c) Punched out lesions in the skull
(d) Septic necrosis of femoral head.
155. Enchondroma should be treated by
(11) Irradiation
(h) Amputation
(c) Local excision
(d) Curreiagc and bone chips grafli ng.
156. Adamantinoma of limb bones are most frequently
found
in the
(11) Ul na
(b) Radius
(c) Femur
(d) Tihi11
157. Chondrosartoma arising in long bones occurs most
frequently in the:
(a) Tibia
(h) Fenmr
(c) Humerus
(d) Radius only.
158. AU the followingare true about osteo clastoma.
EXCl<:PT
(a) Commonest site is lower end of femur
(b) Commonly occurs in you ng adu lt
(c) Soap bubble appearance is seen in X-ray
(d) Never recur after local removal ..

I ANSWERS I
1. 9. (c) 2.
(e)
(h) 13. 14. (b)
Ca) 6.
5. (c)
(b) 10.
(b)
9
9
3.
(b) 4
7. (c)
.
11. (a)
(
JS. (a) b
)
8. (bl
1
2
.

(
d
)
1
6
.

(
a
)

9
9
17. (a) 18 (d) 19. (d) 20. (d)
21. (a) . (a)
22. 23. (d) 24. Cb)
25. (d) 26. (b) 27. (a) 28.
29. (b) 30. (bl 31. (d) (b) Cbl
32.
33. (d) 34. (b) 35. (d) 36. (/J )
37. (c) 38 (d) 39. (c) 40. (C)
41. (b) . (c)
42. 43. (a) 44. (d)
45. (d) 46. (b) 47. (d) 48. (h)
49. (b) 50. lC) 51. (a) 52. (a)
53. (c) 54. (/J ) 55. (<") 56. (cf)
57. (b) 58. (c) 59. (b) 60. (d)
61. (d) 62. (a) 63. (a) 64. (d)
65. (d) 66. (d) 67. (Cl ) 68. ( I:)
69. (c) 70. (a) 71. (1') 72. (d)
73. (a) 74. (a) 75. (c) 76. (d)
77. (c) 78. (c) 79. (c) 80. (n)
81. (b} 82. (d) 83. (d) 84. (b)
85. (d) 86. (a) 87. (d) 88. (11)
89. (b) 90. (a) 91. (a) 92. (c)
93. (c) 94. (b) 95. ld) 96. (b)
97. (b) 98. (d) 99. (d) JOO. (b)
101. (d) 102. (c) 103. (c) 104. (b)
105. (b) 106. (d) 107. (d) )08. (d)
109. (c) 110. (a) 111. (a) 112. (cl)
113. (c) 114. (a) 115. (a) 116. (c)
117. (a) 118. (d) 119. (b) 120. (b)
121. (d) 122. (a) 123. (a) 124. (c:l
125. (c) 126. (b) 127. (d) 128. (a)
129. (h) 130. (a) 131. (c) 132. (c:)
133. (c) 134. (b) 135. (d) 136. ((,)
137. (n) 138. (c) 139. (b) 140. (e)
141. (d) 142. (a) 143. (b) 144. (11)
145. (b) 146. (a) 147. (a) 148. (d)
149. (c) 150. (b) J SI. (d} 152. (C)
153. (b) 154. (d) 155. (d) 156. (d)
157. (b) 158. (d)

100
" NEUROLOGICAL AND
IW .
_MUSC_ULA_R_D__S_O_RD
E_R ,
1. Cla w hand is seen in
(a) Ulnar nerve injury
lb) Carpal tunnel syntJrome
(c) Syringomyelia
(d) Cervical rib
PD/ 85.Kemla 88. .ILPMER 86.t\11MS
85
2. Total Claw hand is caused by injur:y to
(o) Radial Nerve
(b) Ulnar and Radial Nerve
(c) Ulnar and Medial nerve
(d) Radial and Median Nerve Al-'- ft,'fDA 93
3. Polio paralysisdiffers from paralysisdue to other causes
(a) Weakness
(b) Deformi ty of limbs
(c) No sensory loss
Cd) Full recovery is possible KARNATAKA \14
4. Foot drop is seen in
(a) Tibi:.il nerve injury
(b) Achilles tendon injury
(c) Poplitcal nerve injury
(d) Comm01, peroneal nerve injury D/:,LLJJ.1988
5. The most common cause of pressu1·e sore in the foot in
India is
(a) Leprosy
(b) Thora prick
(c) Diabetes

101
(d) Syringomyelia KERALA.1988
6. Polio paralysis differs from paralysis 1lue to other
c:iuses
(1!) Weakness of muscles
(h) Deformitv of limbs (c} No
sensory los
(d) FuU re.covery is possible. KARNATAKA: 1994
7. Club foot is commoner among
(a) Males
(ii) Binovular twins
(c) females
(d) Uuiovular twins RA.IASTAN. / 99'.!
8. Which of the following statement is wrong in Tendon
transfor?
(a) Contractrue should be released priorl y
(h) Synergistic m useles are used for tendon transfer
(c) Adequate tendon should he mobilized lo gain length
(d) A IIof the above NIMS: 2000

9. Which of the following is not t r ue about Myosi tis


ossificaos?
(a) Associated with musde Lendon ruptu re
(b} Inflammation around Lhe ruptured muscle dcpos1110n
of hydroxyapatitc crystals with
(c) Common in supracondylar fracture
(dl Ossification of musculo-perio,teal haematoma
PC/ : 201!0
10. Tendon transfer in a 3 year old boy of poliomyelitis is done
(a) Within 6 months of wound healing
(b) After 2 ycan;
(c) Within 6 mont hs of wound henling
(d) After 6-12 months.
ll. The most common site of ivory ostema (Compact osteoma) is
(a) Pelvis
102
(b) Mand ible

103
(c) Skull
(d) Vertebrae BIHAR:199CJ
12. The hasic pathology in Myositis Ossificans Progressiva
is in
(a) Muscle fibres
(b) Serum cbemisLry
(cl Body collagen
(d) None of the above NATIONAL BOARD: 1990
13. Upper motor neuron type paralysis is seen in
(a) Polio11Jyelitis
{b) Peripheral neuropmhy
(r ) Cerebral palsy
(d) Muscular dysLrophy
(e) Paraplegi:1 due to L2 fractures dislocation.
PGl. 1990
l4. Hypercalcemia is caused by all
except
(a) Multiple Myeloma
(b) Hyperparathyroidisrn
(c) Sarcoidosis
(d) Myosi1is ossificans progressiva AflMS: !99/

I ANSWERS I
1.
(a)
2. (C) 3. 4. (d)
S. (c)
6. (c) 8. (b)
(c) 7. 12.
9. (11)
\a) JO. (b) (a)
14. (d) 11.
13. (c) (c)
m
OF
REGIONAL CONDITIONS

· c.N_ECK A_ND UPP_E_R_L_IMB ,


l. Dupuytren 's contracture is;
(a) Thickeni ng of palmar fascia.
(b) Base of lillle finger involved
first.
(c) Seen in cirrhotics. .
(cl) Seen in epileptics on hydantoin.
A.l.88
(e) All of the above.

2. Dupuy11·e11s con tr·act.ure is


fibrosis of
(a) Palmar fascia
(b} J'oream1 m usdes
KARNAT 96
(c) Sarto1ius fascia
(d) None of the above
3. Al'ler L4-L5 or LS-SI the next commonest site of'
.lJltervertebral disc prolapse is
(a) C7, T l
(b) T l2, LI
(cl L I L2
(d) L2 L2
4. Osteoph ytes developi ng at the join t at Luscka
characteristically compresses spinal nerves at
(a) Lntcrvenebral fornmen
(b) Antcri CJr part of body
(c) Posterior part of b,ody
(d) Parndural areas NTMS 96
5. Carrying angle is decreased in
(a) Cubitus varus
(b) Cubims valgus
(c) Gcnu valgum
(cl) Genu varum A
.P.97
6. The nerve involved in Carpal tunnel syndrome is
(a) Ulnar
(hJ Rad ial
(c) Median
(d) Anterior cuLaneou;; nerve.
7. The level of the constricting nodule in case of a Trigger
finger is a!
(a) Neck of the correspondi ng metacarpal hone
(b) Mcmcarpophalangeal joint
\c) Proximal interphalangeal joi n1
(d) Distal iotcrphalangeal joint

8. True about Dupuytren's contracture


((I) Slight preponderance in males
(b) Slight preponderance in females
(c) Much more common in ma les
(d) Much more common in females OELHI : /994
9. Thoracic Kyphosis in children is most often due to
(a) Traunu1tic
(b) Por.t's spine
(c) Congenital
(d) Normal lJEUil:/ 994
10. The best treatment for Dupuytrcn 's conh·actrue is
(11) Fasciotomy
(b) £'asciccromy
(c) incision and release
(d} Fasciectomy + Skin transplantation. ANDH RJ\ 1994
11. Tinel's sign is positive in
(a) Peripheral nerve regeneration
(b) Tendon injury
(c) Tenosynovitis
(cf) Rheumatoid arthri ANDHRA: l'J'N
ti
12. All of the following are true about Duchen ne
muscular dystrophy, except
(a) Both sexes are affec1ed
( /J ) Peu<lohypcnrophy
(c) Hereditary
(d) Death in second decade. AN Df!Rll: / 994
13. The most common cause or scoliosi& in children is
(a) Hemivcncbrae
(b) Unequal limb length
(c) Post poliomyel it is
(d) Maifan 's syndrome .IIPM/iR: 1993
14. The least common cause of brachial neuralgia is
(a) Cervical spoadylosis
(b) Pancoast tumour
(c) Cervical rib
(dJ Ticrze· syndrome ANDHN!l /993
15. The thoracic outlet syndrome may be produced by all
the following except
la) Cervical ri b
(b) Scalenus imticus syndrome
(c) Costoclavicular comprcsion
(d) Raynaud 's disease
(e) Hypcrabducrion syndrome NIMS: 1992
16. Pbalen 's test is positive in
(a) Carpal runnel syndrome
(b) De Quervain 's diease
(c) Tennis elbow
(d) Ulnar bursi,is
17. Cervical spondylosis is more common at
(a) C1-C2
(b) Cz-C3
(c) C6-c7
(d) C4·C5
18. Which of the following movements arc restricted in
Frozen shoulder"!
(a) Abduction & lmemal rotation
(b) AdducLion & external rmmion
(c) All range of movements
(d) Only abduction ANDHRJ\: 2001)
19. Consider the following statements about Car1>al tunnel
syndrome
1. It may occur in Acromegaly
2. Itmay occur in pregnancy
3. It causes delayed ulnar nerve concluclion
4. Itmay he associated with wasting of abductor
pollicis brevis
of Lhcse statc-mcms
(a) L.2 and 3 are correct
(11) 2,3 and 4 are correct
(c) l.2 and 4 are correct
(d) I.'.l ru1d 4 ace correcL UPSC:!999
20. \.\'hicb one of thefollowing is a correct statement
regar
ding idiopathic Scoliosis?
(a) Lateral curvature of spine
(b) Curvature in ru11cro-posterior direction
(c) Cu rvature in postero-anterior direction
((!) Curvature in diagonal direcLion.

21. In de Quervain's 1lisease the following telldons are


involved
(a) Abductor pollicis longus + Extensor pt,tlicis brcvis
(b) Abductor pollicis brevis + Extensor pollicb lungu
(r) Adductor pollici brevis + Extensor policis Jongus
(ti) Extensor pollicis longus + Fle.xor pollicis longu,
NIMS:2000
22. In cervical spondylosis whi('h part of vertebral body
involved
(a) Tnferior articular facet
(b) Pars interarticu I aris
(c) Superior articula r facet
(d) All of the above. OEU-/1: 1999
23. In carpal tunnel sync!J-ome, features arc of
(Cl) Compression <>f ulnar nerve
(b} Compress ion of med i a n nerve u nder the flexor
retinaculum
(c) Anaesthesia over lhenar eminence
(cl) Atrophy of hypothenar muscles JIPMEI? . /999
24 The foUowing structure is involved in Oupuytren's
con tractrue
(a) Thickeni ng of the palmar foscia
(b) Thicken ing of the dorsal fasda
(f') Contractruc of the flexor tendons
(d) Posi burns comractn1e ALL IND/A : / 994
25 Trigger finger is
(a) A feanu-e of carpal tun nel syndrome
(b) Injury to fingers while operating a gun
(c) Stenosis tenovaginitis of flcxor tendon or affected
finger
(d) Any of the above. NATIONAL BOARD :
2000
26 Cervical spondylosis
(b) Most frequently , ults from an incidence of acure
trauma
(b) Causes compression of nerve roots to produce ru1
upper motor neuron lesion i n 1·he lower limbs
(b) Prnduces pain and Paraslhesia over the lateral
apect of the forearm and thu mb when affecting the
61h cervi ca l nerve
(b) Most frequcruly affects tl1e u pper cervical venebrae.
Blf!AR : 1998
27. Dupuytrcn's contractrue is seen in
(a) C<>lles' fracture
(b) Thickening or plamar fascia
(c} Radial nerve palsy
(d) Supracondylar fracture of humerus PUl:1998
28. Trigger linger occurs in
(a) Rheummoid anhrilis
(b) Trauma
(c) Osteosarcoma
(d) Osteoarlhriti PG!: ICJ<J8
29. Carrying angle is decreased in
(a) Cubitus varus
(b) Cubitus valgus
(c) Genu varum
(cf) Genu valgum AN DHRA: / 997
30. In Children, facture neck of fomu r is best treated by
(al Plaster in abduction
(b) Closed reduc:tion (Internal fixation)
(c) Plaster i n adduction
(d} Traction DEL/11: 1990
31. True dislocations or spine are commonest in the
(a) Lumbo sacrnl juncLion
(b) tvUd lumbar region
(c) Cervical region
(d) Dorso lumbar junction DEL/ II: 1990
32. Painful arc syndrome is due to
(a) Fracture of greater tubercle of humerus
(b} Chronic supraspinanis tcndonitis
(c) Subacromial bursiLis
rd) Al l of the above. Jf PMER.·/ 990
33. The most common deformity in club foot is
(a} Equinus
(b) Equ ino valgus
(c} Equino varus
(d) Calca11eo valgus ORfSSA: / 990
34. Trigger finger is due to
(a) Tenovagiitis
(b) Synovitis
(c•) Bursitis
<d) Fibrnsi tis OR/SSA: 1990
35. Gradual painful limitation of shoulder movements in
an elderly suggest that the most probable diagnosis is
(a) Arthri1ig
(b) Osteoarthritis
(c) Periarthri tis
(d} Myositis Ossificans
(e) Fracture - dislocation ON/SSA: 1990
36. Adson's test is performed in
(a) Sca lcnus anticus syndrome
(b) Cervical Rib
(c) Both of these
(d) None of these BIHAR: 1991
37. Which nerve is involved in carpal tunnel syndrome?
(a) Ulnar nerve
(b) Radial nerve
(c) Median nerve
(d) Posterior cutaneous nerve of the forcnrrn.
B!HAR: / 99 /
38. Du puytren 's Contractu re of the hand commonly
starts in
(a) Thu mbs
(b) Index finger
(c) Midd le finger
(d) Ring finger
(e} Little finger only. B!HAR.·/ 99/
39. Dislocation of spine is commonest at
(a) c,c6
(b) c.-c,

11
0
<b) D,,-L,
(<") L,- L,
(a) All of these. Bil/AR: 199 I
(d)
40. Trigger finger i!l caused by
(a) Rheumatoid anhrit is
Osteoanr
(b) Tcno,ynovitis
uilis
Delhi:
(c) Colics· fracture
/99/
41. The most
common
form of
Scoliosis
ii;
(
a
}

l
d
i
o
p
a
t
h
i
c

s
c
o
l
i
o
s
i
s
( i
h c
)

s
C
o c
n o
g l
e i
n o
s
i i
s
t (d}
a Postural
l scoliosis
ANDI IRA
: / 991
s
c 42. Cubilus
o valgus
l may
i actually
o predispo
s se
i become
stretche
( d or
< exl)osed
" unduly
)
to
trauma.
P
(a) radial
a nerve
r (b)
a median
l nerve
y (c) ulnar
t nerve
(d} All of thee.
43. Regarding carrying angle,
which is correct'r
(a) ft is about IO" in male
(b) ll i aut>ul 20° in
(male
(c) The carrying anglein
male i. always greater than
tcmale
(d) Only a and b.
44. Tennis elbow is:
((I) Olccranon bur.;itis
(b) Pain over the med ial
epiconclylc
(t) Pain over the lateral
epicondylc.
(d) Myositis ossificans.
45. Finkelstein's test is
associated with
(a) Dequervai ns disease
(17) Dupuytren·$
Contr:acturc

1
1
1
(c) Carpal tunnel syndTome
(cf) Any of Lhe above.
46. Which movemen t at shoulder gets restricted when
Strpraspinatous torn?
(a) Flex ion
(b) Adduction
(c) AbJuctiou
(d) Rotation only.

I ANSWERS I
1. (e) 2. (al 3. (None) 4. (ll)
5. (a) 6. (cl 7. (b) 8. (c)
9. (c) 10. (c) 11. (ll) 12. CaJ
13. (c) 1.4. (d) 15. (d) l6. (a)
17. (c) 18 (b) 19. (c) 20. (a)
21. (a) . (a)
22. 23. (b.c) 24. (ll)
25. (c) 26. (c) 27. (h) 28. (a)
29. (a) 30. (b) 31. (c) 32. (/1)
33. (c) 34. (a) 35. (c) 36. (ll)
37. (c) 38. (d) 39. (a) 40. (b)
41. (a) 42. (c) 43. {cf) 44. (c)
45. (a) 46. (c)

112
m REGIONAL CONDITIONS OF
..T.HE SPINE AND LOWER
LIMB ,
I. Commonest cause of loose bodies in joint5;
(11) Tuberculous lenosynovitis.
(b) Rheumatoid arthritis.
(c) Osteo arlhritis.
{d) Osteochondrit.is clt\scican. A.l.l:J8
2. Traction injury to epipbyses of the Yertcbra is known
as;
(a) Osgood Schlatter":,. disc:-.a e.
(b) Sinding Larsen dis:ease
(c) Scheurmaun ·s d isease.
(d) Severe's disca.\e. .IIPMF.R '8
3. Commonest site of Disc prolapse is
(a) C5-C6.
(b) T8-T9.
(t) L4-L5.
(d} l5-S I. KERAIA.90
4. The following is true of sponrlylolisthesi.
Ca) Slipping of SI over LS.
(h) P()Sterior arch defect.
(c) Congenital defect.
(d) More in pregnancy PG/.9(1
5. Recurrent dislocation of patella in an adolescent could
be treated by
(a) Patellectomy.
(b) Excision arthroplasty.
(c) Puttiplat operation.
(d) Lateral release PGl.82
6. Causes of paiofuJ limb arc all except
(a) Perthe's disease.
(b) Congenital Coxa vara.
(c) Sl ipped femoral epiphysi>.
(dl TB hip. ALL IND/t\.95
7. [n Hallux,·algus surgery, the patients who are likely
to be most satisfied are:
(a) TI1ose with pai n.
(b) Those with hammertoe.
(cl Those wilh metatarsus primus varns.
(d) You ng age. AIIM S.95
8. Slipped femoral epipbysis i.s comonly seen in the
(a) I" decade
(b) 2'"' decade
(c) 3"' decade
(d) 41" decade A./.88

9. Rocker bottom foot re$ull from


(ll) Congenital vertical lalus.
(b) Poliomyei tis
(c) Club foot over correction
(d) Spim1 l>irida PD/ 88
10. Coxa vara is round i11
(a) Pe1the's disease
(b) Tuberculosis
(c) Rickets
(d) Rheu matoid arthritis PG/ 8(1
11. Idiopathic scoliosis is
(a) A lateral curvature or the spine
(b) Rotation of the spine
(c) Lateral curvature with rotation of the
spine

114
(d) Flexion defonnity of the spi ne 8/ HAR./ 988.

115
12. The most common site of Disc prolapse is
(a) L
-L... (bl
L3-L, (cl
L,-L,
(d) L,-S1 DELH I. / 988

13. The most common site for Prolapse of intervertcbral


Disc is
(a) Cervica l region
(b) Lower thoracic region
(c) Upper thoracic region
(d) Lwnbar region ORISSA: / 98/i
14 Plantar fasciitis
(al is not associaced with infection else where i n the body
(b) is caused by a bony spur on the plantar surface of the
OS cakis
(cl can be relieved by supplying insoles
(d) is a type of Dupuytren·s disease. Jlf'MER:1981.)
15. The most common catL e of quadriceps femoris
fibrosis is
(a) Arthrogryposis
(b) Trau ma
(c) Repeated injections
(d) Chron.ic osteomyclilis of femur ANDHRA.1989
16. Common vertebral level of spondylolisthesis is
(a) L..._
5
(b) L3_.
(c) L1.?
(d) T,2.u
(el Ls-S, RIHAR.1989

17. After L,-511 the next commonest site of


intervertebral disc prolapse is
(al C6-C,
(bJ T11-L 1
(c) L -L
1 1
(d) L:-L1
(el L.,-L,
18. Narrow lum bar canal syndrome
(a) Associated with neurological claudication
(b) Caue pain on rotation of pine
(c) Can be diagnosed. only by Myelography
(d) May lead to spastic piu-aplegia
(e) Occurs due LO congeniml venehra l anomaly.
PG/: /969
19. The most importa n t single special investiga1ion in
lu mba r disc prolapse is
(a) Epidurography
(b) Myelography
(c) MRI
(d) Discography
(e) Spinal venography NAT!ONAL BOARD: / 992
20. Known factors for idiopathic scoliosis is
(a) U nknown
(b) Polio
(c) Postural
(d) Congeni tal DELHI: !992
21. HaUux valgus means
(11) Outward deviation of great toe
(b) Inward deviation of great toe
(c) Outward deviation or
l'ifth toe
{d) Inward deviation of fifth toe ANDHRA : '2000
22. AJI are tnie of Dupu ytren's cont ractrue except
(a) Usually 411' linger i s involved
(b) Bilateral disease .is rare
(c) Surgical release Ls useful
(d) May be associated with Pyronie disease
PG/: / 999
116
23. Which sign is positive in de Quer vain 's disease'!
(a} Phalcn's sign
(b) Fromcn t', sign
(c) Cozcn's sign
(d) Finkclstei n's sign MAN/PAL : 1000
1
24. hich one of the following is a correct statemen t
regarding idiopathic scoliosis?
(a) Latera l curvatu re of spine
(b) Cu rvai-ure in amcro-posterior direc11on
(cl Gun1ature in posrero-ruitcrior direction
(d) Curvature diagnol direction NIMS : 2000
25. Milwaukee brace is used for
(a) Club fcxll
(b) Spon<lylol isthesis
(c) Scoliosi
(a) Knock knee DEUIT: /999
26. Rocker bottom feet deformlty is caused b)
(a) Vertical tal us
(b) Prolonged immobi lization tn plaster and Ovt!
rcorrenion
of cluh feet dcforrni ty
(c) Over con-ection of Grac1s operation
(d) Pc>Liornyeliti·;

27. HaUm:valgus is associated with aU except


(a) An exostosis on the medial side of 1.he head of 1.he
first metatarsal
(b) t\ bunion
(c) Os1.eo arthritis of the metatarsophalangeal joint
(d) Over-riding or undc r -riding or the econd toe by
the third. PG!: Dec -
2000
28. Abnormally high patella is associated with
(a) N,liI patella syndrnme
(b) Rupture of quad riceps tendon
117
(c) Recurrem dislocation
(d) Fracture through p,ttell,1 UP : 1999
(cl)
30. Rocker Bottom f'oot occurs in
Conge
((I) HalJ ux va lgus
nital
( b) Hallux rigidu,
club
(c) Copngenital vertical talus
foot

KARN
A
FAKA
: 1999
31. Mana
gemen
t in
case
of
ruptu1·
e of
disc at
L5.Sl
is
(a) E
m
er
ge
nc
y
re
m
ov
al
of
di
c
(b)

118
Joint fusion
(c) lmmobiliwtion for 2
weeks with pi naJ back
(d) Traction
32. \Vhich one of the following
statemen t regarding bow leg
is correct
ta) Physiologically c(>rrccted
(b) Surgery is required as
c:u·ly as pos,ible
(c) IL i a progressive
deformity
(d) ldiopaU1ic variety is
rarest type
33. Commonest cause of
scoliosis is
(a) Idiopathic
(b) Traumatic
(c) Congeni tal
(ii) Tuberculosis.
1998
34. In spondylolisthesis, there is
fractu re of vertebra in
(a) Spinous process
(b) Neural arch pars inter
articula1is
(c) TraJ1Svcrse process
(d) Body
35. Cobb's angle is mea..<:ured
for
(a) Lordosis
\b) Lateral 11cxion
(c) Kypho,is
(d) Scoliosis

119
36. Rocker bottom foot is due to
(a) Congenital
(b) Over correct ion of dub foot
(c) AbsenL navicular bone
(d) Flacid medial longitudinal arch ANf )HN: / 999
37. The most common cause of acute sciatic'.l is due to
(a) Trau ma
(b) Secondaries of spi·ne
(c) Acute prolapsed lntervertebral disc
(d) Tuberculosis of spine t\NDHRA: / 999
38. TI1e carpal tunnel syndrome usually occu r in
(a) Cushing"s disease
(b) Addison·s uisea. e
(cl Acromegaly
(d) All of the above BIHAR: / 999
39. Ln cervical rib following are seen except
(a) A trophy of muscles
(bl Cervical ri b pa lpable
(c) lschaemic pain of muscles
(d) Radial pulse not pal pable PG/: /997
40. Slipped femoral epip.hysis
(a) Tends to occur in overweight boy between 10- 18
yar,,
(b) ls fortunately a unilateral condition
(c) Is a forward andamcrior slip
(d} May be sudden or slow
(e) Is confinned by a lateral X-ray of the hip
N,\T!ONAL BOARD: /
990
41. Fibrositis is commonest in
(a} Tcndocalcaneus
(/J) Stcrnoclcidomastoid
(c) Trapezius
(d} Serratus anterior NATIONAL BOARD: 1990
42. The primary defect inflat foot is the following
(a) Weakness of shon plantar ligamcm
(b) Collapse of lateral longitud inal arch
(c) Collapse of media l longitud i nal arch
(d) Shortening of plan tar aponeurosis ORtSSIA:/ 990
43. A building contractor suddenly complains of lower
backache which increa5e on bending down Re bas
(a) Renal colic
(b) Tuberculosis of spine
(c) Disc prolapsed
(d) Fibrositis' UPSC: / 990
44. W1tich of the following cysl is medially situated"?
(a) Housemaid's knee
(b} Clergyman· knee
(c) Bursa anscrine
(d) Semimcmbranosus bursi tis
(e) Mnrrant Baker· cyst NATIONAL BOARD:
1991
45. True about spondylolisthesis is I are
(a) Congenital clefcct of posterior arch
(b) Slipping of L5 over SJ
(c) Prngressive slippi ng
(d) Abnormal congenital development
(e) Commonest in cervical region. PG I : 1991
46. Which of the following causes kypbosis?
((1) A n kylosing spondyl i tis
(b) T.B. of spinal column
(c) Scheuem1ru1's osteochondritis
(ti) All of the above.

I ANSWERS I
I. 5. (d) 6. (b)
(c)
2. (<")

120
3.
(c) 4.
(b)
7.
(d) 8. (/,)

120
9. (ac) 10. (a) 11. (c) 12. (cl
13. (d) 14. (t'J JS. ( c) 16. (a)
17. (a) l8. (a) 19. (cl 20. (al
21. (a) 22. (b) 23. (d) 24. (a)
25. (c) 26. (ab) 27. (a} 28. (c)
29. (a) 30. (cl 31. (c) 32. (a)
33. (a) 34. (h) 35. (cl 36. (h)
37. (c) 38. (c) 39. (d) 40. ((Ide)
41. (c) 42. (c) 43. (c} .t.i. (a/w)
45. (d)

Em
CES IN ORTHOPAEDIC SURGERY APPEND1C

1. Formula of dry Plaster of Paris is


(a) CaS00 112 H2 0
(b) CaSOJ only
(c) CaSO, 2H,O
(d) CaS04 5H;o
(e) MgSO., 2HO B!HAR./ 9X9
2. The best bone graft is
<a) AJlogran
(b) Autograft
(c) Deprmei11ised graft
(d) Demineralised graft

3. Treatment of choice for fracture shaft femur in childrl!n


(a) Gallows or Russell's ttactioo
(h) Internal fixation
(c) Kuntscher nail
(d) Thomas splint
4. Rone chip grafts are used in
(a) Osteotom y
121
(b) ln Arlhrodesis
(c) Arthrnpla. ty
(d) Pseudoanhrnsis
(e) Filling gap after removal of tumour. PG/.· 1!19 /

I ANSWERS I
1. (a) 2. (b) 3. (a) 4. (b.e.cl)

I l IJNOrI_NRTSIEs_T_o_B_o_NEs_A
f i ND ,

i .
1. Myositis ossificans is most common around lhe ......
joint
(a} Knee.
(b) El bow.
(c) Wrist.
(d) Hip
2. The most important sign in Volkma nn 's ischaemic
contracture is;
(a) Pain.
(b) Pallor.
(c) Nu mbness.
(d) Obliteration of radial pulse. TN.90
3. Treatment of Acute Myositis Ossificans is
(a) Active mobilization .
(b) Passive mobi lization .
(c) Infra Red Therapy.
(d) Immobilization. A/IMS. YI
4. \Vhich fracture in children requires open reduction:
(a) Fracwre tibial epiphysis.
(b) Fracture shaft of femur
(c) Fractw·e both bon.es foreann.
(cf) Fracture femoral condyle Al!M S.91

5. Volkmann' s ischaemic contracture mostly involves


(a) Flexor digitorum superficialis.
(b) Pronator leres
(c) Flexor digitorum profundus.
(d) Flexor carpi radialis longus A /IM S.92
6. The most important factor in fracture healing is;
((I) Good alignment.
(b} Organisation of blood c:101.
(c) Accurate reduction and l00% apposition of fractured
fragments.
(d) Immobilisation.
(e) Adequate calciu m intal,,e. ROHTAK.X9
7. A Lady presents with a History of fractureradius.
which was put on plaster ()!' Paris cast for 4 weeks.
After that she developed swelling of bands witb shiny
skin. What is the most likely diagnosis?
(a) Rupn1re of cxten.sor prolicis longus tendon.
(h) Myositis ossifican,
(c) Rc11ex sympathetic c.J ysu·ophy.
(cf) Mahmion J\lllv/S.2K
8. Which among the following benefits from cervical
sympathectomy
, (a) Sudcck·s dystrophy
Cb) Compound palmar ganglion
(c) Ostel'rartl)lit ig of first MCP joint
(d) De qucrvain's tenosynov itis A!IMS 99
9. In Volkman's ischcmia,surgery shouJd be done:
(a) lnm1ediately.
(/J) After 6 hou rs.

123
(c) 24 hour.:
(d) 72 hours. AA1U
117
10. The treatment of choice in pathological fractw·es is
(a) Interna l fixation
(b) Plaster Clf Pari casts.
(c) Skin traction
(d) Externa l skeletal fixation . RII /AR / 988
11. Volkma nn's Ischaemic contracture is due to
(a) Arterial injury
(b) Venous injury
(c) Nerve injury
(d) lncrease of compart ment pres Ltre in the l imb.
KARNATAKJ\: / 9{;8
12. Earliest Ischaemic feature after reduction of Supra
condylar fracture is
(a) Coldness
(b) Pain
(c) Swelling
(d) Tingling ANDHHA.1989
J 3. The time necessa1·y for healing of fracture depends on
the following factors
(ll} Age of the paLient
(b) Locati\ln of the fracture
(c) Type or the fracture
(d) Degree of damage to soft tissues
(e) All or the above. NB: 1989
14. The most important factor in fracture healing is
(a) Good alignment
(b) Organisation of blood clot
tc) Accurate reduction 8nd I 00% apposi tion of f'racn,rcu
fragments
(d) Immobilisation
(e) Adequate calcium intake.
124
15. The most common cause of an terior compar tmen
t synd rome is
(a) Fracmre
(bJ Post ischaemic swelling
k) Superficial injury to muscles
(cl) Operati ve trauma_ !'GI: 1993

16. Earliest symptom of Volkmann' s lschaemic is


(a) Pain in flexor muscles
(b) Absence of pube
(cl Pain on passive extension
(d} Cvanosis of limb .1/PMEN: / 993

17. Fat Embolism occurs in


(a) 1-lypcrlipidernia
(b} Diabetes mellilus
(c) Castor oil intake
(d) Fracture femur ANDHR.4:/ 993

18. Earliest sign of Volkmann's lscbaemia is


(a) Pallor distally
(b) Numbness
(c) Pain
(d) Parasthesia ORJSSA: / 992
19. All of the following are iudjcations for open reduction
and internal fixation of' fractures except
(a} Compound fracture
(b) Unsatisfactory closed reduction
(c) Multiple trauma
(d) Intra-articu lar fi:acturc CSE : 2000
20. A 4 year old female brought to casualty department with
multiple fracture ribs, and inconspicuous history from
pa rents. On examina tion show multiple bruise and
healed fractures. The provable diagnosis is
(a) Polytrauma for evaluation

125
(b) Fl ail chest
(c) Munchausen 's syndrome
(d) Battered baby syndrnme NIMS : 200V
21. After an operation on femur bone, chest X-ray shows
widespread mottling throughout the lu ng lield like a
snowstorm. Itis diagnostic of
(a) Fat embolism
(b) Shock lung
(c) Bronchopneumonia,
(d) Atelectasis CS£: I 991.J
22. Sudeck's atrophy i1; associated with
(a) Osteoporosis
(h) Osteophyte fonnation
(c) Osteopcnia
(d) Osteochomlritis DELHI JCJ99
23. Last step ill fracture heali ng is
(a) Haematoma
(b) Callus f'ormarion
(c) Remodelling
(d) Consolidation RAJAST!i4N : 1993
24. lnlcrnal fixation is proba bly needed in all of the
following except
(a) rracture condyle of humerus
(b) Fractu re neck of femur
(c) Fracture of Olecranon
(d) Fracture of scaphoid UPSC:J9CJ3
25. One of the following is associated with Volkmann's
ischaemic contractrue
(a) Supracondylar fracture of humerus
(b) Fracture shaft humerus
(l') lntercondylar frach.trt! of humerus
(d) Dislocation of elbow. PG/ : 2000

126
26. A patien t presenting with Volkmann' s lschaemia all
of the following are done except
(al Split open the plaster of Paris cast and bandage
(b) Dccom pression by fasciotomy
(c) Explormion
(d) SympaU1ctic ganglion blockade PC/: 2000

27. All arc causes of Pathological fracture except


(a) Anaemia
(b) Osteoporosis
(c) Radiation
(dl Osteomalacia PG/ : 2000
28. The most common cause of scoliosis is
(a) Idiopathic
(b) Congenirnl
(c) Paralytic
(d) Postural PG! : 199./

29. Pathognomonic sign of trnumalic fracture is


(a) RedneS$
(I>) Swelling
(c) Crepitus
(d) Ten<lerness .1/PEM ER:/ 1)98
30. Sudeck's atrophy is more common in
(a) Malunitcd Colle. ·fracmre
(h) Malunited fraclltre femur
(c} Pott"s fractu re
(d) Carries spine KARNATAKA: / 998
31. The most common cause of non union is
(a) Lnfecti.on
(b) Inadequate immobilization
(c) lschaemia
(d) Soft tissue interposition UP: / 998

127
32. A 6-yea r-old child falls in right-sided forearm region
and develops fracfure in dorsal surface of mid region
of' radius. The best t reatDlcnt is
(a) Antibiotics & sedative
(b) Bone plati ng and externa l fixat ion
(c) Slab with wait for bone remodeling
(d) Break the cortex other side ,md immobi lization by
POP.
UP: / 998
33. Volkrnann's contractrue
(a) ls localized thickening. of pal mar fascia
(b) Develops at the ankle jJl a case of chronic venous ulcer
(c) Follows lschaemia of the forearm
(l[) rs due lo excessive scarnng of the skin of the a.nn
following a bum. UPSC:/
998
34. Dislocations occu r most frequently in the
(a) Shoulder joi nt
(b) Elbow joint
(c) Hip joint
(d) Knee joi nt AN DHRA: I 999
35. Which of the following statements pertaining to green
stick fracture is true'!
(a) Any fracture in a ch ild
(//) Fracture only i n rickety children
(c) Only if t here is no deformity
(d) All of the above. AN DHRA: 1999
36. One of the features given below is essential in Lhc
diagnosis of a fracture of a bone
(a) Deformity
(b) A crepitus
(c) A partial or complete Joss of continuity of Lbc bone

128
(d) None of the above. ANDH RA:/ 9YY

129
37. The most commoo nerve involved in Volkamann's
iscbaemic contracturc of fore arm includes
(a) Radial
(b) Ul nar
(c) Median
(d) Posterior imerosseou ALL INDl1\: / 999
38. Fat Embolism is commonly due lo
(a) Fracl ure femur
(bJ Fract11re both bones of fore aim
(c) Fracture calcaneum
(d) Crnsh injury of foot AIJMS: / 999
39. A 40-year-old patient sustained leftshoulder injury and
developed antei-ior dislocation of shoulder joiul
causing sensory loss over latct"al side of forearm.
The nerve involved is
(a) Radial nerve
(b) Auxiliary nerve
(c) Musculocutaneus nerve
(d) Ulnar nerve A/IM S: 1999
40. Stellatc ganglion block is mainly used fo•·
(a) Compound palmar ganglion
(b) Dequervain's synovi1is
(c) Sudek's dystrophy
(a) Osteoarthritis of carpometacarpal joint. A/IMS:/ 99Y
l. A 23-year-old female bas brought lo the causality after
sustaining a road traffic accident.When she was going
to her residence in her vehicle, lhe bike skidded which
lead to a cross injury to her with deep bone exposure.
The best graf t used to cover her exl)osed part of hone
is (a} Full thickness graft
(b) Partial thickness grafl
(c) Pedicle g-raf1
(d) Muscle nap with skin graft AJJMS:1999
42. Match List-Iwith List-Tland select lhe correct
answer using lhc codes briveu below the lists.
List J List ll

(11) l . Complicaticm or fmclurc


Volk mann's comraciure
neck of femur
(b) fal embolic syndrome 1. Often assocmte<l with
upraconJylar fracllH'C
of humcru
(c) Nerve 3. Common i n J'raciurcti hia.
injury 4. Complication of f'racLu re
(d) :'Ion u nion humeru,
CODES :
A R c D
A 3 2 I 4
B 2 4
c 3 2 4 l
D 2 .) 4
43. Volkmann·s ischacmic contracturc is co11m11mly due
to
(11) Tight plaster
t br Tigh1 splim
(c) Both
(r/ ) None B!HAR: J()VO

44. Fractw·e shaft of femur in adult unite!> by


(11) 3 lo 4 weeks
(b) 3 ro 4 weeks
(c) 3 to 4 months
(d) 4 to 6 months
(e} None of these R/1/AR: !990
45. Myosilis ossifirans is due to
(a} Ossiticacion
(b) New bone fonnaticm
(c) Ossification of subperiostcaJ haematoma
130
(dJ Migrn1ion osieoblat to haematomll
(e) All or 1he above. BIHA R·J9'JU
46. Initial stage of clinical u nion of bone is equivalent to
(a) Ca l lu; fonnmion
(b) Woven bone
(c) Haem,1toma form,11ion
(1[) Calcification only
(e) None of !lie ahove 811-/AR: J 9WJ
47. The most com mon cause of pathologi<:111 fracturei;
(a) Delayed llllion
(bl Mal union
(cl Non u nion
(rl) Secondary ucpoits
(1,) Any of these BIHAN: / 990
48. Delayed u nion or fracture of a hone following a
surgical treatment may be due to
(a) nfection
(h) Inadequate circulation
(<") Toadcquate mobi lization
(dl Al l or 1he above. NATJONAL BO,\RD: / 990
49. A recessive form of Os(cogenesis impert'ecta may closely
n:scmblcs
(a} l\.lkaptonuria
(/,) Cretinism
(c) 1-lypophosphata in
(d) Homocysti nuria N,\TIONAL BOAi?[): / 990
50. The most common complica tion of' suprncondya r
fracture is
(a) VIC
(b) Cubirns varus
(c) Ma luoion
(d) Delayed union

13
1
51. The one most consistent sign of fresh fracture is
(a) Crepitus
(c) Bony tenderness
(d) Deformjty
(,[) Abnom1al mobili y
(e) Shonening of bone.
52. The most importan t sign in diagnosing Vol k man n's
ischaemia of forearm is
(a) Pallor
(b) Pain
(c) Parathesia
(d) Pulselessne$S of radial artery TAMIL Ni\DU./ 9'}/
53. Treatment of choice in Acute myositis ossilicans is
(a) Immobilization of elbow
Cb) Short wave diathermy
(c} Passi ve movements of am,
(d} Active exercises 7i\MIL NADU: 199/
54. Volkman n 's Ischaemic Coutracture is due to
(a) Injury to ul nar and median nerve
(b} Injury 10 median nerve alone
(c) Contrnclllrc of lhe pal mar fascia
(d) Ischaemic vascular inju'ry 1n lhe muscle
(el All of tbe above. BIHII R: 1991
55. Which of the following is regarded as a definite sign of
fracture?
(a) Tendeme,s
(b) Local bony in-egularity
(r) Crepitus
(rfJ Swelling
56. Strike the false statement(sJ
(a) In sprain the ligament is often tom
(b} In dislocation there is no contact in between the bmh
the component of the joint
132
(c) Sclerosis of the fractured ends ind icate ab,olute
nonunion
(d) None of the above.
57. Following are recognized F/0 Myosi tis ossifica ns
EXCEPT:
(a) lt is a post u·aumalic ossification
(b) It follows either a poster ior d isloca t ion or ,,
supracondylar fracture of elbow joi n t
(c) The complication is. Jess likel y in chiluren
(d} Diagnosis is made with certai nty by skiagraphy.
58. Which statements pertaining to green stick fracture is
correct?
(a) Any fracture (#) in child
(b) Is generally incomplete
(c) !t only in rickety children
(d) All of the above.
59. Fat embolism may ensue following:
(a) Fracture of spine and ribs
(h) Fractured fibula
(c) Fracture of skull b,rne
(d) Fracture of long bones only.
60. Which of the following, statement regarding green stick
fracture is correct'?
(a) rracture only occurs in rickety children
(b) Any fracture in a child
(c) Ts generally incomplete.

61. Which of the following is true about fat embolism?


(a} Usually ensues after fracture of the lower limb
(b) Uncommon complication of fracture::
(c\ Sp<>ntaneously reversible process
(d) All of the above.

133
62. 'W"hich is the commonest. fracture in children?
((I) Fractu re clavicle
(b) Supracondylar fractu re
(cl Green Lic:k fracture of lower .:nd ol radi m,
(d) All of the above.

63. Jn Volkmann'sischacmia,su rgeryshould hedonewithin:


(a) 24 hours
(bl 36 how·s
{c) Ihour
(d) 6 hour.

SWERS
l. (h J 2. (a) 3. 4. (<I)
5. (() 6. (d) (d)
7. ( (." ) 8. (11)
9. (a) 10 (a) 11. (d) 12. (l))
13. ( P) . (d)
14. 15. (a) I(,. Ir')
17. (d) 18. (c) 19. (Cl) 2(). ldl
21. (a) 22. (c) 23. (c) 24. None
25. (ti) 26. (d) 27. (rl) 28. (<I)
29. (c) 30. (al 31. (bl 32. (d)
33 (c) 34. (11) 35. 36. IC)
. (c)
37. 38. (a) (rl
39. (b) 40. (, )
·U (c) 42. (d) 43. (cJ 44. (l)
.45. (e) 46. (b) 47. (t!J 48. (ti)
49. (cJ 50. (h) 51. (a) 52. (h)
53. (o) 54. (d) 55. {I") 56 (d)
57. (c) 58. (b) 59. (d) . (cl
60.
61. (ti) 62. (a) 63. (c)

134
II OF THE SHOULDER

l. Duga's testis helpful in


(a) DislocaLion or hip.
(b) Scaphoid fracwre
(c) Fracture neck of femur.
(d) Anterior di. locarion of shoulder J fPi'vfF:R.91
2. Treatment of choice for fracture neck of humerus in a
70 year old male
ta) Analgesic with ann sling:.
(I!) U-Slab
(,) Anhroplasty.
(ti) Open reduction - l nLemal fixal ion. PAL"96
3. Luxatio erect'.\
(a) Tear of Lhc glenoitlal labiu m.
(b) Inferior dislocation or shoulder
(c) Ame1ior di lot:ation of shoulder.
(d) Defect in the humeral head TN'98
4. A patient with recureent dislocation of shoulder pre
sent. to the hospital. The doctor tries to abduct his
a1·m and to extend the elbow and external rotation,
but the patient doesn't allow to do so. This test is
called:
(a) Duga·s test.
(h) Hamil ton·s test.
(c) Callway's test.
(d) Apprehension test. A/IMS 2001
5. Common injury to balJy is
(al Frncn,rc humerus.
(b) Fracture clavicle.
(cl Fracture
(d) Fracture femur. CMC VELL<JRE
135
6. Ideal treatment with fracture neck of humerus in
a lady wiU be
(a) Triangu lar sli11g.
(b) Hemiarth roplasty.
(r) Chest arm bandage
(d) Internal Fixation. UP 2K
7. Banka rt's lesion involves
(a) A nterior aspect or che head of hu mcru. .
(b) Anterior aspect of glenoid Iabrum
(c) Poscerior aspecl of glenoid labrum
(d) Posterior aspect of head of humerus. UP 2K
8. Commonest type of shoulder dislocation;
(a) Subcoracoid.
(b) Subglenoid
(c) Posterior
(d) Subclavicular PGI 87
9. Treatment of fracture clavicle inan infant is best
treated by
(a) Cuff and sling
(b) Figure of 8 ba11dage
(c) Open reduction
(d) Shoulder cast JIPM!:.R 87
10. All arc related lo recurrent shoulder dislocation
except
(a) Hill sachs defect
(b) Bankart Jcsiun
(c) Lax capsule
(d) Rotator cuff injury CMC 2001
11. Fracture of the clavicle are very common injuries. The
most frequent complica tion of this fracture is
(a) Malunion
(b) Delayed union
(r) Non u nion
(d) Nerve injury. KARNATAKA./ 989
12. Io Recurrent Anterior dislocation of shoulder. the move
ments that causes dislocation is
(a) Flexion and internal rolation
(b) Abduclion and external rotaiion
(c) Abduction and internal rotation
(ti) Extension ALL INDIA.1989
13. Recurrent dislocation is most common in the shoulder
joint. Which one of the following is not an important
cause for the same?
(a) Tear of the anterior capsule nf the shoulder
(b) Associated fracmre neck of the hu merus
(c) Tear of the glcnoid labru rn
(<fl Freedom of mobility in the shoulder.
KARNAJ"AK A: 19119
14. Recurrent dislocation is least common in
(a) ShOLllder
(b) Knee
(c) Patella
(d) None [)J:,LHI /994

IS. All of the following statements about dislocation of the


shoulder are true, except
(n) The injury is produced by a fall with the arm fully ab
ducted
(b) The commonest position for the head of the humerus
to move into is L,l e subspi nous
(c) The auxiliary (circumflex humeral) nerve is likely to
be inju red
(d) The easiest way to reduce it is hy simple pressu re with
the patient under general anesthesia with muscle relax
ation. MAN!PJ\L /
994
16. Attitude in subcoracoid dislocation of shoulder includes
(a} Adduction

137
lb) Limb on side of body

138
tc) Abduction
(d) Elevation AN DHRA: /
994
17. All are lruc regarding Clavicular fracture exl:Cpt;
(") May be caused by a fol l on tO the outstrecched ann
<bJ Commonly occurs between tJ1e inserLiom,of t he
coraco
Clavicular and the c:osto Clavicular ligaments
(c) May jeopardize bloo,l supply Lo the overlying skin
(d) Usually reqwres careful reduction.
TAMIL NADU: /992
18. All are true regarding Clavicle except
\a) First bone tn ossify
(b) No treatment required for fracture except rest
(c) Ossifies in membrane
(d) Break al mid point ·1ff,\4S: /993
19. Recurrent dislocation f shoulder occurs becau,c of
(a) Crushed glenoid labrum
(bJ lncomplete labrum
(c) Wcak posterior cap:o;ulc
fr {) Supernddecl sccom.lary infection ,\I/MS: IY 93
20. In Anterior dislocation of' the shoulder the ner ve
involved is
(a) Radial nerve
(b) Circumflex nerve
(c) Ulnar nerve
(d) edian nerve (a)
Anterior
21. Hill-Sachs lesion is ass(1ciated with border of
(a) Recurrent dislotatiun of shoulder head of hu
(b) Recurrent dislocmion of hip merus
(c) Pen.bes' disease
(a) Fracture neck or femu r.
22. Bankart 's lesion is sa in
138
NfJMS:1992

JIPM ER:/ 992

139
(b) Po,terior border of head of hwnern
(c) Anterior glonoid cavity
(d) Posterior glenoid ca\'ity ,.\/JMS:2000

23. True about fracture clavicle is


(a) Jvt1lunion
(b) Most common sit.e is medial 1/3''1 & lateral 'J./3'"
(c) Comminuted fracture
(d) Due to fall on oul.strelched hand AU INDIA: 2000

24. Clavicular fracture is usually treated by


(a) Tract 1cin
( b) Open Reduction & lntcrnal fixation
(c) Figure of eight bandage
(dl Pl ate & Screw fixation. TAMI/, NA/JU · /
<)<)9 In a fracture shaft humerus, which or the
follc>1d ng com plication req uires inuuediate surgery;?
(ti) Compound rracturc
(b) Nerve injury
(r) Brachia( artery occlusion
(d) Comminuted fracture NIMS:2000
26. A 65 years female presents with impacted fractu re
surgical neck of humerus. Treatment of' choice is
(c1) Arthroplasry
(b) Arm-Ches!strapping
(r) Triangular sling
(d) Wail and watch PG/:2000
27. A 22 year old male is admitted with fracture of tl1e
left femu r. Two days later, he becomes mildly
confused. has a respiratory rate of 40 I min and
scattered petechial rash on his u pper torso. Chest
X-ray shows patchy alveolar opacities bilaterally.
His arterial blood gas analysis is abnormal. The
most likely diagnosis is
((I) Cerebral oedema with early nell rogenic pulmonary

139
oedema

139
(b) Pu lmonary thrombo-embol ism
(c) Chest contusion
(d) Fal embolism CSE. / 998
28. The most common complication of clavicle fracture is
(a) I njury to brachia] plex us
(b) Yl.alunuion
(c) Sriffness of shoulder
(d) Non union. MP:198
29. The most common bone fractured during birth
(a) clavicle
(b) Scapula
(c) Radi us
(d) Humenrs MP:/<JW:!
30. Following statement rega1·ding dislocation of shoulder
a1·e true except
(a) Head of hu merus usually dislocates forwa rd from
shou Ider joint
(b) Injury is prod uced hy forced e)((Cm,ion & cxcerna
l rotation of abducced ann
(c) In posterior disloca tion. appearance or shoulder is
not normal
(d) None or the above. NIMHANS: / 997
31. rn treating a fractured clavicle in a 14 month old
child,
the best procedure is
(a) Open reduction
(b) Shoulder cast
(c) Figure - of -eight bruidage
(d) Kirshner pin ANDHRA: / 999
32. The position of arm in anterior dislocation of shoulder
is
(a) By the ,ide
[b) Tn abduccion
140
(c) I n abduction
(d) Lu external rotalion ANOH RA: 1999
33. Anterior dislocation of shoulder is most commonl y
complicated by
(a) Axillary artery injury
(b) Circumflex nerve injury
(c) Recurrent dislocation
(d) Axillary nerve injury PGI: 1997
34. The Rotator cuff is composed of four of the following
muscles except
(a) Tcrcs minor
(b) Supraspinatus
kl lnfraspi natus
(d) Teres m,tjor
(e) Subscapulari, NATIONAL BOARD: / 990
35. Meyer's procedure is a method for treatment of
(a) Recu rrenc shoulder dislocation
(b) Habitual disloca1 ion of patella
(r) Congeni tal dislocation or hip
(cl) Fracture neck of femur PG/:/ 990
36. Fracture of clavicle is commonest at
(a) .Junclion of medi.al 113ni and lateral 2/3"1
(b) Junction of medial 2/3nl and lateral l/3"'
{c) Midpoint
(d) Scapular end Delhi: 199/
37. \Vhich of the following is true of shoulder joint?
(u) Composed of onHy 2 joinL
(h) Anterior posterior gliding of scapula of never occurs
(c) Acromio clavicuJar joim is more important
(d) AJ!ows flexion. rotation and abduction.
.IIPMER: 199/

141
38. The most common complica tion of dislocation of
shoulder joint is
(a) lujury 10 brachia) plexus
(b) lnjury t(l circumHcx nerve
(c) Ru pture of supraspirriatou $ muscle
(d) Rupture of deltoid muscle ON/SSA: ICJ9 J
39. Hill-Sachs lesion in recurrent shoulder dislocation b
(a) Tn_jury to humenil head
(/1) Rupture of tendon of supraspinatus muscle
(r) Avulsion of glenoid labrum
(d) None of' the above. A/IMS . /9CJ2
40. Regard ing Recurrent disloca1ion of the shoulder,
which of the following is false?
(11) A l l traumat ic Ll islocations wi l l be rccun-en1
(b) Recu rren t dislocation resu I ts when tht! capsule is
strippcu, not tom
(c} The humeral head is always within the capsule
(ti) A ll nf t he,c. AN/)/-JRA : 1992
41. Which is true rt,garding shoulder dislocation'!
(a) Posterior d i,locatio1l is oflen over-looked
(hl Pain is severe in amcrior dislocation
(r) Radiography may be mislead ing in pos1<:rior <foloc
tion
(r/) Al l of the above.

42. "Figure or Eight" bandage used common!) in the


fracture of:
{u) Scapul
(b) Humerus
(c) Clavicle
(d) Metacarpals
43. Which is true about shoulder dislocation'?
(a) Anterior dislocation is common than posterior
(b) Fixed medial rotation in po,te1ior d islocation
142
(c) Kocher' s manoeu vre is effective in anterior
dislocation
(t{) All of the ahove.

44. Fracture neck Humerus is common in:


(a) Elderly woman
(bi Young lady
(c) Elderly rnnn
(d) All of these.

ANSWERS
I. (d) 2. (a) 3. (b} 4.(cf)
5. (b) 6. (ll) 7. (h) 8. (11 l
9. (h) 10. (d) 11. (a) 12 (b)
13. (b) u. (b) 15. (d) . (/;)
16
17. (d} 18. (d) 19. (aJ . (/))
20.
21. (n) 22. (c) 23. (a) 24. k)
25. k) 26. (c) 27. {d) 28. (/1)
29. (n) 30. (L' ) 31. (c) 32. (II)
33. (11) 34. (d) 35. (d) 36. \ /))
37. (d) 38. (b) 39. (Ct) 40. (a)
41. (d) 42. (c) 43. (d) 44. ((I)

11119,11!.l_FINOJR ILSn_To_w
_,_ .
UERAIERsM_oA_FN_T
_HWE RE
D
1. The best radiological view for l'racture sc.aphoid is;
(a) AP.
(b} PA.
(c) Lateral.
(d} Oblique. A.l.89
2. Conuuonest fractures in childhood is;
(b) Femur.

143
(b) Distal humerus.
(c) Clavicle.
(d) Radius. BHU.88
3. Tdangular relation of Elbow is maintained in (d)
(a) fracrure ulna. Supra
condy
(b) Ante1ior dislocation of El bow
lar
(c) Posteiior dislocation of Elbow fractu
re
MS.9
1
4. Avas
cular
necro
sis of
bone
is
most
com
mon
in
(a)
Scap
ula.
(b)
Scaph
oid.
(c)
Calca
neus.
(d)
Cervic
al
spine.

JIPM
F..H.
93
5. Pulled
Elbow is;
(a) Disaniculatiou of elbow.
{b) Subluxation of disllil rad io-ulnar joint.
(c) Subluxatiou of proximal radio ulnar joi111.
(d) None of the above !'GI
79. AMC.85
6. Obliq ue view is required to diagnose fracture
of
(a) Capitme.
(b) Scaphoid.
(c) Navicular
H A
M
7. Suspected medial epicondylar fracture of
humerus in a 4 year old child requires:
(a) X-Ray both am1s w i th elbow for
comparison.
(h) X-ray same limb only
(c) Examination u nder general anaesthesia.
(rl) POP in full nexed position.
8. A young adult presenting with oblique,
wsplaccd frnc tu1·e olecranon treatment of
choice;
(a) Plaster cast.

144
(b) Percutaneou wiring.
(c) Tension band wiring.
( cf) Removal of displaced piece wi th 1riceps repair.
A /IMS.06
9. In coUes fractu re not seen is
(a) Proxima l impaction.
(b) Lateral rotation.
(c) Dorsal angulat ion.
(d) Medial rotation
All MS.97
10. Carpal bone which fracture commonly:
(a) Scaphoid
(b) Lu nate
(c) H,rn,mate
(d) Pisciforrn NIMHANS 87, KERAU 87
11. The cause of gun stock deformity is
(a) Supracondylar fractu re
(b) Fracture both bones forearm
(c) Fractu re surgical head of bumcrns
(d) Fracture fibula PG/ 86
12. Osteotoruy done for mal united supracondylar
fracture is
(a) French
(b) Shanz's
(c) Mc Murry's
(d) Mc Alister J&K 2001
13. Which one of the following statements is not correct
regarding fracture of the scaphoid?
(a) Tt is the most commonly fractured carpal bone
(b) Per.;istent tenderness in the anatomical snuffbox is
highl y suggestive of frnctwe
(c) Immediate X-ray of baud may aot reveal fractu re line
(d) Mal union i s a frequent compl ication UPSC 200/

145
14. The most common elbow injury in children is
(a) Extension type of supracondylar fracture of l1umcrus
(b) Dislocation of elbow
(c) Fractu re lateral condyle of humerus
(d) Fractu re medial cpjcondyle of' humcru,
KER,\l,A / 9/iX
15. In supra condyla r fractu re of humerus, the distal
segment is often di placcd to;
(a) An
teriorly (b)
Laterally (c)
Posteriorl y
(d) Medially Kerala. / 988
16. Long term administ ration of ACTH produces
(a) Ostcopetrosis
(b) Ostcochondritis
(c) Ostcosarcoma
(d) Ostcoporosi,
(e) Calcification of c:utilage. Kem/a, / 988
17. Main risk infracture Scaphoid is
(a) Non union
(b) Ma lunion
(c) Delayed ,mion
(d) Avascular nccro,i
Ie) Carpal tunnel synd.rome AMC. 1989
18. Fracture lateral condyle of tile humerus is a common
injury inchild ren. Which oneof the following isthe
most ideal treatment for a displaced fracture lateral
condylc of the humerus in a 7-year-old child?
(a) Open reduction and plaster immobilizatioo
(b) Closed reduction and plaster immobilization
(c) Open reduction and ioternal fixation
(d) Excision of the fractured fragment
KARNATAKJt 1989
146
19. Fall on outstretched hand may lead to fracture of
(a) Shoulder
(b) Clavicle
(c) Scaphoid
ldJ Coronoid process DELHI: ICJ89
20. The most common deformity seen in supracondylar
fractun of humerus is
Ca) lnabiliLy to supinate and pronate
(h) Varus
(c) Valgus
( cf) None

21. Ta,·dy ulnar nerve palsy occurs as a delay sequela of


(a) Supracondylar frncrure of humenL
(b) Pos1erior dislocation of elbow
(C') Fractu re of lateral condylc of humeru, in child1en
(d} Fractu re of olecrrumn.

22. Excision of' head of rad ius in a child should not be


done
because
(a) I t produces instabi lity of elbow joint
(b) lt lead, to secondar·Ostco m'lhritis of elbow
(t') Ir cause.s ublu xatiou of inferior rad io UlnILI' joi n t.

23. Complication of fracture !icaphoid is


(a) Avuscular necrosis of distal part
(/J) lnjury·to radial artery
(c) lnjury to radial nerve
(d) Avascular necrosis of proxi mal part. KERA/ A: 199../

24. Avascula r necrosis of bone is most commonly seen in


(a) Calcaneus
(b) Cervical Spine
(c) Scaphoid
(d) Scapula .lf PMER: / 993

147
25. Medial epicondyle fracture results in injury to
nerve.
(o) R adiaJ
(b) Median
(c) Ulnar
(d) Axillary NB: / 99J
26. increased intercondylar distance is seen in fracture of
au except
(a) Olecranon
(b) Media l epicondyle
(c) Lateral epicondyle
(d) Lateral cortdyle DF.LH/: / 993
27. The most commou injury in a 7 years old child due to
fall on outstretched hand is
(a) Dislocation (lf shoulder
{b) Colics' fracture
(c) Fracture of clavicle
(d) Supracondylar Fr icture of humcrns ANDI.IRA: 1993
28. Rarton 's fracture of the wrist
(a) l.nvolves radio carpal subluxation
(h) Is a severe form of a Colles·fracmre
(c) Is often treated by open reduction and internal fixation
(d) All of lhe above. TAMIL NADU: 1992

29. The bony triangle is maintained in


(a) Supracondylar fracture of humen1s
(b) Posterior dislocat ion or elbow
(c) lmcrcondylar fracture of hwnerus
(cf) Lateral condylar fracture of humeru.
ALL JNDJA: JYY:!
30. Tardy ulnar nerve palsy occur as a delayed seq uele of
(a) Supracondylar fracture of humerus
(b} Posterior dislocacion of elbow
(c} Fracture of lateral condyle of humerus in children
(b) Fracture of Olecr.anon PGI: 1992

148
31. The treatment of choice of fracture of radius and ulna
in a an adult is
(n) Plasier for 4 weeh
(bJ Closed reduction and calipers
(c) Only plates
(d) Ku n tscher nails IJ/iLHJ: 1992
32. Jn Mont.eggia fracture, which is true abou t ulna
r fractw·e and bead of rad ius
(a) Uoth ulnar fractu re and head of rat.liu;, b Ji,placcd
posteriorly
(b) Both ulnar fracture and head of radius is displaced
antcriorly
(c) Ulnar fracwres is posteriorly and head of radiu 1,
displaced anteriorly
(d) Ulna r fracture is anteriorly and head or rad i11 is
displaced postt'r iorly. JIPMER I 992
33. Relation between3 bony point in elbow is reversed by
((I) Fracture lateral condyle r>r humerus
(b) Fractu re medial conclyle of hu merus
re) Posterior dislocation of elhow
(cf) Supracondylar fracture of hu merus ;VIMS: 2000
34. Not a complication of Colles' fracture
(a) Stiffness of wrist
(b) Stiffness or
shoulde,
(c) Carpal tun nel syndrome
(d) Wrist drop ./IPM/::J, ?: 2000
35. Commonest complication of Colles' fracture is
(a) Noouuion
(b) Mal uaion
(c) Vascular inJury
(d) Sudeck 's osteodysrrophy KARNATAKA: 2000

149
36. Which carpal bone fracture cause5 median nerve
involvement?
(a) Scaphoid
(b) I.u nate
(c) Trape7ium
(d) Trnpernid NIMS: 2000
37. The complication of CoUes' fracture is
(a) Radial nerve palsy
(b) Stiffnc,s of wrist joint
(c) Ulnar nerve palsy
(dl None of ll1e above NATIONAL BOARD. 2000
38. The most common type of' supracondylar fracture iJ.
(a} Neutral
(h} Flt,xion
(c) Extension
\d) Lateral TAMIL NADU: 1993
39. lnternaJ fixation is proba bl) needed in all of the
following except
(a} Fracture condyle or humerus
(I)) Fracture neck of fernur
(c) Fracture nr Olecrfnon
(d) Fracwrc of caphoid UPSC: 1993
40. CubitLL varus is most commonly seen in
(a) Rickets
(b) Post inflammatory epiphyseal damage
(c) Frncture later) condyle humcnis
(d) Ma l united supracondylar fracture ALL IND!!\: 1994
41. Treatment of Smith's fractu re is
(a) Above elbow cast applied n extension
(b) Colles ·pl aster
(c) Dorsal spl.intage
(d) lmemaJ fixation ALL !NOIA: /9()4

150
42. A 12-year-old child presents with tingling sensation
and numbness in the little linger and giveshistory of
fracture in lhe elbow region 4 years back. The
probable fracture is
(a) Laiernl condylc fractLtre humeru:;
(b) Injury to ulnar nerve
(r) Supracond ylar fracture humerus
(d) Dislocation of elbow

43. Three point relationship is maintained in the


follo\\;ng (al Dislocation of elbow JOilll
(b) Supracondylar fracture of hu merus
(c) Fracture medical condylc frac1ure of hu menJs
(d) Lateral condyle fract u re of humerus.

44. A lO-yea1·-0Jd boy presented with pain and massive


swelling left thigh. On examination reveals diaphyse.al.
lesion and soft tissue swelling. The likely diagnosis is
(a) Oteosarcoma
(b) Ewing's sarcoma
(<:) Osteoclastomu
(d) Aneurysmal bone cyst AIIMS:NO\l:2000
45. Pulled elbow is
(a) A sprain of extensor tendons
(b) Dislocation of head of radius
(c) fracture of lateral condyle of hu merus
(d) Dislocation at elbow PG!: / 994
46. Which is not a feature of pseudoachond roplasia?
(a) Shor1 limbs in comparison to trunk
(b} Kyphosis
(c) Nom1a l size skull
(d) None of the above PG/ : 1994
47. Galeavj fracture is fracture of
(a) Upper end of ulna
(b) Lower end of ulna
151
(c) Upper end of radil!ls
(d) Lower end of radius ANDH RA : /
998
48. A pa t ient reported with a history of fall 011 a n
out tretched hand, c.omplains of pain in the anatomical
snuffbox and clinically no deformities visi ble. The
diagnosis is
(a) Colles· fractu re
(/J) Luna le d islocatioo
(c) Barton's fracture
(d) Scaphoid fracture NIMS: 1998
49. The most common nerve involvement is dislocation of
Lunate is
(11) Median nerve
(b) Anterior interosseus
(c} Posterior interosseus
Cd) Median nerve. UP·l 9W:
5(). Galeazzi fractu re is
(a) Supracondylar fracture of the h.u mcrus
(b) Fracture of the distal rad ius with inferior rnuin ulnar
joint dislocation
(tJ Fracture of radius in 1.he proxi mal site and dislocation
or 1he
elbow
(d) Fracture of the rad ia l head. KARNATAKJ\: /997
51. If an adolescent boy falls on a out-stretched ha nd,
the
most common bone to be injured is
(a) Fracture of lower end of rad ius
(b) Fracture of both bones of forearm
Cc) Scaphoid fracture
(d) Supracondylar fracture of hu merus AN J)J-1/?A: /999
52. Posterior intcrosscous nerve is injured in
(a) Pos1e1ior di$location of elbow
(h) Monceggia fracture dislocation
152
(c) Reversed Monteggia fracmre di locatio11
(d) Supracondylar fracture of humerus ANDHRA ·/999
53. The most common cause of Volkman n 's ischacmic
cootracture (V.I.C) in a child is
(a) lntercondylar rracture or hu men1s
(b) Fracture both bone of forearm
(c) Fracture lateral condyle of humeni
(d) Supracondylar fn1cture of humcrns :\/IMS: /99')
54. The most common complicaliou of supracondylar
fracture is
(o) Genu valgum
(b) Blood vessel injury
((') Volkmann's ichac111ic con tractun:
(d) Malu 11ion with gun stock deformity f'(j/: /997

55. G1mstock det'onnity is due to


(a) Fractu re ofI" meLacarpal bone
(b) Fracture of lower end or radius
(c) Supracondylar fracture of humcrns
(d) Lateral condylar fracture of h umcrns
TAMIL NAOU : /997

56. Rupture of extensor pollicis longus occurs four weeks


after
(a) Collcs" fracture
(b) Radial styloid fracnirc
(c) Smith's fracture
(d) Scaphoid fracture
(e) All of these. Bl/ JAR: / 990

57. The most common cause of nonunion is


(a) Periosteal elevation
(b) IJnematoma between fragments
(c) Callu formation
(d) Diminished blood supply
(e) All of the foregoing.
153
58. Tardy ulnar neuritis may be due to
(a) Advanced osteo arth ri tis of elbow
(h) Cubitus valgus deformity
(c) Both of the above
(d) None of the above. 8/HAR. l 'J'JO
59. The true statement regarding supracondylar fracture
of the Humerus in children
(a) Admission to hospital is essential following reduction
(b) It i s due to a fall on the point of the elbow
(c) iirequ res open reduction
(d) lt is usually C()mpound KERA LA:/ 990
60. A CoUes fracture is
(a) Common in adolescence
{b) A fracture abolll the ankle joi n t
(c) Comm<m in eltlerly women
(d) A fraclUre of head of the radius
NA 1'1.0NAL BOARD: J 990
61. Which of the following is known for Non u nion in
children, if lest untrea ted?
(a) Fracture shaft of hu merus
(b) Fracture shaft of femur
Cc) Fracture distal l/3ni of t ibia
(d) Fractu re lateral condy le of humems
(e) Supracondylar fracture of humerus
NA 1'/0NAL BOARD: J9'JO
62. The following fractures areknown for Non-union except
(a) Fractu re of lower half of ti bia
(b) Fracture of neck of femur
(c) Fracture of scaphoid
(d) Frncture of patella
(e) Supraconclylar fracture of bu m<::rus
NATIONAL 80ARD:J9'JO

154
63. Fraclure oflateral coodyle of humerus Set)n in age grou
p
of
(a) 2-J years
(b) J-5 year.;
(c) S-15 years
(d) 15-25 years
(e) 35-45 yers !'GI:1990
64. The most important cause of Nonunion of fracture of'
humeral shaft is
(n) Comminutcd fracture
(b) Compound (Open) fracture
(c) Overrid ing uf fradurc ends
(d) Distraction at fracture ,ite
(e) Operative reduction OR!SSJ\: !990
65. About djsplacemcnt of'distal fra!!,mentin Colles fr.ictu rc,
true is
(a) An teriorly and mediall y
(b) Posteriory and laterally
(c) Anteriorly and laterall y
(d) Po,teriorl y and med ially. UPSC: 1990
66. ln children the following fracture often require open
reduction
(a) Femoral condylar fracture
(b) Suprac.ondylar racrure of the hu meru,
(c) Fracture of the hoth bones of forearm
(d) Fracture of laternl condyle of the h urncru.
AU IND!A- 1991
67. Tardy Ulnar nerve palsy is caused by
(a) Fractu re lateral epieondyle of humerus
(/J) Fracture medial cpicondyle of humerus
(r) Elbow dislocation
(d) Supra condyl ar Fracture of hu merus I\IIMS:/ 99!

155
68. Open red uction i:n children is rcttuired for
(a) Fracture both hone'i or forea rm
(b) Femoral condyles
Cc) Lateral humeral condyle
(d) Distal ti bial epiphysis A/IMS:/9Y /
69. The basic principle in the lreatmenl of fnictures of both
bones of' the forearm is to
(a} Reduce 1mgulation of radi us ru1d ul na
(b) Restore the normal relationshi p of radiu and ul na
(c) lmmobilizc I.he elbow only
(l[) Prevent over ridi ng of fragments
(e) All of the above. Blf lAI?:/ 99
/
70. 'Dinner fork deformity' is present in case of
(a) Smith 's fracwre
(h) Studen t' elbow
<c) Colle · fraccurc
(d} A 11 of these. HJHAR:/ 99!
71. Which of U1e following is commonly seen in Colles'
fracture'?
(a) Non union
(b) Delayed union
(c) Malu11ion
(d} Rapid union DELHI: 1991
72. The commonly inju1·ed carpal bone next to scaphoid is
(a) Trapeiu m
(b) Trapezoid
(f.') Lunate
(d) Capitate
(e) Hamate NATIONAL BOARD: / 99/
73. In Scaphoid fracture, important views are all except
(a) AP
(b) Lateral

156
(c) Obliq ue
(a) Cone !'GI: /99/
74. Ifhead of the radius is removed, it will result in
(a) Lengthening <>f limb
(I>) Valgus deformiLy
(c) Varus deforrniLy
(dJ No deformity PG/ : /
991
75. In supra condylar fracture of humerus, the ner\'e
most commonJy injured is
(a) Radial nerve
\b) Ulnar nerve
(c) Median nerve
Cd) AuxiJiary nerve UPSC-1991
76. Moneteggia fracture is fracture of
(a) Lower 1 /•..i or Rad,us
(h) Upper 'I,"' of Radius
(c) Lower 1/,..i of Ulna
(d) Upper 11,..i of Ul na UPSC: / 99/
77. The most common cause for tardy Ulnar palsy is
(o) rractu re of lateral condyle in child bood
(b) Supra condylar rract1.1re
(c) Olecranon fracrure
(d) Mon1eggia's fracture Al/MS : / 992
78. Which fracture requires open red uction in children?
(a) Fracture of boLh hones of foreunn
(b) Epiphyseal separntion of tibia
(r} Tntercondylar fracture of femur
(d) Lateral condyle fracture of hunieru, All1\IIS: / 992

79. Following are the c-0mmon sites of Ava cular


necrosis. EXCEPT:
(a) Proximal half of scaphoid
(b) The body of talus
157
(c) Pacella
(d) Head of the femu r.
80. \'Vhich of the following. statemcnt(s) is I are true?
(a) Oedema & tenderne.. s over 1he anatomica l snuff box is
the paihognomonic femu re;. of Fracture of 1hcscapho1d
(b) Norn1ally Lhc rad ial stylo1d is 112" lower thai1 the u
lnar
(c) Di n ner fork defon11i1y i charac1er i stic of Col
le's Frnc:1t1re
(d) A 11 of the above.

81. Strike the false statement (s):


(a) Reversed Colic's i, called Smi1h's Frac1ure
(b} I f the lower limb is externally rotated. i t is moM likely
d ue to Frac1u rc neck of fermtr
(c) If the thigh assumes nexion. :1dduc1ion and
internal rotation following a history of severe injury. a
postenor d islocation is the likdy cause.
(d) None of these.

82. \J\,l1ich complica tion may arL c aJter supra-


condylar fracture'?
(a) Median nerve injur,
( /J ) Damage to brai nch ild artery
(c) Cuhitus varus
rd) All of the above.
83. Which tendon gets involved in Colle's fracture?
(a) Abd uctor pollicis lo11gus
(b) Exu:ns1)r pollicis brevis
(c) Ex tensor pollicis longus
(d) All the above.

I ANSWERS l
1. 2. 15
(d)
3. (d)
(r.) 8 7. (0)
S.(c) 6. (bJ
4. (b)
8.
(d)
9. (d} 10. (a) 11. (a) 12. (a)
13. (d) 14. (a) 15. ( Cl 16. ti/)
17. (d) 18. (d) 19. (c) 20. (/))
21. (c} 22. (c) 2J. (d) 24. It)
25. (c) 26. (a) 27. (d) 28 (a)
29. (a) 30. (c) 31. (cl . //))
32
33. (c) 34. (d) 35. (h} . (/))
36.
37. (/1) 38. (cl 39. Cd) 40. <cl)
41. (c) 42. (a) 43 (b) 44. (b)
45. (I>) 46. (cl .47. Cd) 48. (d)
49. (a) 50. (b) 51 (C) 52. (h)
53. (d) 54. ( tl) . (Cl
55. 56. Ill)
57. (d) 58. (/>) 59. (a) 60. ( c )
61. (d) 62. (el 63 (b) 64. rdJ
65. (/)) 66. (d) .67. (a} 68. (c)
69. ( /)) 70. (c) 71 (cl 72. r el
73. (d) 74. (/,) . (c)
75. 76. le)
77. (a) 78. (c{) 79. (CJ 80. r d)
81. (d) 82. (d) 83. (c)

mI INJURIES OF THE HAND


l. Bennett's fracture is fracture dislocation of base of
......metaca rpal:
(a) 4th.
(b) 3'11_
(r) 2'J.
(d) ) "· UP.88
2. Mallet finger is
(a) Avulsion fracture or extensor tendon of distal
phalanx
(b) Fracture of distal phalanx
150
(c) Fracture of middle phal:HlX
(d) Fracture of proxima l phalanx
UPSC 85.88. Kera/a 87
3. The term Bennett's fracture is IL Cd lo describe
(a) Fracture-dislocation of metacarpophalangeal joint of
thumb
(b) lnterphalangea l fracture dislocation of thu mb
(c} An terior marginal fracture of distal end of radius
(d) Fracture dislocatiou of trapezometacarpal joi n!.
Kamawka; ICJ89

4. Which of the foUowing Scaphoid fraclure is most prone


to develop Avascular necrosis?
(a) Fracture of waist of scaphoid
(IJ) Fracture of tu bercle
{<-) Fracture of distal pole
(d) All of the above. KARN A TAKI\: / 992
5. A Bennett'sfracture isdifficult to maintain in a red uced
position mainly because of the pull of the
(a) Flexor pollicis longus
(b) Flexor pollici, brevis
(c) Extensor pollicis brevis
(d) Abd uctor poll icis longus t>GJ. 1992
6. Tardy ulnar nerve palsy occur as a delayed sequele of
(a) Supracondyhrr fracture or humerns
(/J) Posrc1ior di,location of elbow
(c) Fracrure or lateral condyle of humerus in chiklrcn
(d) Fract ure of Olecranon PC!: 1992
7. A middle aged male fell Crom a height presents with
shortening of leg, internal rotation and tenderness in
Scarpa's triangle. Diagnosis is
(a) Hip Di$location
(b) Fracture neck of femur

160
(h) Fracmre shaft or femur
(b) Fractu re of acetabulum UPSC: / 993
8. Position of the lower limb i11 1>osterior dislocation of
hip is
(a) Flexion, Abduction anti external rot ation
(b) Flexion. Adduct ion nnd external rotation
(c) Flcxion, Abduction nnd internal rotation
(({) Flcxion. Adduction ,ind internal rotation
A/IMS: .VOV-2000

9. A Bcnnet's fracture is difficult to maintain in reduced


position because of the pull of'
(a) Extensor poll icts longus
(11) Extensor pollicis brcvis
(c:)Ahducmr pollicis longus
(d} Abductor pollicis brevjs KARNATAKA . / 999

10. Avulsion of extensor tendon gives rise to


(a) Mallet finger
(b) Dupuytren' s comracm1c
(c) Trigger finger
\d) Swan neck deformi ty ANDHRA: 199
U. Lower branch of brachia] plexus injury leads to
(a) Erb's palsy
(b) Klumpkc"s palsy
(l") Bell"s palsy
(d) Wrist drop ANDHRJ\:IY 97
U. Galeazzi's fracture is
(a) Lower 1/3'" fracture ulna with radioulnar dblocmion
(b) Upper lf3'd fracLure ulna wHh inferior radioul nar
dislocaLion
(c) Lower l/3nl fracture radius with rad 1.oulnar
dislocation
(d) Upper fracLu re ulna with supet ior radioul nar
1/3"'
161
dislocation PG!: I
997

162
13. Middle palmar space rods distally
(a) Along U1e digital sheaths
(b) Into the nexor tendon sheaths
(c) Into tbe web space
(d) By mixing with the superficial pal mer ,pace
NATIONAL BOARD: /<I<;/

I ANSWERS I
1. (II)
2. 3. 4. (a)
5. (d) (a) (d) 8. (d)
9. 6. (c) 7. (aJ
(c) 12. ( c)
10. (a} 11. (h)
13.
{c)

1111. _ --J_UG

s_o_F_T_A_..:_HIP

ANo ,
1. Commonest dislocation of the h.ipis
(a) Posterior.
(b) Anterior.
(c) Central.
(,{) None. TN 1$9
2. The following is true in the treatment of posterior
dislocation:
(u) Closed reduction under anaesthesia.
(b) Open reduction.
(c) Skeleial traction.
(d) Soft tissue PG!.'JO
3. Flex.ion, adduction and internal rotation is
characteristic posture in
(a) Anterior dislocation of hip joi n.
(b) Posterior dislocation of hip jo i nt.
(c) Fracture of femoral head.
(d) Fracture shaft of femu r. .l!PMF.R.95
4. Treatmen t of choice for old non-united fracture of
shaft of femur
(a) Compression platiJ1g.
(b) Bone grafti ng.
(c) Nailing.
(d) Compression plating with bone grafti ng. A /IMS .CJ4
5. Tn 65year old male with history ofl'racture neck
of1'emur 6 weeks old, treatment of choice
(n) SP nailing.
(b.) Mc Murray 's o,teotomy.
(c) 1-lcmianbroplaty.
(di None. A/IMS.94
6. In the case of 65 year old person with fracture neck
of femur the treatment of choice is
(c,) Closed reduction .
(b) Closed reduction with internal fixation
(c) Open reduction.
(d) Replacement of head and neck of the fomur wnh u
prosthesis.

UPSC
7. Commonest complica tiuu uf extra capsular ft·acture
of neck of femur is:
(a) Non union
Cl>) lschcmic necrosis
(c) Malunjon
(d) Pulmonary complications A.I. R8
8. Fracture femur in infants is best treated by:
(t1) Open reduction
(b) Closed reduction
(c) rM Nai ling

163
(d) Gallows's splinting PD/ 88

164
9. The attitude of limh in traumatic disloi:ation of hipjoinl
is
(a) Flexion,adduction. external rotat ion
(h) Flex1on.adducuon. internal rotation
(c) Flexioll. adducLion. and cxtcmal rotation
(d) Flexion and adduct ion onl y Delhi: / <)R8
1O. Flexio11,abduction and external rotation at hipjoint with
limb length discrepancy is seen in
(a) Fracture neck of remur
(h) Amerior dislocation of h.i p
(c) Poterior d islocaLi m1 of hip
(tf) None ON/SSA_ 1988
11. Fractured neck of femur is associated with all except
(a) Causes shortening of the: J1,;g
(b) Causes internal rotation of the leg
11·) May be pathologic::il
(d) May be Lrealed witb hcmi arthroplasty .
TAMIL NA{)U·/ 988
12. The trca1ruent of choke for a 4 week old Femoral
neck fracture in a 55 years -0ld man is
(a) Open reduction and inicmal fixation
(b) Mc Murray·, Oteotomy
(c) Hem i replacement arthroplasty
(d) Total hip replacement Al.I. INDJ,\.1989
13. Which of the following fractures would best be lrcatcd
by Open reduction?
(a) Fracture of the femoral haft of the chill.I
(bl Collies·fracture
(c·) Displiiced fracture of the femoral neck
(d) Fracture of humera l shaft. AMU .- /
989
14. Fracture of femur at tl1e level of isthmus is best treated
by
(a) lntramedullary n ail fixation
(b) Plate and screws
(c) Closed method
(d) External fixation ANDHRA. 19/N
15. Nonunion is a very common complication of intracap
sular fractures of the 11eck of femur. Which of the
follow ing is not a very important cause for the same?
(a) Inadequate immobilizalion
(b) Inadeq uate blood supply
(c) lnhibi tory effect of synovial fluid
(d) Stress,ll fractu re ite due to muscle spasm
KARNATt\KA:IW?9
16. Traumatic dislocation of hip is characterized by
(a) Adduction internal rotation deformity
(b) Abduction external rotation deformi ty
(c) Adduct ion external rotation deform i ty
(d) Abduction interna l rotation deformi ty
NATTONAL BOARD. 19/N
17. Fractw·e shaft of femur in children of less than 2
years old is treated by
{a) Open reduction
(b} External lix Lion
(c} Gallows's traction
(d) Closed reduction ANDHRA.IY89
18. Mc Mu rray's ostco tom y is based on the following
principle
(a) Biological
(b) Bio mechanic,ll
(c:) J3io technical
(d) Mechanical ANDHRA 1994
19. A patient with hip inadduction and medial rotation and
is unable to move .Pt-obable diagnosis is
(u) Posterior dislocation head of femur
(/J) Fracture shaft o femur

165
(c) Fracture neck of femur
(d) Sciatica ANDHRA / 9Y4
20. Trochan teric fracture of femur is best treated by
(a) Dynamic hip screw
(b) Inlay Plates
(c) Plaster in abduction
(d) Plaster i n ah<luction and internal rotation. PG! 199.1
21. In fracture of femur popliteal artery is commonly
damaged by
((I) Pmximal fragment
(b) Oistal fragment
(c) Muscle haematoma
(d) Tissue swelling PG/ f Y<)J
22. A fracture neck femur in a child is best treated by
(a) Spica in abduction
(b) Spica in abduction + internal rotation
(c) Masterly inactivi ty
ld) Open red uction and internal fixation. PG! 1993
23. The most common complication of' Transcervical
fracture of Femur is
(a) Avascular necrosis
(b) Malunion
(C') Non union
(d) None TAMIL NAOU 1992.
24. Behcet's syndrome is commonest in
(a) Ankle
(h) Wrist
(c) K nee
(d) Hip 'KARNATAKA: 1992
25. Characteristic features of the acute compartment
syndrome in the lower leg include all 01' the
following except;
(a) Acute pai n on employing the stretch test
(b) Normal pulses
(c) Non11al sensation distally
(d) Venous occlusion.
26. \.Vhich ismost appropriate treatment for the femur
with nonunion more than 3 weeks'?
(a) Tmemal fixation
(h) Rone grafting with intemal fixation
(c) External fixation
(d) Prosthesis Al!MS:1994
27. In which one of the following femoral fractures is
Avascular necrosis common?
(a) Pcrtrochanrcric
(b) Transccrvical
(c) Snb-Trochanteric
(d) Shaft of femu r CSE : 2000
28. In the case of a 70 year old lady with i.nlra capsula r
fracture or the neck of femur,the ideal treaf ment
would be
(a) Cloed traction
(b) Hemiarthropla ty
(r) Internal fixation with na.i l
(d) Interna l fixaLion with on.i i and plate CSE : 2000
29. Late complicafion of Aceta bular fracture with
dislocafion or hip inclucles
(a) Prone to trauma
(b) Recurrent deislocation
(c) Osteoarthritis
(tf) Stiff hip UP : 1999

30. In u pper one third femor al shaft fractu re, the


displacement of proximal segn1ent is
(a) Flex.ion.abduction and external rotation
(b) Hexion,abduction and external rotation
(c) Flexion, abduc1ion and internal rotation
(d) Flexion . abduction and imcmal rotation CSE : 1999
31. ln anterior disloca tfon of hip, U1e posture of lower
limb
will be
(u) Abduction. externally rotated and extension
(b) Abd uction. external ly rotated and tlexion
(cl Abducted externally rotated and tlcxion
(d) Adducted, internal ly rotated and nexion
DHLJJJ . 1999
32. The femur is fractured at birti1at
(a) Upper thi rd of shaft
(b) Mic.ldle third of shafl
(c) L-owr thi rd of shaft
(d) Neck region Rajc1.<tha11: 1993
33. 3 days old intracapsular fracture neck of femur in a
50 yea r old patient is treated by
!cl} POP hip spika
(b) Total hip replacement
(c) Cortical screw fixation
(d) Hemi replacement arthroplasty UP : 2000
34. Avascular necrosis of head of femur occurs commonly
at
(a) Transccrvica l regmn
(b) Trochanteric region
(c) Subcapital regioa
(d) Subchondral rcgi<>n NATIONAL BOARD : 2000
35. Pau vel's angle is
(a) Neck shaft angle of femur
(b) The difference between neck shaft angle bet ween two
femu rs of a patient
(c) Fonncd by joining a line extended from frat:ture line
of femu r neck LO an arbitrary l i ne depicting t he
horizontal plane

168
(cf) None of the above. RAJASTHAN: 1994

169
36. Garden's classification is applicable to
(a) Inlertrochante1ic fracttrre
(b) Fracture neck of femur
(c) Epiphyseal separation
(d) Posterior dislocation or hip UP: / 99.J
37. 65-year-old lady falls from height. On examination the
leg is extended and externally rotated. Diagnosis is
(a) Fracture of acetabulu m
(b) lntcrtrochanteric fracture
(c) Neck femur fracture
(If) Posterior dislocat ion of hip

38. In per rectal examination, femoral headis palpable in


(a) Anterior dislocati on of hip
(b) Posterior dislocai ion of hip
(c) Central dislocmic)n of hip
(cl) Lateral disl ocatio11 of hip ANDHRJ\: / 998
39. Prosthetic replacement of remoraJ bead is ind icated for
one of the following sites of fractures.
(a) lntcr-rrochamcric fracture of femoral neck
(b) Subcapital fracrnre neck femur
(c) Transu·ochantcric fracrure femur
(d) Basal fracrnrc of femoral neck BIHAR: / 998
40. Femoral neck fracture of three weeks old in an youn,::
adult should be best treated one of the following
(a) Total hip replacement
(b) Reduction of fracture and femoral ostcmom)' witli head
(c) Prosthetic replacement of femoral head
(cf) Reduction of fracture and multiple pin or screw
fixation
(e) Upper femoral displacemem osteotomy.
BIHAR:1998
41. lntramcdulJary fixation is ideal in a case of fracture of
shaft of femur when there i.s
(a) A tTansverse fracture
(b) A compound fracture
(c) Soft 1issue interposiiion between the frac1urcd end
(b) Such a fracture in a child. CSE: 1998

42. Features of fracture neck of femur includes


(a) Flexion 31 hip and lateral rotation
(b) Flexion at hip abduction
(,·) Sho11eni ng and lateral rotation
(d) Shonening and tlexion MAN/ PAL : 1998
43. A 60-year-old man fell in ba throom and was una ble
to stand on right buttock region echymosis with external
rntation of the led and lateral border of foot touching
the bed. The most probable diagnosis is
(a) Extra capsular fracture neck of femur
(b) A nteri or dislocation of hi p
(C') Intra capsular fracture neck of femur
(d) Posterior dislocation of hip UP:/ 998

44. Prosthesis at bead of femur applied in


(a) 40 years young male wilh # head of femur
(b) 40 year young male with # neck nf femur
(c) 40 year young male with posterior dislocat ion or hip
(d) 65 years old male wilh non united fracture neck of femu r
UP:/ 998
45. Telescopic ll-st is useful to diagnose
(a) Perthe's disease
(b) lntracapsular fracture neck of femur
(c) Maluni ted Trochanteric fracture
(d) Aukylosis of hip joint

46. Dashboard injury resul ts in


(a) Anterior dislo,:ation of hi p
(b) Posterior dislocat ion of hip
(c) Central dislocation of hip
(d) Fracture neck femu r ANDHRA: 1997

170
47. Match list J with List Il select the correct answer.
List l Lhit 11
(Sites of Fracture Fem ur) (Methods of treatmen t)
(a) Intracapsular neck fracture l. Thomson Prosthesis
(b) Extra carsular subtro
chameric Fracture 2. Conservative treatment
(c) Shaft fracture in children 3. lntcmal Gxation u,ing
screw and Place
(d) Supracondylar fracture 4.Stein man pin,
A B c D
A 2 4 J
B 2 I 3 4
c 2 3 4 I
D 3 2 4
48. The most common com plication of extra capsula r
fracture of neck of femur is
(a) Avascular necrosis
(b) Nonun ion
(c) MaJun ion
(d) Myositis ossificans DELH I: /
9Y7
49. A 3year old chlld presents Wit h fracture of femoral
shafl had immobilized on tr.action constantly for two
mont h. .
The next step of management is
(a) Hip Spica and if necessary nternal fixation
(b) Gallow traction for 2 months
(c) Open reduction aud Kuntschcr"s lover leaf intramc
dulla ry nailing or plating
(d) Traction by Thomas splint BIHAR:1999
50. A 65yea r old pa tient who presents 10days after fractu
re neck femur is best managed by
(a) Tnternal fixation
(b) Replacement anhroplasry
(cl Mc Murray's procedure
(d) Traction for 6 weeks. Ill/MS: / 99fi
51. Concemjng fractures of Lhe neck of the femur which
statement is considered now to be incorrect?
(a) Tt is common ill elderly women
(h) Tt can occur in young adults due 10 fatigue
(c) Tt can occur in young adul ts following severe
violence applied in th long axis of the remur
(d) The bone fractw-e,:- in an dderly woman because she
frills. 1MU. / 996
52. The most common complica tion of intraca psula r
fracture neck of femur is
(a) Mal union
(b} Osteoarthritis
(c) Non-Union
(d} Shortening .1/PMER: / 990
53. All of the following names arc associated with tests I
op· erations around the nip joint except
(a) Bryam
(b) Shenton
(c) Mc MLlll"ay
(d) Salter
(e) Nelton NATIONAL BOAR.D:1990
54. Flexfon, abduct ion and external rotat ion wit h limb
length discrepancy is seen in
(a) Posterior dislocation of hip
(11) Central dislocation of hip
(c) Anterior dislocation of h ip
(d) Fracture neck of femur OR/SSA: 1990
55. Prosthetic replacement of femoral bead is usually indi·
cated ror
(a) Fresh intracapsular fractare head of femu r in old pa
Liem,
(b) Fresh intracapsular frcturc of femoral neck in a
young
adult
(c) Unreduced poslerior dislocation of hip
(b) Untreated femoral neck fracture in a patient over 65
ytars
(/)) Pathological femoral neck fracture due lo
secondaries.
rGl:1990
56. Vascular sin of Narath is noticed in
(u) Fraclure neck of femur
lb) Penhes disease
(r) Poslerior dislocation of hip
(d) All nf the ahove. NATfONAL BOARD: /'-)9 /
57. Inspondylarth riliclis all arc true except
(r1J 50% of seronegative cases have HLA 827
(bl Muy be precet.led by UreLh ri ti.,
(c) SLE causes ero,1on of cartil age
( ti) I-lave a greatly increased im.:idenc<! of Porialic:
arlhnti,.
Nl /11S: / 99 /
58. Treatment of choice for a 4 week old femoral neck
frac ture in a 55 years old male is
(u) Open reduction and internal (ixation
(b) Mac Munay's ostemomy
(cl Hemi replacement anhropliL l)'
fr/) Total hip replacement RAIAST/lAN: IIJ<J I
59. Commonest coinplication of Trans-cervical fracture of'
femur is
(a) Non union
1b1 Maluclioa
(c) Avascular nccrosi:;
(d) All of the above.
60. The commonest hip injury iu the elderly patien ts is
173
(11) Stress #
(b) Extracapsu l ar #

174
(c) Tmpac1ed # neck of femur
(d) Sub capi tal capsular frnctt1rc neck of femu r.
61. Fractu re femur in children is treated by
(a) Open reduction
(b) Gallow's splint
( t·) Intra medullary nai l ing
(d) Closed reduction & spl intage.
62. Which is true about dislocation of hip joint'?
{a) Posterior dislocation is commoner
(b) In posterior dislocation whole lower limh i$ ro1a1cd
med i a l ly
{c) In anterior dislocation whole lower limh i rotated
lat
erally
(d) All or the above.

1. (a)
I ANSWERS
2. (ll ) 3. (b)
I 4. (d)
5. (c) 6. (d) 7. Cc) 8. (d)
9. (b) 10. (bl 11. (b} 12. (cl
13. (c) 14. (Cl) 15. (d) l6. (a)
J7. (c) 18. (b) 19. (II) 20. (11)
21. (b) 22. (b) 23. (c) 24. (d
25. (c) 26. (b} 27. (b) 28. (bl
29. (c) 30. (c) 31. (b) 32. (a)
33. (d) 34. (11) 35. {c) 36. (b)
37. (c) 38. (d 39. (I>) 40. (cl)
41. (a) 42. (c) 43. {a) 44. (d)
45. (b) 46. (b) 47. (d) 48. (,)
49. (n) 50. (bl 51. (c) 52. (c)
53. (d) 54. (c) 55. (d) 56. (c)
57. (rf) 58. (c) 59. (a) 60. (b)
61. Cb) 62. (t{)
m 1
JES OF THE KNEE AND

I. Which of the following causes acute compartment


syndrome most l'requcntly
{a) Fractures.
(/;) Postischcmic swelling.
(c) Exercise in i tiated syndrome
(d) Soft tissue injury PGJ.80, Al/MS.86
.2. ln transverse fracture of the patella, the treatment is
(a) Excision of a small fragmem
(b) Wire fixation
(c) Plaster cylinder
(d) Patellcctomy A/IMS 87, Kera/a 87
3. Acomminuted fractureof the Patellashould be treated bJ
(a) Inserting screws and wires
(b) Physimherapy alone
(c) Removal of all the pmclln
(d) Removal of smallest piece only
(e) Plastering & lmmobilization. BIH/\R. / 9fi9
4. Non union is a common feature of fracture of
(a) Supracondylar humerns
(b) Clavicle
(c) Lower 1ibia
(d) Coracoid process DELHI. J 989
5. A patient develops compartment syndrome
(Swelling, pain and numbness) following manipulation
and plaster for fractu re of both bones of leg. What
is U1e best treatment?
(a) Split Lhe plaster
(b) Infusion of low molecular weight dextran
(c) Elevate the leg after spli ning the plaster
(d) Do operative decompression of (ascial con partmenr.
6. Compartmental syndrome i5 treated by
(a) Fasciotomy
(b) Bicarbonate
(c) Chloride rich fluid
(d) Early aggressive 11ujd UI' : l\i'iY
7. ln fracture neck of fibula, the following nerve is invol\·ed
(a) Common peroneal ner ve
(b) Anterior 1ib1al nerve
(c) Posterior Li bial nerve
(di Medial popli tcal nerve lVfMS : 2000
8. Club foot with hypoplasia of calf musdes with inability
to extend knee and hips is seen in
(a) Arthrogryposi s
(b) Congenilal dislocat ion of hip
(c) Congeni tal myotonia
(d) Still's dit!llse
(e) Rei1er's disease PG/: /990
9. Healing below knee join t is slow because of
(a) Decreased subcutaneous fat
(h) lncreased movemen t
(r) We.ight bearing
(d} Poor vuscu l arity .IIPMER: l'NI
10. The classical example of m uscular violence is
(a) # of fibula
< b) # of patella
(c) # of clavicle
(d) All of these.
11. Tram,"Verse fracture of tile patella wiU1 separation of
fragments i5 best treated by
(a) Closed reduction wtth cylinder cast
(b) Open reduction wilh crew fixmion of 1he [ragmem,
(<') BIind fixation of thelwo fragmellls with Kirschner
wi,-:
(d) Open reduct on with Kirschner wire fixation of the
fragment an tension band wiring.
ANSWERS
1. 2. 3. 4. (r)
(a)
(bl (c) 8. (a)
5.
6. (al 7. (a}
(d)
10. 11.
9. (b) (d)
(d)

l. One of the following fr.ictm·e requires plaster of paris


cast with equinus position
(a) Distal frncture bolh bones leg
(b) Distal fracture fibula
(c) Bimalleolar
(d) Fractu re Talus PAL. 96

2. Fatigue fractures (Stress fractures) are most


commonly
in
Sl'ell
(a) Metatarsals
(bl Tibia
(c) Fibula
(cl) Neck of temur BIHAR. 1989
3. Cotton's fracture is
(a) A vulsion frncture of C
1
(b) Trimaoleolar is
(c) Bimalleolar (a) Collon'
(d) Bursi fracture of atlas s fracture
(e) None of lhese (b) Pons·s
fracture
4. Fractu re involving both the malJeoli
Bl/lt\R.1989
(c) Pirogoff s fracture
(d) Dupuytren's fracture. NIIMS:/ 992
5. Fall on bl with fractu re os calcis is associated
with commonly
(a) Fracture clavicle
(b) Fracture vertebra
(c) Fracture femur
(d) Posterior dislocaLion of hip DELHI: 1992
6. What is March fracture?
(a) Fracture of 2"'1 metatarsal
(b) Fracture of 4•• metatarsal
(c) Fracture of cuboids
(d) FratLUre of tibia
7. The usual site of stress fracture includes
(a) Tibia
(b) First metacarpal bone
(c) Second metacarpal hone
(d) Second metatarsal bone TAMILNADU : / 993
8. Stress fracture not involves
(a) Metatarsals
(b) Metacarpals
(c) Tibia
(d) Calcaniu rn PG! : 2000
9. The mecha nism of i11jury of transverse fractu re of
medial maUeolus resttlt due lo
(a) Abduction injury
(b) Adduction injury
(c) Rotation injury
(d) Direct injury ANDHRA: / 999
1O. Avascular necrosis is .a complication of
(a) Fracnu·e of talus
(b) Fracture of medial condyle of femur
(c) Olecranon fracture
(d) Radial head fraciuire ALL INDIA : / 999
11. Inwhich type of fracture,the tuber-joinl angle is
reduced to about half
(n) Crush fracture of calcai1eum
(b) Fracture neck of humerus
(c) Dislocatior: of shoulder
(d) Spilt fracture of calcaneum
(P ) Frnctu(C neck of femu r. AMU:1 990
12. Pott's fracture is fracture of
(a) Lower end of tibia
(bl Lower end of tibia + fibu la
(c) Lower end of tibia + C'alcancum
(d) Calcancu m + Talus />GI : / 9':ll
13. The commonest cause of palhological fracture is
generalised affeclion is:
(a} Carcinoma
(b) Os1coporosis
(c) Cy1
(d) A ll of the above.

ANSWERS
I. 2. 3. 4. (b)
(tf) 5. (aJ (b) 8. (h)
(h) 9. 6. (a) 7. 12. (b)
(1;) 10. (d)
13. (aJ 11.
(b) (a)
EllI INJURIES OF THE SPINE
I. Jefferson rractu re occurs at
(a) CI.
(b) C2.
(c) C l . C2.
(d) C2, C3. AU TND!A.99
2. ln spinal cord injury, the patient should be lrau sferred
with pad and bandage in Ute following posi tion:
Ca) Supine
(d) Prone
re) Lateral
(d) Semi prone ON/SSA . 1988
3. Cause of atonic bladder is
(a) Injury Lo sacral plcx u,
(b) Inju ry Lo upper L11oracic cord
(c) Pregnancy
(d) UT! UPSC./W!8
4. The comprcs. ion fracture is commonest in
(a) Cervical pine
(b) U pper d1orud pine
{c) Lower thoracic spine
(d) Lumbo,ac:ral region NAT!ONALBOARD: / 91J2
S. Emergency treatment required in
(a) Fracture humerus
(b) Fracture pelvis
(c) Comminuted fracture femur
(d) Fractu re tibia & Fibula RJ\JASTAN :/ 992
6. Bu rst fracture of cervical spine is due to
(a) Whiplash injury
(b) Fall of weight on neck

180
(c) Veriical compression injury
(d) Car accident IU/fAR.19R9
7. Spinal shock is associated with
(a) lncre,ased spinal renexes
(b) Absem spinal retlexcs
(c) Loss of autonomic reflexes
(d) Bizarre reflexes AIIMS:1990
8. Position
for
transport
of a
patient
with
fracture
lumbar
spine in a
patient
(a) Neutr
al
(b) Hyper
nexio
n
(c) Ahem
ating
(d)
Hypercxte
nsion

99-1
9. A pa
ralysed
bladde
r
followi
ng
spinal
injury
181
is hest managed by e
(a) t
G c
i r
b
b (
o c
n l
'
s F
o
c l
a e
t y
h '
e s
t
e c
r a
t
( h
b e
) t
e
M r
a (cl)
l Metallic
i catheter
c
o 10. Vertebral
t disc
prolapse
c with
, intact ,
i ·ertebra
t com mone
h st

182
in
(a) Idiopathic
(b) Mali&naocy
(c) Trauma
(d) Tuberculosis
11. Careless handling of a
suspected ca5e of cervical
spine
injury may result in
(a) I njury to the spinal cord
leadiJ1g to quadriparesi, or
q
u
a
d
r
i
p
l
e
g
i
a
(b) lntracranial haemon·hagc
with cerebral imtation or
unconsciousness.
(c) Cervical haematoma with
compression of brachia!
ve. sels
(d) Complete paralysis of the
affected upper extremity
UPSC : 1999

183
12. Inspinal shock
(a) Knee jerk is tl1c fll'Sl reflex to return
(b) High thoracic lesion, are common!) a, ociated with
more severely a sociated with more severe
neurological delicits
(c) Failure of retu rn of cord activity withi n 48 hou r
i1t a very poor progno tic sign
(d) Botl1 B & C NIMHANS ·/ 91)9
13. Retu rn of Bulbocaveroous reflex in spinal shock
(ll) Sign of recovery indicates from paraplegia
(b) Partial lesion of spinal cord
(c) Complete u-ansecL on of spinal cord
(d) Incomplete tra nsection of spina l cord JIPMER : N99
14. Disloca tion without fracture is seen in
(11) Sacral spine
(b) Lumbar spine
(c) Cervical spine
(d) Thoracic spine DELHI ·1999
15. True regarding Hangman's fracture is
(a) Odotoid process fracture of C2
(b) Spondylolislhesis of C2 over C3
(c) Whiplash injury
(d) Fracture of hyoid bone MAN/PAL : 2000
16. The most important step in primary management of a
patient with fracture vertebral column
(a) Careful transport of patient
(/;) Maintenance of airway
(c) Treatment of hock
(d) None of the above PG/ : 1994
17. Hangman 's fracture is
(a) Subluxation of CS over C6
(b) Fracwre dislocation or C2
(c) Fracture dislocation of ,mkle joim
(d) Fracture of o<lontoid KARNATAKA: 1998
18. A patient presen ted with Saddle anaesthesia, bladder
& bowel are normal and muscle power is normal. The
diagnosis is
(a) Cauda equjna syndrome
(b} L3-L4 root invol vement
(c} Conus medu l la ris .lesion
(d) L4-L5 disc prolapse NIMS:/1.)98
1.9. Fatigue fracture does not occur in
(a) Tibia
(b) Calcaneum
(c) Metatarsal
(al Metacarpal PG/ : 1998
20. In case of unconscious patient spinal injuryis assessed by
(<1) Absence of response 10 pai nful stimulus
(b) Absence of deep reflexes
(c) Abdominal respirati1Jn
(d) Spinal tenderness. WEST BENGAL 19')8
21. A scooter is hit from behind. The rider is thrown off
and he lands with his bead hitting the kerb. He does
not move, complains of seve,·e pain in the neck and is
unable to turn his head. Well-meaning onlookers rush
up to him and t ry to make him sit up. What would be
the best course of action in this situation?
(a} He should be propped up and given some water to drink
(b) He should nm be propped up but turned on his face ,ind
rushed to the hospital
(c) He should be turned on his back and a support should
be placed bellind his neck and transported to the
nearest hospi tal
(d) He should not be moved at all but carried to the nearest
hospital in tbe smne position in which he has been since
his fall. UPSC :
1997
22. Stress fracture is treattd by
vi) Rest
(b) Cast immobilisaLi ou
le) Closed red uction
(d) Internal fixation DEUil :
199()
23. In ulna r nerve palsy, st.ructurc pa ralysed is
(a) All Jumbricals
(b) lnterossei
(c) Sensation on medial aspect of fingers
(d) Adductor policis l)EUfl : / 990
24. Regarding whiplash i-njury, a true statement includes
(a) Contusion of the spina l cord and fracture of
venehra (b) Fracture of vcrtcbrnl body
(c) Spi nal cord inju ry withc,ut vencbraJ fracture
(4) Vencbral fracture wit hou t spinal cord injury
AN DH RA:/ 991
25. The commonest cause of spinal cord injuries in our
country is:
(a) Road traffic accident
(b) Fall from a height
(c) Fall into well
(d) House collapse.

I ANSWERS I
1. (a) 2J . (c)
2. (<1) 3. (a)
s. 25. (b)
Ii. (c) 7. (b,c)
(c) 10. (a) 11. (c,)
9. 14. (C) 15. (b)
(c)
18. (cl 19. (d)
13.
(a}
22. (b) 23. (b.d)
17.
(b}
4.
(c)
8
.

(
a
)

U
.

C
b
}

L
6
.

(
/
}
)
20. (a.
cl
24.
(c)
mI FRACTURE OF THE PELVIS
1. Late complication of Acetabular fracture
(a) Avascular necrosis of head of femur.
(b) Avascu lar necrosis of lilac crest
(c) Fixed defom1i1y of the hip jo i nt.
(d) Secondary osteoarthritis of hip jo in t P(il.97
2. In pelvis fracture, tl1e amount of blood loss is arou nd
(a) J -4 units
(h) 2-4 un its
(c) 2-6 u nits
(d) 4-8 units TAMTl NAl>U : 1999
3. If a patient wilh a supected fractu re of the pelvis
has some bleeding from the urethra and is u nable to
pass urine
(a) He should be encouraged to pass urine after bei ng
gi ven an ti biotics and analgesics
lb) He should be immediately cathete1ized in the ward
(c} A ho1waler bonle should be given followed by injection
of carbachol
(d) He should be prepared for surgery and catheteri1.ation
altempted io tJ1e Q.T. UPSC :
1997

I ANSW ERS
1. (d) 2. (d) 3. (d)

185
EllI POLYTRA

UMA
1. Patients comes with fracture femur inan acute
accident, the first things to do is
(a} Secure airway and Lreat the shock.
(b) Splint ing.
(c) Physical examination.
(d) X-rays. f'Gf.86
2. In an injury with mul tiple fractures, most importalll is
(a} Ail'way main tenance
(b) Blood transfusion
(c} lntravenou fluids
(d} Open reduction of fractures. DEU/f.1988
3. The correct order of priorities in the initial
management of he.ad injury is
(a) Airway, Breathi ng. CircnJ ation Treatment of
extra cran ial inju ries
(b) Treatment of extra crnn.ial injuries Airway, Breathi ng.
Circulation
(c) Circu lation , Airway, Brcatl1ing Treatment of excra
cranial injuries
(d) Ai rway, Circulatio:t1. Breathi ng Trenuncn L of extra
cranial injuries CSE : IY9Y
4. Consider the following sign (S)
(I) Increasing pallor
(2} Restlessness
(3) A i r hunger
(4) Water-ham mer pulse
Haemorrhagic shock due to aeute blood lo,s incl udes
(a) I& 4
(b) I & 2
(c) l,2 & 3
(d) 2,3 & 4 CSE : 1999
ANSWERS
J . (a) 2. (a) 3. (a> 4. (c)

Ell I COMPOUND FRACTURES

1.Compound Fracture is
(a) Fractt1re with artery involvement
(b) Fracture with nerve involvement
(c) Fracture with muscle involvement
(If) Fracture with ski111 invol vement DELHl./ 98/oi
2. In shotgun injuries
(a) Each and every shot should be removed
(h) All the shots within accessible limits may t>e
removed and thorough debridemenl or the tissue done
(c) Shots lodged in joi nts must be removed
(d) All the above are trnc

3. Tetanus is noticed usually in


(n) Rum cns,s
(b) Wounds contamilllated with faecal matter
(c) Open fractures
(d) Gunshot wounds
(e) All of the above.
4. Following are principles in the treatment of compound
fractures except
(a) Wou nd debridemem
(b} Lmmediace wound closure
(c} Tendon repair
(d) Aggressive antibiotic therapy J!PMER: 1993

187
5. A pat ientpresent with. compound fracture of Tibia with
1cm opending in skin.Which grade it belong. ?
(a) Grade r
(b) Grade Tl
(t) Grade ITTA
(Ii) Grade T1TB JJPMER . 2000
6. Which of the following is the most appropriate hospital
treatment of a patient with compound fracture'?
(a) Under aneslhesia. thorough scrnbbing and cleaning of
tJ1e area gett ing Lhe fracture end inside, smuri ng the
wound and applying continuous skelt!tal Lracc ion
wilh adequate antibiotic cover
(b) Clea ning and suturi og tJ1e wound , oi pplying
pla,ter Spica u nd er lraction on a Kar l y's
tabl e and ad mi nistering antibiotics round the dock.
(c) Scru bbi ng ruid cl eanin g the area. resect ing
the protrudi ng one inch of the bone, sutu ring the
wound, bringing 1he fraclured end, i nto a lignmem and
apply i ng plaster Spica with continuous anlibiotic
cover.
(d) Thorough cleaning of the area.extendjng chc
wound. bri ngi ng the fragments into alignment
under vision. fixi ng them wi th inlra-mcdul lary na
il and gi ving an ti biotics to 1he patienr. UPSC : /
999
7. Internal splints are used in all except
(a) Compound fractures
(b) Multi ple fractu res
(c) Fractures in elderly palicnl
(d) Fracture neck of femur MAN/PAL : 1999
8. Open fracture is treated by
(a) Toumiquet
(b) Internal fixation
(c) Debridement
(d) Extemal fixation UP : I998
9. A compound fracture is initially treated by antibiotics,
wound toilet and
(a) Skin cover
(b) Extemal splintagc
(c) Prosthesi;,
(d) lntemal fixation

I O. Immediate treatment of compound fracture of tibia is


(a) JntravenotL an tibiotics
(b) Thorough irrigation of wound with sal ine and
splintage (r) Wound deb1idemenl
(tf) Internal fixation of fractu re OF.I.HI: / 997
11. Symmetrical multiple spontaneous fractures occur in
(a) Milk man syndrome
(b} Ostcogencsis impc1fecta
(c) Rickets
{1/) Scurvy A/IM S : / 992

ANSWERS

1.
2. 3. 4. (b)
(d) (b)
(b) 8. (cd)
5. 7.
(a)
6. (a)
10. (a)
9.
(b) 11.
(b)
(h)

fflSOFf TISSUE INJURIES


5iWINCLUDING SPORTS INJURIES
1. \Vhich of the following is con·ect in medical meniscus
tear
(a) Rotalioo of femur oo tibia
(b) Mcosici do not heal
18
9
(c) Locking aod unlocking episodes

18
9
(d) Menisci shou ld be excised
C.,) A II of the about are correct .!IPMER 87.Kem/11 87
2. Best diagnostic procedu re for ant. cruciate ligament
injury is
(a) L11chman's test
(bJ Pivot shift test
(c) Anterior drawer test
(d) Mc Murray's test .IIPMER 2002
3. Lach man test is positive in
(a) Anterior cruciate liga ment iojury
(b) Posterior Cruciate ligament injury
(c) Medial meniscus injury
(d) Larcral meniscus inju ry DELHl. 1988
4. The most common site for ligamentous injuriesare
those of the
(a) Shoulder joi nt
(b} Elbow
(c) Knee joi nt
(d) Ankle joint ANDHRA: 1999
S. Injury to the medial meniscus is rather impossible
when the knee jont does not
(a) Extend
(b) Flex
(c) Rotate
(d) Abduct Adduct NATION BOARD: 1989
6. All of the following features in the knee are
recognized to be consistent with a torn medial
meniscus, except
(a) Excessive forward glide
(b) Giving way
(c) Locking
(d) Mc. Murray' ,ign. MAN/PAL : /

190
994

190
7. Investigation for il\ju ry of knee cartilage
(a) Aspiration
(b) Anhrography
(c) X-ray
(d) Arlhrnscopy A/ IM S: 1994
8. Which is the investigation of choice for a sport injury
of the knee'?
(a) Ultrasonography
(b) Plai11 radiography
(c) Anhrograph y
(d) Arthroscopy AN DH RA: /
993
9. Drawer sign seen in
(al Cruciale Ligament injury
(b) Scurvy
Cc) Pcrches's disease
(d) Hyperparathyroid ism RA.!ASTHAN:1992
10. Athletic sustained an injury around the knee join t
suspecting cartilage damage, which of the following i
an inve-stigation of choice ?
(a) Pain X ray
(b) Clinical examination
(c) Arthroscopy
(d) Arthro1omy ANDHRA ·2000
11. Ruptured tendon is most commonly seen in
(a) Stab injLUy
(b) Sof1 Tissue tumotU·
(c) Over use
(d) Congen i tal defect ALL.LVDIA : 2000
12. In Neu ronotemesis, the following statement is correct
(ll) There is on l y a ph ysiological u isru ption . hu t
anatomically the nerve is in1.11c1
(b) On ly the Axon sheaih is damaged
(c) Axonal sheath and supporting cells are damaged

191
(d) Axon ru1d Ax.on sheath s are damaged. NIMS : 2000

191
13. The most common type of mode of meniscal injury is
(a) Extcnsio of knee
(h) Piexion of knee
(c) Flexion and rotation movement
(d) Extension and rotation of movement. ANDH RA: 1999
14. Injury arou nd the ankle joint occurs at
(a) Inversion of foot
(h) Eversion •>f foot
(c) Internal rotation or foot
(a) External rotation of foot BIHAR:/ 999
15. The most common ligament injm·ed arou nd anklejoint
is
(a) Anterior talolibular
(b) Deltoid ligament
(c) Posterior taloli bular
(d) Spring ligament ALL IND!A. /
99<'1
16. Bucket handle tear at knee joint is due to
(a) Inj ury to medial collateral ligamem
(b) Injury to lateral corlateral ligament
(c) Tnjury to ligamentum patellae
( d) lojury t<i menisci OR/SSA: 1991
17. Whlcb of the following statement is I are t rue?
(a) Acute OsteomyeliLis is common in children
(b) Osteogenic sarcoma and Ewing 's tu mou r ocrn r
commonly in 2"" decade
(c) Ewing's tumour arises from tl1cdiaph.ysis of long bones
(d} All of lhese.
18. A patient gives a H/0 twisting strain and locking of the
knee join t, the most likely diagnosis is:
(a) A vulsion f tibial tu bercle
(b) Meniscal tear
(c) Tearing of lateral coUateral ligamenr
(d) Tear of anterior cruciate ligament.
192
ANSWERS
I. (e) 2. (a) 3. <al .... (d )
5. (r} 6. (a) 7. (r/) 8. (d)
9. (t,) 10. ll. ( 12. none
13. (c) <:) 16. ( ti)
(c} 14. ( 15. (Cl
17. b) )
(J) 18. (b)

maI NERVE

INJURIES
1. Tardy ulnar nerve pal y is due to;
( rt ) Cubitus valgus.
(b) Fi xation of nerve 111 lhe groove by ns1eoanhti1i ,
(c) Excision or elbow jo i nt
( d) Fracm re of internal c-ondyle UPSC.86. KERA/ .11.Rl
2. Tardy ulnar nerve pals)' is seen wit11
(a) Lateral humeral condyle fractu re.
<b) Supracondylar fracture
(c) Medial humeral condyle fractu re.
Cd) Fracture capitulum Alli'v/S. 9J
3. A Patient presented nith claw hand after a Supracondy
Jar fracture was reduced and plaster applied. The
diag nosis is;
(a} Median Nerve injury.
(b) Volkman n' ischacmic: C(mtracture
Cc Ulnar nerve injury.
(d) Dupuytren's contrac1urc ALL, JNDIJ\.92
4. The "Card Test" tests the function
of
(a) Median nerve.
(b) Ulnar nerYe.

193
(c) AxiJ lary nerve.
(d) Radial nerve UPSC.95
5. Dislocation of which one of lhc following carpal bones
can present a median nerve palsy?
(a) Scaphoid.
(b) Hamate.
(c) Lunate.
{d) Trapezi um UPSC.95
6. Cock up splint is used in management of
(a) Ulnar nerve palsy.
(b) Brachia!plexus palsy.
(d Radial nerve palsy
(d) Combined ulnar & Median nerve palsy AIIM S.95
7. Injury of media n nerve at wrist is best detected by
(a) Action of abductor poll icis brevis.
(/)} Action of flexor pollicis brevis
(c) Loss of sensation of radial ha l f of pal m
(d) Loss of sensation -0f tip of ring finger ALI. INDJA 97
8. Non Dynamic splint is :
(a) Banjo
(b) Opponens
(c) Cock up
(d} Brand l'Gf 87
9. Pointing index sign is seen in ...... nerve palsy
(a) Ulnar
(b) Radial
(c) Med ian
(d) Ax illary UPSC 86', Kera/a
10. Commonest cause of wrist drop is 87
(a) Intramuscular injection
(b) Fracture humerus
(c) Dislocation of elbow
(d) Dislocation of shoulder

194
11. The lesion in Klumpkc's paralysis is at
(ll) Cervical plexu
(b) Lower brachia!
(c) Upper bracfljaJ
(d) Sacral Plexus Al' 88
12. The mode of injury of Klumpke's paralysis is
(a) Forcible increase in the neck shoulder angle (as duri ng
child birth}
(b) Forci ble rotation of upper ann
(cl Forcible hyperabduction or upper arm ( Fal l ing person
tryi ng to catch an object}. B!H,\N : J 988
13. Injury to which of the following nerves is most likely
to result in Wri t drop?
(a) R adial
(h} Ulnar
(c) Median
(ti) Musculocutaneous
14. Legg-Calve-Pen.he' s disease iscommonly seen in the
age
group of
(a) 1-3 years
(b) 3- 10 years
(c) 10-20 years
(d} 20 years & 11bovc. AU. !Nn!A.·1911()
JS. Satu rday nigh t palsy involves nerve
(a} Radial
(b) UJ nar
(c) Median
(d) Axillary
(e) Any or t he above. AM C.!989
16. After inJury to Radial nerve iu radial g1·oove, first sign
of recovery is
(a) Tincl"s sign
(b) Power in Extensor carpi rad ialis longu$
(l') Power in Brnchioraclialis
(ti) Power in Triccp, Alllr!S. I Wll.J
17. ln fracture of distal half of humerus. the nerve injured is
(t1) A xillary
(b) Med ian
(c) Radial
(d) Ulnar ES/.
f<)8<)
18. In Seddon 's classitkation. complete division of ner ve
is
(a) Neuropraxia
(i?) Axonotmesis
(c) Ncurotmcsis
(d) None or t he .ibovc. ROH7i\ K 1989
19. Anterior interosseous nerve is a branch of
(u) Musculocutaneou
(h) Radial
(c) Med ian
(d) Uln,1r A LL INI>IA. l
t.J8<J
20. Froment's sign is pre.sent in paralysis of .................
il\j ury.
(a} Llnar
(b) Radial
(c) Median
(d) A xilhu·y J)f;L/ /1 JQCJ./

21. Which of the following ner ve injuries produce the


deformities?
J. Upper Iru nk C T (iii) IT (i1• J
[I. Ulnar nerve D T (iv) H (iii )
lll. A xillary nerve
rv. Radial nerve
A I (i) II (ii)
B l (ii) 11
(iii)
l. Wrist drop
II. Porter's ti p hand
ill. Cla\\ Imnd
IV. Flattening of the
shoulder
lll (,ii) TV (i1·)
ITT (iv) [V (i)
Ill (ii} IV (i)
m t iiJ rv (i)
DNH /
993
22. "Vbich of the following nerve is most often damaged
in fracture of shaft of humerus
(a) UIMr
(h) Median
(cl Radial
(d) Ax illary l'Gl: J9<J3
23. J•'oUowing indicate better prognosis is in inju ry e;1;cept
(a ) Neu roproxia
(/J) Younger age
(c) Pure motor nerve inju ry
(ti) Proximal injury JJPMER /99.l
24. Meralgfa Paracsthetic.a involves
(11) Axillary nerve
(b} Sural nerve
(c) Med ian Nerve
(d) Lateral cutaneous nerve of thigh. DNB /993
25. Find the wrong match
(a) Ankylosing spondyli tis -Lum bosacral spine in vol ved
(b) lntervertebral disc rrol;ipsc -Sensory deficit
(c) Pott' s spine -Tuberculosis
(d) Saturday night pa lsy -Radial nerve palsy in wrist.
UPSC.1992
26. Median nerve is injured during
(a ) Elbow dislocation
(b) Fractu re lateral epicondyle of htutieru
(c) Fracture medial cpicondylc of humerus
(d) Supracondylar fracHLre of humerus .lfPMER : '2000
27. Feature of radial nerve injury at spiral groove
(a) No wrist drop
(b) Sensory loss over deltoid
(r.) Thu mb. finger. wriS1 drop
(d) Wrist drop + Extensors of Forearm paralysis
TAMIL NW, U ·/ 999·

197
28. Erb's palsy is due to involvement of'
(a) Upper trunk of brachiaJ plexus
(b) MiddJe tru nk of brac hia! plexus
(c) Lower trunk of brachia! plexus
(d) Long thoracic nerve UP . / CJIJ9
29. Claw hand is seen in
(a) Cervical rib
(b) Carpal tunnel syndrome
(c} Multiple scelerois
(d) Ulcer nerve irtjury NATIONAL BOARD: /999
30. Which one of the folloVl'ing statement is wrong
about Erb's palsy?
(a) Abd uctors of the shoulder arc weak
(b) Upper part or hrachial plexu is i nvol ved
(c) Supinators are nom1al
(d) Internal rotation of am1 NIIMS :
21X/O
31. In Neurotemesis, the following statement is correct
(a) There is only a physiological disruption. bul anatomi-
cally Lhe nerve is inutcl
(b) Ortly the Axon sheath is damaged
(c) Axoual sheath and supporting cells are damaged
(d) Axon and Axon sheaths are dnmngcd NIMS : 2000
32. Erb's point is located at the junction of
(a) Anterior primary rami of C5 and C6
(/,) Anterior primary rami of C8 and Tl
(c) Superior and middle trunk of Brachia! plexus
(d) None of the above.
33. Match the following:
(i) Erb's paralysis (iv) Fractures sargicaJ
(ii) Klumpkc:' s neck h u merus.
paralysis
(iii) Crutch paralySi$

198
(i) Lower cru n k injury
(ii) Axillary nerve
injury
(iii) Rad ial nerve inury
(iv) Uper trunk inju ry

199
A 1 (iv) u (ii i) ill (ii) IV (il
B T (iii) II (ii) w (iv) IV (i)
c I (iv) u li) ill (iii) IV I ii)
D l (i) II (iii) ill (iv) rv (ii )
NATfONAL BOARD: 1999
34. Consider the following nerve injuries
(I) A xonotmcsis
(2) Neurotmesis
(3) Ncuropraxia
The Correct seq uence of these injuries in term,
o f 'increasing ' severity of damage to the nerve fibres
and sheath. is
(a) 1,2 and 3
(b) 3.2 and I
(c) 2.l and 3
(d) 3. l and 2 CSE : IY 99

35. Following an incised wound in the front of wrist,


the subject is unable to oppose the tips of the little
finger and the thumb.The nerve (S) involved is I are
(a) Ulnar nerve alone
(b) Median nerve alone
(c) Median and ulnar nerves
(d) Radial and ulnar nerves UPSC:2000
36. The nerve root5 involved in Erb's palsy are
(a) CJ C4
(b) C4 C5
(c) C5 C6
(d) C6 C7 TAMfLNADU : 1994
37. Radial nerve is most commonly injured in fracture of
(a) Mid shaft of humerus
(b) Neck of hu merus
(c} Lateral condyle of lnumerns
(d) Lower 11,n1 of humeru s UP : /994
38. "Ulnar paradox" is related with the following
(Cl ) Lumbricals
(b) Intrinsic muscle
(c) EPL
(d) U lnar h alf of FDP (Ip 1994
39. Vl'rist drop results in injury of
(o) Ulnar nerve
( /,) Med ian nerve
(c) Posterior intcrosseus nerve
(d) Radial nerve UP 11.)9./
40. Match the following
I. .crb·s par:J lys1s
II. Klumpkc' parn lysi,
nr. Crmch p.u·alysis
IV. Frucwre urgical ne,k hu111eru:,
(i ) Lower tru nk injury
(ii ) Axilktry nerve inju ry
(iii) Radial nerve injury
(iv) Upper tru nk injury
A. I (iv) Tl ill IV Ul
(iii) (ii) N (i)
B. l (iii) 11 Ill
(i;J (11•)
C. T (Iv) Tl (i) Iii (iii) J y (ii)
D. l (il 11 (iii l fTI ri\·l IV (ii)
NA T/ ONAT BOARD:1998
41. Meruan nerve injury at the wrist causes
(a) Claw hand
(b) Loss uf apposition of tl:tumb
(c) Policeman '!; ti p dcfom111y

(d) Satui·day rrigh palsy (h)


Claw
42. Ulnar nerve injury at wrist result in
hand
(a) Unable to gra p card between the (c)
finge1"l'
200
Inabi l ity to abduct or adduct t he finge.rs !'GI: 1998
(d) All of the ubov.:. MAN/PAL: /')':17

201
43. In a full term newly horn male baby, on routine check
up soon artcr delivery ha weak Moro's retlex on right
band bu t int.act Grasp's retlex with weak abduction
and supination movemen t of'hand is found. Thi.s will
suggest that most likely injury would be at
(o) c,.,, nerve.
(/J) C,.C, nerve
(cl C,.T, nerve
(d) T,.T:nerve. AlltdS /999
44. A 25-year-old lady su tained a lacerated wound on
the back of right thigh by a horn of a bull.The wou nd
wa sutured. Two months later she developed foot drop
and an ulcer on the dorsu m of the foot. The most
likely diagnosis is
(a) Chronic ischaemia to limbs due to Poplite.11,inery inju1)
(b) Partial injury to sciauc nerve
(c) Complelt! di, ision of sciatic nerve
(d) Injury lo hamstring muscles Ul'SC:19'17
45. Nerve suturing in a dean cut injury is done best in
(a) 6 hOLlfS
(/J) l 2 hours
f cJ After one day
(d) After two tlay. DJ::I.Hf: f <Jl.)0
46. Tourniquet paralysis is an unfortunate complica tion
leads to
(a) Neuropraxia
(/J) Axonotme,is
(c) Neu rotmesis
(d) None of the above KARNATAKJ\ : / 990
47. Prognosis after secondary nerve s11turing is better in
r>ure than inmixed ones. Based on this criterion, which
one of the following nerves should be given the best
result after suturing in identical conditions?
(a) Common peroneal nere
(b) Radial nerve
(c) Ulnar nerve
(d) Median nerve KAl?NATAKA:/
91)0
48. Disability of ha nds is maxi mum with a lesion of
(a) Median nerve at elbow
(b) Median nerve at wrist
(c) Ulnar nerve at elbow
(d) Ulnar nerve at wrist
(e) Radial nerve lesion. PG/ : / 9YO
49. Following structures pass through spiral groove of
humerus
(a) Radial nerve
(b) Radial artery
(c) Anterior ci rcumflex humeral artery
(d} Posterior eircumtlcx humeral artery
(e) Ancria profunda brachii PG!:1990
50. Inju ry to radial nerve - mjd humerus level
following occurs
(a) Paral ysis of extenso1:s of hand und wrist
(b) Equal ly diagnosed by sensory and motor teting
(c) Can be approcherl by a lifting triceps
(d) Can be approached by splitting laLissimu dorsi
(e) Paralysis of Triceps
51. Section of lateral Popliteal nerve at the neck of fibula
causes
(a) Fool drop
(b) Trophic ulcers on Loes
(c) Atrophy of calf muscle
(d) Pain in the leg PG!:1990
52. Rate of regeneration of severed nerve is
(a) 0.1 nun/day
(b} 1mm I day
202
(c) Icm/day
(d) None. A.ND /-ffl4 :/ 991
53. "Tinel's sign'' indicates
(a) Neu rofibroma
(/J ) lnjury to peri pheral nerves
(c) Atrophy of nerve,
(d) Regeneralion of nerves. BIHAR: / 991
54. 'Ape tbumb deformity ' is observed in lesion. or
(a) Radial nerve injury
(bl Ulnar nerve injury
(c) Median nerve injury
(d) Circumflex humeral nerve injury
(e) Posc.:rior inlerosseous nerve i11jury BIHAR: { 99 i
SS. Cock's splint is used in
(a) Radial nerve palsy (/J)
Ul nar nerve palsy
(c) Med ian nerve palsy
(d) Dupuytren's Contr.acture DELH I: 199/
56. Ulnar nerve severed a.hove elbow causes
(a) Complete loss of sensation in 4"' and 5'11 finger
(b) Paralysis of all lumbricals
(c) Paral ysis of all interossei
(d) Paralysis of llexor carpi ulnaris
(e) Paralysis of llexor profundus P(;J : / 99 J
57. Foot drop result because of injury to
(a) Superficial peroneal nerve
(b) Deep peroneal nerve
(c) Posterior tibial nerve
(dJ Anterior libial nerve PG! : / 99 /
58. A factory worker has laceration proximal to wrist
with no opposition of thumb and fingers, loss of
sensation over lateral aspect of' thumb and fingers.
Diagnosi i injury to
(a) Median nerve + Ulnar nerves

203
(bJ Median + Rd ia l ner\'CS
(c) Radial + Ul nar nerves
(d) Medi<1n nerve nnly l/ l'SC :
1991
59. Claw hand occurs in
(d)
(a) Volkmann· 1whaemic Contraclurc
Sudeck's
(b) Dupu ylrcn· Concracw re
alrophy
Cc) Ulnar nerve injury
INDJA
·19<,;J
60. Best
results of
nerve
:suturing
are seen
in
(
a
)

M
e
d
i
a
n

n
e
r
v
e

204
)

U
l
n
a
r

n
e
r
v
e
(c) SciaLic nerve
(ti) Radial nerve

ANSWERS
I. (a) 2. (aJ 3. (h)
5. (c) 6. (c) 7. (n)
9. (cl IO. lhl 11. (b)
13. (a) 14. (b) 15. (n)
17. (c) 18. (c> 19. Ir )
21. (h) 22. (C) 23. (d)
25. (d) 26. <a) 27. (c)
29. (d) 30. (c} 3L None
33. (I') 34. (d) 35. f c)
37. (a) 38. (b) 39. (d)
41. (h) 42. (lfJ 43. ({I)
45. (al 46. (a) 47. (b)
49. (ae) 50. (a) 51. (n)
53. (d) 54. (c) 55. (n)
57. (b) 58. (a) 59. (t•J

205
mI
AMPUTATIONS
1. Mc. l\ilurray's sign is seen in in.jury to
(a) Med i al 111c1tlscus
(b) Medial collateral ligame111
(t} Anterior cruciate ligamenl
(d) Posterior cruciatc ligament Ul'!)C./ 9Xl'.

2. Dbtancc from Olecran-0n in amputation should he


((I) 5-10 cm
(b) I0-15 crn
(c) 15-20 cm
(d) 20-30 cm
3. Ring sequestrum is seeo in
(11) Typhoid osicomyelitis
(b) Chronic OS\COIUyelitis
(c) Amputation slwnp
( d) Tuberculosis ostcomyclitis TAMIL NADU: /992
4. In flap method of amputation which struclure is kept
shorter than the level of ampu ta tion:
(a) Bone
(b) Muscles
(c) Nerve$
<b) Skin
(e) Ye,sels NATIONAL BOARD :1992

I ANSWERS I
1. (a) 2. (a) 3. (<') 4. (Cl )
LATEST QUESTIONS
AIIMS: NOVEMBER 2002
1. Medical meniscus of the knee joint isinjured more
often than the lateral meniscus because the medial
ml'niscus is relativdy:
(a) More mobile
(b) Less mobile
(c) Thinner
(d) J\Llached lighLly to femur

2. Which of the following is not a component of the


crush syndrome:
(cz) Myohemoglobi nu ria
(b) Massivc cru.5him!uf muscles
(r) Acute tu bular necrosis
(d) Bleeding dimhesis

3. A 40 year old man was repairing his wooden shed on


Sunday morning.By afternoon, he fell that the ham
mer was becoming heavier and heavier. He felt pain in
the lateral side of elbow and also found that squeezing
water out of sponge hu rt his elbow. Which of U1e m
uscles are most likely involved:
(a) .Biceps brachii and supinator
(b) Flexor digitonun superficialb
(<') ExtCU$Or carpi rad i.alis longus and brevis
(d) Triceps brachii anul anconcus.
4. Tn dislocation of Uie jaw, displacement of the articula
r disc beyond the articula r tubercle of the
tcmporomandi bular joint results from spasm or
excessive contraction of the followiog mtL cle:
(a) Buccinator
(b) La teral pterygoid
(c) Masseter
(d) Temporalis

206
5. The classical llexion am!rotation deformities at hip
and knee joints, a a sequelac of poliomyelitis are due
to tJ1e contracture of:
(a) Tensor fascia lata
(b) Gas1rocnemius
(c) Tcndo Achille.
(d) Hamstri ngs
6. \\'hat would he the most reliable test for an acutd;v
inju red knee of a 27 :year old athlete:
(a) Anterior drawer test
(b) Posterior drawer tC$l
(c} Lachman test
(d) Steinmann test

7. A 11year old boy presented with the complaints of


1>ain in the right arm near the shoulder. X-ray
examination revealed an expansile Jytic lesion in the
upper third of humerus. The most Hkely diagnosh is:
ta) Giant cell rumour
(b) Unicameral bone cyst
lC) Osteochomlroma
(l[) Paroteal osteosarcoma

8. Which activity will be difficult to perform for a patient


with an anterior crnciate deficient knee joi nt?
(a) Walk downhill
(b) Wall. uphi l l
(c) Sit cross leg
(d) Gelling up from sitling.

ANSWERS
1. (
b) 2. (d) 3. 4. (/))
6. (c) (c) 8. (al
5. (a)
7. (b)
MAY 2002
T. Most common site of adamantinorna of' the long bones
is:
(a) Femur
(b) lJJ na
(c) Tibi11
(ti) Fibula

2. Chond roblastoma most commonly occurs in:


(a) ."vlclaphysis
(b) Diaphysis
(<") EpiphyNb
(d) Medullary cavity

3. Ram u , an 8 year old boy presented with pain inthe


arm. On x-ray, hi upper end of' humerus
demonstrates an expansile lesion in the mctaphysis
witb breech of the overlying cortex. Most likely
diagnosis is:
(£1) Ancurysmal bone cyst
(b) Unicameral bone cysl
(c) Chondroblatoma
(,[) Os1eoc lastoma

4. A twisti ng injury of knee in flexed position would result


in injury to all except:
(a) Mcniscal tear
(/J) Capsul,U' tear
(c) Anterior t'.ruciate ligame111
(d) Fibular collatc:ral ligament.
S. Chondrocalcinosis is seen in:
(a) Ochronosis
(b) Hypoparalhy roi<lism
(c) R ickets
(d) Hypcrvirnmino is D

208
6. Torsion of' knee results in injury most commonly to:
(a) Anterior cruciaLe ligament
(b) Medial meniscus
(c) Fibular collatera l l igament
(d) Tibial collateral ligmnent

ANSWERS
I.
2. 3. (al -'· {d)
(a) Cc)
5. (a)
6.
(b)

NOVEMBER 2001
l. Painful arc syndrome is seen in all except:
(a) Complete Lear of supraspinatus
( /)) # grea1er 1uberosity
Cc') Subacromial bursitis
(d) Supraspinaws tendonitis

2. A patient of nephritic syndrome taking sferoids for 6


ycaras presented with a limp gait and of limitation of
abduction andinternal rotation. Re most probably ha
:
(a) Renal ostcodystrophy
(b) Avascular necrosis of 1he hip
(c) Septic arthritis
(d) Osteomyelitis of the hip joint.
3. An 11 year old 70 kg child presents with limitation of
abduction and internal rotation. There is tenderness in
the scarpa's triangle. On flexing the hip, the limb is
adducted. The diagnosis is:
(a) Perthe's disease
(b) Sliped capital femoral epiphysis
(c) Observmion hip
(d) Tuberculosis hip
4. A 19 year young patient with sclerotic lesion at
the diaphysis. The diagnosis is:
(a) Osteoid osteoma
(b) Ewing's sarcoma
(c) Osteoclastoma
(d) Metastasis.
5. Most common lesion of tJ1e mandible is:
(a) Ameloblastoma
(b) Squamous cell carcinoma
(c) Osteosarcoma
(ti) Ostcoclastoma

6. Most common site of origin of adaruantinoma is:


(a) Mandi ble near molar tomlt
(b) Middle a lveolar margi ns
( c) Hard palate
(d) Mandible near symphysis men ti.
7. Test for tight iliotibial band is:
(a) Ober's test
(b) Osbcr's lest
(c) Sirmnand 's test
(d) Charnley" test
8. Bone apposition is best in:
(a) Osteoblastic activity ar lhe area of su-ess
(b} Endochondral ossification
(c) Subperiosteal cambian layer
(d) Osteobla.stic activity in Howshi p's lacunae

ANSWERS
1.
2. 3. 4.(a)
(a)
(b) (b) 8. (c)
5.
6. 7. (a}
(b) (a)

210
MAY 2001
1. A 20 year old male, following a road traffic accident
was brought to the casualty. His right leg is shortened,
internally rotated and adducted. The diagnosis is:
(a) fracture neck of femu r
(b) Anterior dislocation of Lhe hip
(c) Poterior d islocat ion of the hip
(d) Trochanteric fractuTe of the femu r.

2. An 8 year old boy presents with a grad ually


progressing swelling and pain since 6 months over
the. upper tibia. On x-ray, there is a lytac lesion with
sclerotic ma rgins in the upper tibial metaphysis.The
diagnosis is:
(a) Osteogenic saracoma
(b} Osieoclastoma
(c) Brod ie's abcess
(d} Ewing's sarcoma

3. An 8 year old boy wiith a histo,·y of fall from HI


feet height complains of pain in the right ankle.X-ray
taken at that time are normal without any fracture
line.liut after 2 years, he developed a calcaneovalgus
deformity. The diagnosis is:
(a) Undiagnosed malunitcd fra<.:t ure
(b) Avascular necrosis tal us
(c) Tibial epiphyseaJ injury
(d) Ligamentous injury of ankle joint
4. A 6 year old boy has a history of recurrent dislocation
of the right shoulder. On examination, the orthopaedi
cian puts the patient in the supine position and abducts
his arm to 90 degrees with the bed a the fulcrum and
then externally rotates it but the boy doesnot allow the
test to be performed. The test done by the
orthopaedkian is:
(a) Apprehension test
(b) Sulcus te t
(c) Dugas lest
(d) McMurray' test
5. Regarding bone remodeling , all are true except:
(a) Osteoclastic act ivity ,ll t he compression site
(b) Osteoclastic activity at t he tension site
(("} Osteocl astic act ivity and osteohlastic aclivi t) arc both
needed for bone rcmodeLingin co11ical and
cancel lous bone
(d) Osteoblasts transfo1ms into osteocytes.
6. A child is spinned around by holding bis hand by his
father. While doing this, the child started cr)ing and
doesnot allow his father to touch hiselbow. The
diagnosis is:
(a) Pulled elbow
(b) Radial head di,location
(c) Ann u lar ligament tear
( cf) fracture olecrano:n process

7. An 18 years old boy was playing football when he


suddenly twisted his knee on the ankle and fell do\\>,i.
lie got up afler 10 minutes and again started playing,
but next day, hisknee wasswollen and hecouldnot
move
it The most probably cauase is:
(a) Medial meniscus rear
(b) Anterior cruciate ligament tear
(c) Medial C()llateral ligament injury
(cf) Posterior cruciate ligament injury.

ANSWERS
J . (c)
2. 3. (c) 4. (a)
5. (c)
(a}
7.
6. (a)
(a)
NOVEMBER 2000
1. Posterior dislocation of hip leads to which attitude of
the lower leg:
(o) Adduction. external rotation mid
flexion (b} Adduction.internal l'Otation
and flexion tc) Abduction , externa l
rotation a11d flex1on Cd) Abduction,
internal rotation and flcxion
2. After falling from a height, a child lands on his fulJy
out-stretched hands. On examination there is pain and
swelling over his right elbow. Give your proba ble
diagnosis:
(a) Frnc.turc olecranon
(b) Posterior dislocation of elbow
(c) Suptacom ylar fracture or humerus
(tf) Fracture both bone; foreann

3. A patient has 2 months POP cast for tibial fracture


of left leg. Now he needs mobilization with a sinle
cru tch. You will use this crutch on which side:
(a) Left side
(b) R ight side
(c) Any side
(d) Both sides.
4. A child 10 yea rs of age presents with a mass on his left
thigh. The mass seems to be arising from the diaphysis
of'femur and involving the sort tissue of thigh. The
child is having fever also.Give your most probable
diagnosis:
(a) Ostcosarcoma
(b) Ewing's sarcoma
(c) Chondmsarncoma
(d) Malignant fibrous histiocywrna

ANSWERS
213
1. (b) 2. (c) 3. (b) 4. (b)

213
JUNE 2000
l. Positive pivot shift test in knee is because of injury to:
(a} Po,sterior crnciatc ligamcm
(b) Anterior cruciate ligamem
(c) Medial collateral ligament
(d) Posterior elbow ligamenL
2. Three point relationship is reversed in all except:
(o) # medial epicondyle
(b) # lateral cpicondylc
(<') Supracondylar #
(d) Posterior elbow dislocation

3. A patient with leprosy presents with clumsiness of


band. His ulnar nerve is affected. Clumsiness is due to
paJ.5y of which muscle
(a) Extensor carpi ulnaris
(b} Abd uctor pollicis brevis
(c) Opponcns pollicis
(d) lnterosseous muscle

4. What is Kien bock's disease:


(o} Osteochondritis of lunate
(b) Osteohondritis of neck of femur
(c) Osceochondritis of navicular
(d) Osteochondritis of tibial condyle

5. A patient developed paraplegia. On routine examina


tion and x-ray it was found that there was an
osteoblastic lesion in his spine. lvfost probable
diagnosis is:
(a) Carcinoma thyroid
(b) Ca. Porstate
(c) Breast Carcinoma
(d) Pancreatic Carcinoma

214
6. A patient with pain in back. Lab investigation sbows
an elevated ESR X-ray skull shows multiple
punched out lytic lesions. Most important
investigation to be done is:
(a) Serum acid phosphatsae
(b) CT head witb contrast
(c) Wholt: body scan
(d) Serum electrophoresis

7. Which of the following is seen in osteoporosis:


(a) Low Calcium, high P04, high alkaline phosphatase
(b) Low Ca. low P04. low alkali ne phophata se
(c) Normal Ca. normal P04. nom1a l alkali ne
phosphatase
(d) Low Ca. low P04. nom1al alka line phosphatase.
8. A person is not able to extend his metacarpophalangeal
joint. This is due to injury to which nerve:
((I) Ulnar nerve
(b) Radial nerve injury
(c) Median nerve injury
(d) Post. lmerosseous nerve injury
9. What is the most common site of osteosarcoma:
(a) Lower end of femur
(b) Upper end or humerus
(c) Lower end of ti bia
(d) Upper end of femu r
10. Lytic bevelled lesions are s1'.t!n in skull x-ray. Most
likely cause is:
(a) Multi ple myeloma
(b) Eosinophilic grnnu loma
(c) Metastasis
(d) Osteosarcoma
11. True regarding osteosarcoma is:
((I) Occurs because of proliferation of os1coclas1s

215
(b) Sunray appearance in x-ray indica tes new bone
formation

216
(c) Affects mainly mafes in the 5111 or 6., decade
(d) Lymphatic metastasis is most t:om mon
12. Bankart's lesion is St>en at:
(a) PoLerior surface of glcooid labru m
(b) Anterior surface of glenoid labrum
(c) Anterior part of head of humerus
(d) Posterior part of head of hu merus

ANSWERS
1. (ii)
2. (c) 3. 4. la)
5. 6. (ti} (d) 8. (d)
(/>) 9. 7. (cl
10. (a) 12. ( b)
(a)
11. (b)

NOVEMBER 1999
1. A II year old child presents with fracture lnunerus after
trivial inju ry,x-ray shows lytic lesion. Diagnosis is:
(a) Ostcolastoma
(b) Ostcomyelitis
(c) Unicameral bone cy,1
(d) Ancurysma l bone cyst
2. On accident there L damage of cervical spine, first line
of management is:
(a) X-ray
(IJ) tum head lo side
(c) mainta in airway
(d) s1abili7,e the cervical spine
3. Deformity in the anterior cllslocation of hip is
(a) external rotmion. abducti()n, flexioo
(h) external rotation. adduclion , flel<lon
(c:) internal rotation , abduction. ne)(ion
(d) internal rotalion , adduction. nexion
4. Stellate ganglion block is useful in:
(aJ Sudeck·s osteodystrophy
(b) Compound pal mar ganglion
(c) Tenosynovi Lb
(J) Osteoarthritis of first CMC joint

5. A lO year old male with pain in the left hi p and limping


on examina tion restricted abduction and inter
nal rotation, proba ble diagnosis is:
(a.) septic anhritis of hip
(c) tuberculous arthriti of hip
(h) congenital dislocation of hip
(r./) Perthe's disease
6. Commonest dangerous complica tion of poster ior
dislocation <if k nee is:
(a) Popliteal artery injLu)'
(b) SciaLic nerve injury
(c) lsclmcmic of lower leg compartmcrH
(d) Femoral artery injury.

7. A 3 year old male presented with progressi ve anaemia,


hepatosplenomegaly and osteomyelitis of jaw with
pathological fracture, x-ray shows chalky white deposi
t!. on bone, prnbable diagnosis is:
(a) Osteopetrosi$
(b) Osteopoikilocytosis
(c) Alkaptonuria
(ti) Myositis-ossillcans progressiva

8. Most common cause of VIC in children is due to:


(a) Supratondylar # humeru
(h) Lateral condyle II humerus
(c) Medial condyle ii htuncms
(a) both bone foream1#
9. True about bone tumour is:
(a) Multiple myeloma - more than 55 yeru·s age ,uid
abow
(b) Osteogenjc sarcoma - fou1th decade
(c) Chondrosaracoma - first decade
(d) Ostcoclastoma - Ji fth decade

10. Line joining anterior supe1·ior iliac spine to ischial


tubcrosity and passes a greater trochaoter:
(a) Nclaton's liac
(b) Slloemaker's
line (,) Cniene's
line
(d) Perkin's line

ANSWERS
l. (c) 2. (d) 3. 4. (al
5. 6. (a)
(d) 8. (a)
(a) 7.
9. 10. (a)
(u)
(a)

JUNE 1999
1. A 40 yea r old patient having arthritis of PIP and DIP
a.Jong with carpornetacarpaJ joint of thumb and
sparing of wrist and metacarpophalangeal join t, most
likely
diagnosis is:
(a) Rheumatoid arthritis
(b) Osteoanhritis
(c) Psoriatic anhritis
(d) Pseudogou l
2. Traumatic anterior dislocation of shoulder ·with
sensory loss in lateral sideof forearm and weakness
218
of flexion of elbow joint, most likely injured nerve is:
(a) Radial nerve
(b) Ulnar nerve
(c) Axi llary nerve
(ri) Musculocu taneous nerve

219
3. A 65 year old female presented with # of Tl2 vertebra
with increased alkaline phosphalllse and paratbormone
level along with low ,calcium and low phosphate level.
diagnosis is:
(a) Osteoporosis
(b) Pagers disease
(r) Primary hyperparathyroid i,m
(d) Vitamin D deficiency
4. A 6 year old child has anaccident and had # elbow,
after four yearsa presented with tingling and nu
mbnes1, in the ulnar side of finger, fracture is:
(a) Suprn<:ondylar # of humcrns
(b) Laternl condyle # of humerus
(c) Olecranon#
(d) Dislocation of elbow
5. A 50 years old male with fracture neck of femur
come$ after 3 days, treatmeut of choice is:
(a) Hemiartbroplasty
(b} Total hip replacement
(c) Hip spica
(d) CR and LF
6. Fracture of both bones forearm at lhe same
level. position of the arm in plaster is:
(a) fuU supination
(b) JO degree supinatioo
(c) full pronation
(d) mid-prone
7. Treatment of choice in a 65 yea r old female with
impacted # neck of humerus is:
(a) Triangular sLing
(b) Ann chest strapping
(c) A,throplasty
(d) Observatjon
8. A bsol u te indication of X-ray of pelvis in case of
congenital dislocation of hip arc all except:
(a) Positive farniJy hiMory
(b) Brecch presentalion
(c) Unstable hip
(d) Shortening of limb

ANSWERS
I. 2. d 3. d 4. b
b 6. d 7. a 8. b
5. d

22
0
ALL INDIA ENTRANCE EXAMINATION
2003
I. Commonest cause for neuralgic pain in the foot is:
(a) Compression of c-0mrnu nication between medial ,
1nd lateral plantar nerves
(h) Exaggeration of longiludi nal arches
(c) Injury to deltoid ligament
(d) Shortening of plautar aponeuros,,
2. ArnscuJar necrosis can be a possible sequelae of
fracture
of all of the following bones except:
(a) Femur neck
(b) Scaphoid
(c:) Talui.
(d) Calcaneum

3. Kicnbock's disease is due to avascular necrosis of:


(a) Femoral neck
(b) Med ial cuncifonn bone
(cl Lunate bone
(d) Seaphoid bone

4. A classical expansive lytic lesion in the transverse


proCL'SS of a vertebra is seen iin:
(t1) Osteosarcoma
(b) AnellryS!llal bone cyst
(c) Osteoblastoma
(d) Metasta i
5. A 30-year old male was brought to the ca ;ualty
following a road-traffic accident. His ph ysical
examinat ion revealed that his right lower limb was
short. internally rotated and flexed and adducted at
thl! hip. The most likely diagnosis is:
(a) Fracture neck of femur
(h) Trochantcric fraclure
221
(c) Central fracture dislocation of hip
(d) Poterior dislocatiolll of the hip
6. Sciatic nerve palsy may occur in the foUowing injury:
(a) Posterior dislocation of hip joim
(b) Fracture neck of femu r
(c) Trochanteric fracture
(d) Anterior u islocation of hip
7. Osteomalacia is associated with:
(a) Decrease i n osteoid value
(b) Decrease i n ostcoid urface
(cl Increase in osteoid marurmion time
(d) Increase i n mineral apposition ra.te

8. Which one of the foUowiug tests will you adopt while


examining a knee joint where you suspect an old tear
of anterior crnciate ligament?
(a) Posterior drawer tes t
(b) Memurray's test
(c) Laclunan ·s test
(d) Pivot shift test

9. A 64 year old hypertensive obesefemalewas


undergoing surgery for fracture femu r under general
anaesthesia. Intra-operat ively her end-tidal carbon
dioxide decreased to 20 fro111 40 mm of Hg followed
by hypotension and oxygen satu ration of 85%. What
could be the mos! probable cause?
(a) Fat embol ism
(b) Hypovolemia
(c) Bronchospasm
(d) Myoca rdial infarction
10. loactinomycosis of the spine, the abscess usually
erodes:
(a) Incervertebral disc
(b) lnto the p leural cavity
(c) Imo the retroperitoncal space

222
(d) Towards the skin

223
ANSWERS
1. (a)
2. 3.
s. (d) (c}
4. (b)
8. (C)
(d) 6. 7. (r)
9. (a) (a)
10. (d)

2002
1. Carpal tunnel syndrome is due to compression of:
(a) Radial nerve
(b) U lnar nerve
(c) Palmar branch of t he ulnar nerve
(d) Median nerve

2. Most common nerve invol ved in the fracture of


surgical
neck of humerus is:
(a) Medim1 nerve
(b) Radial
(c) Ulnar
(d) Axillary

3. All of the following are associated wilh supracondylar


fractu re of humerus Except:
(a) Tt is u nconunon after L5 years of age
(b) Extension lYPc:: fractu re is more common than the
flexion type
(c} Cubitus varus defom1ity commonly results following
rnalunion
(d) Ulnar nerve is most commonl y involved
4. Ramesh Singh, a 40 yea r old man, was admitted with
a fracture shaft of fem ur,following a road traffic
accident. ON the 2""day, he became disoriented, he
was found to be tachypnoeic and had conjunctiva!
petecbiae.
(a) Pu lmonary embolism
(b) Sepsis syndrome
(c} FaL embolism
(cl) Haemothorax

5. Kuma r, a 31year old moton.:ydist sustained injur) over


his righ t hipjoint, X-ray revealed a posterior
dislocation of the right hipjoint. The clinical altitude
of the affected lower limb will be:
(a) External rotation, extc,nion and abduGtion
(b) Internal rotation. llexion and adduc1ion
(t) Internal rotation, extension and abduction
(d) External rotation. flexion and abduction.
6. Pappu, a 7 years old young hoy, had fractun of' the
lateral condylc of the femur. He developed malunion a
the fracture was not reduced anatomically. Malunion
will produce
(a) Genu valgu m
(bl Gcnu varum
(c) Gcnu recurvatum
( dl Dislocation of the knee
7. Pa tella r tendon bear ing POP cast is indicated in t
he following fracture:
(a) Patella
(/,J Tibia
(C') Med ial MaJJeolus
(cl) Femu r
8. Inversion injury al t he an kle can cause all of the
following except:
(a) 1-'racture tip of lateral malleolus
(bl Fracture base of the 5''' mcratarsa l
(c) Sprain of Ext. Digitorum brevis
(d) Fracture of the sustentacul um tali
9. A previously healthy 45 year old labourer suddenly
develops acute low back pain with the right leg pain
and
weakness of dorsiflexioo of the right great toe.Which of
the following is true:
(a) Jmrnedjate Lreatmen t should i ncl udeanalgesics.
muscle re laxants and back :.u-engthen ing exercises
(h) The appearance of the fooldrop i ndicate caJly surgical
intervcmion
(c) ff the ncUJological sign resolve withi n 2 or 3 week, hut
low back pain persists, the proper treatment would
include fusion of affected lumbar vertebra.
(d) Lf thc neurological sign.fail Lo resolve within one
week. lumbaJ lamineetomy and ex.;ision of an y
herniated nucleus pulposus should he clone
10. Acute osteomyelilis is most commonly caused by:
(ii) Staphylococcus aureus
(b) Actinomyces bovis
(c) Nocardia asteroids
(d) Borrelia vincenlii

11. A 45 year male presented with an expansilc lesion in the


cen ter of the femoral meta1>hysis. The lesion
shows endosteal scalloping and punctate
calcifications. Most likely diagnosis is:
(a) Osteosarcorna
(b) Choodrosaracoma
(l") Simple bone cyt
(d) Fibrous dysplasia

12. Raju,a 10 year old childl presents with predisposition


to fractures, anemia, hepatosplenomegaly and a
diffusely increased radiographic density of bones. The
most likely diagnosL is:
(a) Osteogene$is impcrfecrn
(b) Pyknodysostosis
(c) Myelofibroi
(d) Ostcopctrosis

22
5
13. Harivardhaman, 9years old child presents with
scoliosis, hairy tuft in the skin of the back and
neurological deficit. Plain x-rays reveal multiple
vertebral anomalies aod a vertical bony spur overlying
lumbar spine on AP view The most probable diagnosis
is:
(a) Dorsal dermal sinus
(b) Diastometamyclia
(c) Ti2h1 rtlum tem1i nale
(ti) Caudal regression syndrome
14. Ina patient with head in.jury , unexplained
bypotension
warr.lllts evaluation of
(a) Upper cervical spine
(b) Lower cervica l spine
(c) Thoracic: spine
(d) Lu mbar spine
15. Complete transection of the spinal cord at the C7
level produc all of the following effects except:
(a) Hypoteosion
(b) Limited respiratory effort
(c) Anaesthesia below the level of the lesion
(d) A·retlcxia below the level of the lesion

ANSWERS
1. 2. 3. 4. (c)
(d)
(d) (d) 8. (c)
5.
(b)
6. 7. (b) u. (d)
(a) 11.
9. 10.
(b) (b>
(a) 15.
13. 14. (b)
(b) (c)
226
2001
1. Following anterior dislocation of the shoulder a patient
develops wea k ness of flexion at elbow and lack of
sensation over the lateral aspect of the forearm, nerve
inju red is:
(a) Radial nerve
(b) Musculocutaneou s nerve
(c) Axlllary nerve
(d) Ulnar nerve
2. Babloo, a 10 year old boy present witl1# of bun1eru. .
X-ray reveals a lytic ksion at the u1>pci: end. Likel)
condition is:
(a) Unicameral bone eysl
(t,) Osteosarcoma
(c) Osteoclasloma
(d) Aneurysmal bone cyst
3. A patient sustained injury to the upper limb 3 years
back,he now presents 'lith valgus defonuity intlte
clbo\\ and paraesthesia s over the mcdiaJ border of
the ha nd. The injury is likely to have been:
(a) Supracondylar # humerus
(/J) Lateralcondyle # of humerus
(c) Medial condyle # of humerus
(d) Posterior dfalocation of the humerus
4. A woman aged 60 years suffers a fall, her lower limb
is extended and externally rotated,likely diagnosis
is:
(a) Neck of femur #
(b) lntertrochantcric femur #
(c) Pvsterior dislocation of hip
(d) An terior dislocation of hip
S. Triplt! arthn>desis involves
(a) Calcane.ocuboid. talonavicuJar and talocakaneal
227
(b) Tibiota lar. calcaneocubmd and wlonavicula r
(c) Ankle join1. C'akaneocuhoid and talona\'icu lar
(d) None of the above.

6. Babu, a J 9 year old male has a :;mall circumscribed


sclerotic swellin.ci over diapbysis oJ' femu r. l ikely
diagnosis is:
(a) O,teoclaswma
(b) Os1eoso1rac11mu
(c) Ewing's san:oma
(d) Osteoid osteoma

7. Most common site of ostcogenic sarcoma is:


(o) Femur. upper end
(b) Femur. lower end
(c) Tibia. upper
(d) Tibia. lower end

8. Invol vement of PIP join t, DlP joint and the carpo


metaca r pophala ngeal joi nt of base of thu mb with
sparing of the wrb1is seen in:
(n) Rheu matoid arthritis
(b) Osteoarthrit is
(c) Psoriatic arlliritis
( cf) Pseudogout
9. The pivot test is for:
(a) Anterior cruciate ligan1en1
(b) Posterior cmcime ligamen t
(c) Medial meniscus
(d) Lateral meniscus

ANSWRRS
1. (b)
2. ( 3. 4. (a)
5. (a) 11) (b)
9. (o) 8. (/J )
6. 7. (bl
(d)
228
2000
1. Treatment regariling fractu re of clavicle is:
(a} Most common complication is maluninn
(b} Occurs at the ju nction of medial 1 /3"' a nd latcr,11
:i.nrds (c} U ually occurs due to fall on elhow
(d) Comminu tcc.l fracture i,common

2. A boy fell down from a tree and has fracture neck of


humerus. He cannot raise bis arm because of the
invol vement of:
(a) Ax_ilJary nerve
(hJ Supraspinams nerve
(c) Muscul<Kutaneous nerve
(d) Radi l nerve
3. Incllildren. all are true except:
(a) Dislocations arc rare
(b} Co111minu1ed fractures arc common
(d Thick pcriosteum
(d) Sol'l bones
4. An 8 year old child is brought by parents to the
casualty with a spiral fractu re of feur and varying
degree of ecchymosis all over the body. The etiology
is:
(a) Hit and run acciden t
(b) Ba!tered baby syndrome
(c) Hockey tick inju1-y
(d) Fall from hei ght
5. Tremlelenberg 'ssign is negative in an inler-
t..rochantcric fracture because of:
(a) Gluteus mcdius
(b) Glmcus
maximus k} Gluteus
minimus Cd) Tcsor
fascia laca
229
6. Post-poliomyeli tis. a patient bas grade rr power in
Gastrocnemius, grade ill in peroneus, Grade lV in
Tibialis anterior. The deformity is:
(a) Calcaneoval1',1, ls
(b) Equinovarus
(c) Calcaneovarus
(d'J Genu valgus
7. Decreased mineralization of epiphyseal pla te in a
growing child is seen in
(a) Rickets
(b) Osteomalacia
(c) Scurvy
(d) Osteoporosis
8. Cbondroblastoma is ii tumour of:
(a) Epiphysis
(b) Metaphysis
(c) Diaphysis
(ti) Flat bone

9. Chordoma can occur over all the following sites except:


(a) Rib
(b) CLivus
(c) Sacnun
(d) Vem:bral body

ANSWERS
1.
(a)
2. (a) 3. 4. (b)
6. (c) (b) 8. (a)
5.
(d) 7. (a}
9. (a)

230
PGI ENTRANCE EXAMINATION
JUNE 2003
1. A 18 year old boy presented in OPD with Lt. Mono
articular knee pain. Poi;sible diagnosis is/are:
(a) Gout
(c) Osteoarthritis
(c) Rheu matoid arthritis
(d} Reiter's djseasc
(e) Gonococcal anbritis

2. X-ray feature of ostcosarcoma:


(a) Sun-ray appearance
(b) Codman's triangle
(c)Soap bubble appearance
Onion peel appearance
(ti)
(e) New bone fonnation
3. Topbi of gout is found in:
(11) Articular canilage
(h) Synovium
(c) Skin
(ti) Muscle
(e) Joint capsule

4. Treatment of histologically confirmed giantcell tumour:


(a) Excision. bone grafung and chemical cautery
(b) Excision
(c) Radiotherapy
(If) Chemical excision
5. Clinical featun.'S of T.B. spine are all except:
(b) Loss of lordosis
(b) Night sweal$
(c) Weight gain
(d) Evening rise of temperature
(e) decreased appetite
231
6. Usual site of tubercular bursitis:
(a) Prepatellar
(h) Suhdelloid
(c) Subp,llellar
(d) Trochanteric
(e) None
7. Rocker bottom t'eet i;; seen in:
(a) Congenital verticn.l talLL
(b) CTEV over correction
(c) Grice procedure
8. Treatment of displaced transverse f'rach1re of patella:
(a) POP
(b) Tension hand wiring
(c) Screw
(d) Patellectomy
9. True aboul non-ossifying libroma of bone:
(a) Present u ntil 3"' and lourtJ1decade
(b) Eccentric
(c) Prominent scle.rmic margin
(d) fli tologically giunt cell with areolar tissue
(e) Metaphyscal lesion

ANSWERS
1. (d. e)
2. (11. h. 3. (a. h. c. e) 4. (a. b. (')
5. (c:) e) 7. (a, b) 8. !o. b. c. dl
9. (b, c. d. 6. ( e)
e)
DECEMBER 2003
I. Bone formi ng tumours are:
{a) Osrcosarcoma
{/JJ Osteoid O$leoma
(c) Giant cell tumour
(d) Ostcoblastoma
<e) Chondrosarcoma
2. Benign bone tumours are:
(a) Ostc;:oid ostcoma
(b) O$teochondro111a
(c) Cho11drobl ast0ma
(d) Chondrnmyxoid libroma
(e) Multiple mycloma
3. All are predisposi ng factors ol' osteogcnic sarcoma
except:
(a) Paget's disease of bone
(bJ Radiation
(c) Viral infection
(d) Bone infarction

4. Factors that promotes callus formation:


(a) Paget's disease of bone
(b) Radiation
(c:) Viral infect ion
(d) Bone infarction
S. ln the management of long bone fracture following can
be done:
(a) lntramcduJlary naili
ng (b) Plating
(c) External fixation
(d) Tension band wiring
(e) Screw
6. Poor prognostic factors in Pott 's
paraplegia:
(a) Acute onse1 of paraplegia
(b) Sudden progression of paraplegia
(c) Motor paralysis alone
(d) Long standing paraplegia
(e) Paraplegia i n children

7. Differential diagnosis of simple bone cyst are:


(a) Giant cell 1wnour
(b) Non ossifying fibroma
(c) Enchond ronia
(cl) Fibrous dysplasia
(e) Eosi noph i l ic granuloma
8. True about osteoa rthritis except:
(a) Commonly found in adult before 50 years
(b) Heberden' s nodules are found
(c) si ngle joint involvemenl
(cl) Lower limb defonn.ity is seen
(e) Anky losis

9. True about carpal tunnel syndrome:


(a) Paraesthesia of tl1e lateral 2 fingers
(b) Alrophy of the lhenar eminence
(c) Atrophy of the hypotbenar eminence
(d) Clow hand
(e) Tine!sign is positive
10. True about osteogenesis imperfecta:
(a) Autosomal domimmt
(b) Known as m arble bone disea:;e
(c) Blue sclera
(d) Associated with otosclerosis
(e) Defect in collagen- I

234
U. True about ganglion:
(a) Common in vol ar aspect
(b) Seen adja cem 10 the 1endo11 sheath
(c) Communicates with joi nt cavity and tendon sheath
(cf) It is u nilocular

ANSWERS
1. (a. b, d) 2. (a. b. c.
d) 3. (cl 4. (a. /J )
5. (a, b, c. e) 6. (b. d) 7. (a. b, d. 8. (al
e)
9. (b. e) 10. (a. b, d.e)
8. (b. e)
NATARAJ AN 'S
A Question Bank of
M ultip le Choice estions in
ORTHOPAEDICS
Qu Of.lal TRAUMATOLOGY
As an acaclemiciru, the latest t(enas in the examination system of
medi•lstudents has always be<!n a fascinating study. l ence I have been
following U1e advent of MCQs 1n10 the medical examinalmn s.ystem wi1h g
eat interest. Multiple Choice QueWoos have evo)ved from a stage obtu!fie:d
dJSdain to grudging acceptance to a stage where 1hey have become
ub1qm1ous.Thir value ,n objectwe testmy ol lh• student's grasp oi Ule
$ubject has been widely,i!cknowledged by aCildemicians the world over. So ii is
not surprising 10 s that 1 ey are now so entrenched In 1he system Ula1 lho
UliQer-graduate medlC;il students becomesfamiliar with them from hkfirs1year
itself and is rid of them only after his super-sp-eciality da are bver.
As rt hai always been my endeavour to help students learn the asc,nating subject
of Orthopaed1 lrorn all angels; I sco.ur thi, various books available for
MCQs in Orthopaedics.but Ule dearth of il single.authentic qu01>uon bank for
MCQs 1•,as so gli!ring that 1t prodded me 10 bring out tbe first edl1ion of "A
Quions Ban or Mulllple Choice Question in Orthopaed ics and Traumatology•
This. t hope, wtU addrewthe long·felt need for an au1hentic, researched book
on MCQs for Or\hopaedics.The que1tions have been drawn from varmus
oxammat1ons nducted all over India.Thequestions have been soregated that
theyare arrang,,d
In various chapters corresponding to 1he various topics In "Orthopaedics
and Traumatology•·.The answe have ben drawn from the author's
"Textboo.k of Orthopaedics and Traum_1itology'' and In case of doubt, the
;tudent ,s advised to refer to the book. An added featuce of this book, is
that luncorrates tl,elatest M<.!.ls.Wh,ch ISBN 81 800'1 009·7
have been

III
asked in tl'te recent papers of All !NOIA, AIIMS,
PGI Post-graduate Entranc,a Ex;ims.Thii book is
aimed to satisfy the needs or not onJy the
lfndergraduates preparing for1heir Orthopaedic
Paper of their PG Entrance Exam, bot also 1he
Post-gradu,att's who are preparing fo, their exams. 9 788180 D4DD9c

You might also like